You are on page 1of 150

Chuyn

S HC
Din n Ton hc
Chuyn
S HC
Ch bn
Trn Quc Nht Hn [perfectstrong]
Trn Trung Kin [Ispectorgadget]
Phm Quang Ton [Phm Quang Ton]
L Hu in Khu [Nesbit]
inh Ngc Thch [T*genie*]
c _ 2012 Din n Ton hc
Li gii thiu
Bn c thn mn,
S hc l mt phn mn quan trng trong ton hc gn b vi
chng ta xuyn sut qu trnh hc Ton t bc tiu hc n trung hc
ph thng. Chng ta c tip xc vi S hc bt u bng nhng
khi nim n gin nh tnh chia ht, c chung ln nht, bi chung
nh nht... gip lm quen d dng hn vi s k diu ca nhng con
s cho n nhng vn i hi nhiu t duy hn nh ng d, s
nguyn t, cc phng trnh Diophantine m ni ting nht l nh l
ln Fermat..., u u t tm vi m n v m, t cu b lp mt bi
b 4 chia ht cho 2 n Gio s thin ti Andrew Wiles (ngi gii
quyt bi ton Fermat), chng ta u c th thy c hi th ca S
hc trong .
S hc quan trng nh vy nhng l thay s chuyn vit v n li
khng nhiu nu em so vi kho tng s cc bi vit v bt ng
thc trn cc din n mng. Xut pht t s thiu ht cng nh
k nim trn mt nm Din n Ton hc khai trng trang
ch mi (16/01/2012 - 16/01/2013), nhm bin tp chng ti cng vi
nhiu thnh vin tch cc ca din n chung tay bin son mt
chuyn gi n bn c.
Chuyn l tp hp cc bi vit ring l ca cc tc gi Nguyn Mnh
Trng Dng (duongld) , Nguyn Trn Huy (yeutoan11), Nguyn
Trung Hiu (nguyentrunghieua), Phm Quang Ton (Phm Quang
Ton), Trn Nguyn Thit Qun (L Lawliet), Trn Trung Kin (Is-
pectorgadget), Nguyn nh Tng (tungc3sp)... cng s gp sc
i
ii
gin tip ca nhiu thnh vin tch cc trn Din n Ton hc nh
Nguyen Lam Thinh, nguyenta98, Karl Heinrich Marx, The
Gunner, perfectstrong...
Kin thc cp trong chuyn tuy khng mi nhng c th gip
cc bn phn no hiu su hn mt s khi nim c bn trong S hc
cng nh trao i cng cc bn nhiu dng bi tp hay v kh t cp
d n cc bi ton trong cc k thi Hc sinh gii quc gia, quc t.
Chuyn gm 7 chng. Chng 1 cp n cc khi nim v c
v Bi. S nguyn t v mt s bi ton v n c gii thiu trong
chng 2. Chng 3 ni su hn v Cc bi ton chia ht. Phng
trnh nghim nguyn, Phng trnh ng d c phc ha trong
cc chng 4 v 5. H thng d v nh l Thng d Trung Hoa
s c gi n chng ta qua chng 6 trc khi kt thc chuyn
bng Mt s bi ton s hc hay trn VMF chng 7.
Do thi gian chun b gp rt ni dung chuyn cha c u t
tht s t m cng nh c th cn nhiu sai st trong cc bi vit,
chng ti mong bn c thng cm. Mi s ng h, ng gp, ph
bnh ca c gi s l ngun ng vin tinh thn to ln cho ban bin
tp cng nh cho cc tc gi nhng phin bn cp nht sau ca
chuyn c tt hn, ng gp nhiu hn na cho kho tng hc
thut ca cng ng ton mng. Chng ti hi vng qua chuyn ny
s gip cc bn tm thm c cm hng trong s hc v thm yu v
p ca nhng con s. Mi trao i gy xin gi v a ch email :
contact@diendantoanhoc.net.
Trn trng,
Nhm bin tp Chuyn S hc.
Din n Ton hc Chuyn S hc
Mc lc
i
Li gii thiu
1
Chng 1
c v Bi
1.1 c s, c s chung, c s chung ln nht 1
1.2 Bi s, bi s chung, bi s chung nh nht 4
1.3 Bi tp ngh 6
9
Chng 2
S Nguyn T
2.1 Mt s kin thc c bn v s nguyn t 9
2.2 Mt s bi ton c bn v s nguyn t 13
2.3 Bi tp 19
2.4 Ph lc: Bn nn bit 24
29
Chng 3
Bi ton chia ht
3.1 L thuyt c bn 29
3.2 Phng php gii cc bi ton chia ht 31
57
Chng 4
Phng trnh nghim nguyn
iii
iv Mc lc
4.1 Xt tnh chia ht 57
4.2 S dng bt ng thc 74
4.3 Nguyn tc cc hn, li v hn 86
89
Chng 5
Phng trnh ng d
5.1 Phng trnh ng d tuyn tnh 89
5.2 Phng trnh ng d bc cao 90
5.3 H phng trnh ng d bc nht mt n 90
5.4 Bc ca phng trnh ng d 95
5.5 Bi tp 95
5.6 ng dng nh l Euler gii phng trnh
ng d 96
5.7 Bi tp 101
103
Chng 6
H thng d v nh l Thng d Trung Hoa
6.1 Mt s k hiu s dng trong bi vit 103
6.2 H thng d 104
6.3 nh l thng d Trung Hoa 117
6.4 Bi tp ngh & gi p s 125
129
Chng 7
Mt s bi ton s hc hay trn VMF
7.1 m
3
+ 17
.
.
.3
n
129
7.2 c(ac + 1)
2
= (5c + 2)(2c +b) 136
141
Ti liu tham kho
Din n Ton hc Chuyn S hc
Chng
1
c v Bi
1.1 c s, c s chung, c s chung
ln nht 1
1.2 Bi s, bi s chung, bi s chung
nh nht 4
1.3 Bi tp ngh 6
Nguyn Mnh Trng Dng (duongld)
Nguyn Trn Huy (yeutoan11)
c v bi l 2 khi nim quan trng trong chng trnh s hc THCS.
Chuyn ny s gii thiu nhng khi nim v tnh cht c bn v
c, c s chung, c chung ln nht, bi, bi s chung, bi chung
nh nht. Mt s bi tp ngh v cc vn ny cng s c
cp n cui bi vit.
1.1 c s, c s chung, c s chung ln nht
Trong phn ny, chng ti s trnh by mt s khi nim v c s,
c s chung v c s chung ln nht km theo mt vi tnh cht ca
chng. Mt s bi tp v d cho bn c tham kho cng s c a
ra.
1.1.1 nh ngha
nh ngha 1.1 S t nhin d ,= 0 c gi l mt c s ca s t
nhin a khi v ch khi a chia ht cho d. Ta ni d chia ht a, k hiu d[a.
Tp hp cc c ca a l: U(a) = d N : d[a.
1
2 1.1. c s, c s chung, c s chung ln nht
Tnh cht 1.1 Nu U(a) = 1; a th a l s nguyn t.
nh ngha 1.2 Nu U(a) v U(b) c nhng phn t chung th nhng
phn t gi l c s chung ca a v b. Ta k hiu:
USC(a; b) = d N : (d[a) (d[b)
= d N : (d U(a)) (d U(b)).
Tnh cht 1.2 Nu USC(a; b) = 1 th a v b nguyn t cng nhau.
nh ngha 1.3 S d N c gi l c s chung ln nht ca a v b
(a; b Z) khi d l phn t ln nht trong tp USC(a; b). K hiu c
chung ln nht ca a v b l UCLN(a; b), (a; b) hay gcd(a; b).
1.1.2 Tnh cht
Sau y l mt s tnh cht ca c chung ln nht:
Nu (a
1
; a
2
; . . . .; a
n
) = 1 th ta ni cc s a
1
; a
2
; . . . ; a
n
nguyn
t cng nhau.
Nu (a
m
; a
k
) = 1, m ,= k, m; k 1; 2; . . . ; n th ta ni cc
a
1
; a
2
; . . . ; a
n
i mt nguyn t cng nhau.
c USC(a; b) th
_
a
c
;
b
c
_
=
(a; b)
c
.
d = (a; b)
_
a
d
;
b
d
_
= 1.
(ca; cb) = c(a; b).
(a; b) = 1 v b[ac th b[c.
(a; b) = 1 v (a; c) = 1 th (a; bc) = 1.
(a; b; c) = ((a; b); c).
Cho a > b > 0
Nu a = b.q th (a; b) = b.
Nu a = bq +r(r ,= 0) th (a; b) = (b; r).
Din n Ton hc Chuyn S hc
1.1. c s, c s chung, c s chung ln nht 3
1.1.3 Cch tm c chung ln nht bng thut ton Euclide
tm (a; b) khi a khng chia ht cho b ta dng thut ton Euclide
sau:
a = b.q +r
1
th (a; b) = (b; r
1
).
b = r
1
.q
1
+r
2
th (b; r
1
) = (r
1
; r
2
).

r
n2
= r
n1
.q
n1
+rn th (r
n2
; r
n1
) = (r
n1
; r
n
).
r
n1
= r
n
.q
n
th (r
n1
; r
n
) = r
n
.
(a; b) = r
n
.
(a; b) l s d cui cng khc 0 trong thut ton Euclide.
1.1.4 Bi tp v d
V d 1.1. Tm (2k 1; 9k + 4), k N

.
Li gii. Ta t d = (2k 1; 9k + 4). Theo tnh cht v c s chung
ta c d[2k 1 v d[9k +4. Tip tc p dng tnh cht v chia ht ta li
c d[9(2k 1) v d[2(9k + 4). Suy ra d[2(9k + 4) 9(2k 1) hay d[17.
Vy (2k 1; 9k + 4) = 1.
V d 1.2. Tm (123456789; 987654321).
Li gii. t b = 123456789; a = 987654321. Ta nhn thy a v b u
chia ht cho 9.
Ta li c :
a +b = 1111111110
=
10
10
10
9
.
9a + 9b = 10
10
10
(1.1)
Mt khc :
10b +a = 9999999999
= 10
10
1.
(1.2)
Chuyn S hc Din n Ton hc
4 1.2. Bi s, bi s chung, bi s chung nh nht
Tr (1.2) v (1.1) v theo v ta c b8a = 9. Do nu t d = (a; b)
th 9
.
.
.d.
M a v b u chia ht cho 9, suy ra d = 9.
Da vo thut ton Euclide, ta c li gii khc cho V d 1.2 nh sau :
Li gii. 987654321 = 123456789.8+9 th (987654321; 123456789) =
(123456789; 9).
123456789 = 9.1371421.
(123456789; 987654321) = 9.
V d 1.3. Chng minh rng dy s A
n
=
1
2
n(n + 1), n N

cha
nhng dy s v hn nhng s i mt nguyn t cng nhau.
Li gii. Gi s trong dy ang xt c k s i mt nguyn t cng
nhau l t
1
= 1; t
2
= 3; . . . ; t
k
= m(m N

). t a = t
1
t
2
. . . t
k
. Xt s
hng t
2a+1
trong dy A
n
:
t
2a+1
=
1
2
(2a + 1)(2a + 2)
= (a + 1)(2a + 1)
t
k
Mt khc ta c (a + 1; a) = 1 v (2a + 1; a) = 1 nn (t
2a+1
; a) = 1.
Do t
2a+1
nguyn t cng nhau vi tt c k s t
1
; t
2
; . . . t
k
. Suy ra
dy s A
n
cha v hn nhng s i mt nguyn t cng nhau.
1.2 Bi s, bi s chung, bi s chung nh nht
Tng t nh cu trc trnh by phn trc, trong phn ny
chng ti cng s a ra nhng nh ngha, tnh cht c bn ca bi
s, bi s chung, bi s chung nh nht v mt s bi tp v d minh
ha.
Din n Ton hc Chuyn S hc
1.2. Bi s, bi s chung, bi s chung nh nht 5
1.2.1 nh ngha
nh ngha 1.4 S t nhin m c gi l mt bi s ca a ,= 0 khi
v ch khi m chia ht cho a hay a l mt c s ca m.
Nhn xt. Tp hp cc bi s ca a ,= 0 l: B(a) = 0; a; 2a; . . . ; ka, k
Z.
nh ngha 1.5 S t nhin m c gi l mt bi s ca a ,= 0 khi
v ch khi m chia ht cho a hay a l mt c s ca m
nh ngha 1.6 Nu 2 tp B(a) v B(b) c phn t chung th cc phn
t chung gi l bi s chung ca a v b. Ta k hiu bi s chung
ca a v b: BSC(a; b).
nh ngha 1.7 S m ,= 0 c gi l bi chung nh nht ca a v
b khi m l phn t dng nh nht trong tp BSC(a; b). K hiu :
BCNN(a; b), [a; b] hay lcm(a; b).
1.2.2 Tnh cht
Mt s tnh cht ca bi chung ln nht:
Nu [a; b] = M th
_
M
a
;
M
b
_
= 1.
[a; b; c] = [[a; b]; c].
[a; b].(a; b) = a.b.
1.2.3 Bi tp v d
V d 1.4. Tm [n; n + 1; n + 2].
Li gii. t A = [n; n + 1] v B = [A; n + 2]. p dng tnh cht
[a; b; c] = [[a; b]; c], ta c: B = [n; n + 1; n + 2].
D thy (n; n + 1) = 1, suy ra [n; n + 1] = n(n + 1).
Chuyn S hc Din n Ton hc
6 1.3. Bi tp ngh
Li p dng tnh cht [a; b] =
a.b
(a; b)
th th
[n; n + 1; n + 2] =
n(n + 1)(n + 2)
(n(n + 1); n + 2)
.
Gi d = (n(n + 1); n + 2). Do (n + 1; n + 2) = 1 nn
d = (n; n + 2)
= (n; 2).
Xt hai trng hp:
Nu n chn th d = 2, suy ra [n; n + 1; n + 2] =
n(n + 1)(n + 2)
2
.
Nu n l th d = 1, suy ra [n; n + 1; n + 2] = n(n + 1)(n + 2) .
V d 1.5. Chng minh rng [1; 2; . . . 2n] = [n + 1; n + 2; . . . ; 2n].
Li gii. Ta thy c trong k s nguyn lin tip c mt v ch mt s
chia ht cho k. Do bt trong cc s 1; 2; . . . ; 2n u l c ca mt
s no trong cc s n + 1; n + 2; . . . ; 2n. Do [1; 2; . . . n; 2n] =
[n + 1; n + 2; . . . ; 2n].
1.3 Bi tp ngh
Thay cho li kt, chng ti xin gi n bn c mt s bi tp ngh
luyn tp nhm gip cc bn quen hn vi cc khi nim v cc
tnh cht trnh by trong chuyn .
Bi 1. a. Cho A = 5a + 3b; B = 13a + 8b(a; b N

) chng minh
(A; B) = (a; b).
b. Tng qut A = ma+nb; B = pa+qb tha mn [mqnp[ =
1 vi a, b, m, n, p, q N

. Chng minh (A; B) = (a; b).


Bi 2. Tm (6k + 5; 8k + 3)(k N).
Din n Ton hc Chuyn S hc
1.3. Bi tp ngh 7
Bi 3. T cc ch s 1; 2; 3; 4; 5; 6 thnh lp tt c s c su ch s
(mi s ch vit mt ln). Tm UCLN ca tt c cc s .
Bi 4. Cho A = 2n + 1; B =
n(n + 1)
2
(n N

). Tm (A; B).
Bi 5. a. Chng minh rng trong 5 s t nguyn lin tip bao gi
cng chn c mt s nguyn t cng nhau vi cc s
cn li.
b. Chng minh rng trong 16 s nguyn lin tip bao gi cng
chn c mt s nguyn t cng nhau vi cc s cn li.
Bi 6. Cho 1 m n(m; n N).
a. Chng minh rng (2
2
n
1; 2
2
n
+ 1) = 1.
b. Tm (2
m
1; 2
n
1).
Bi 7. Cho m, n N vi (m, n) = 1. Tm (m
2
+n
2
; m+n).
Bi 8. Cho A = 2
n
+3; B = 2
n+1
+3
n+1
(n N

); C = 2
n+2
+3
n+2
(n
N

). Tm (A; B) v (A; C).


Bi 9. Cho su s nguyn dng a; b; a

; b

; d; d

sao cho (a; b) = d; (a

; b

) =
d

. Chng minh rng (aa

; bb

; ab

; a

b) = dd

.
Bi 10. Chng minh rng dy s B
n
=
1
6
n(n + 1)(n + 2)(n N

) cha
v hn nhng s nguyn t cng nhau.
Bi 11. Chng minh rng dy s 2
n
3 vi mi n N v n 2 cha
dy s v hn nhng s nguyn t cng nhau.
Bi 12. Chng minh dy Mersen M
n
= 2
n
1(n N

) cha dy s v
hn nhng s nguyn t cng nhau.
Bi 13. Chng minh rng dy Fermat F
n
= 2
2
n
+ 1(n N) l dy s
nguyn t cng nhau.
Bi 14. Cho n N; n > 1 v 2
n
2 chia ht cho n. Tm (2
2
n
; 2
n
1).
Chuyn S hc Din n Ton hc
8 1.3. Bi tp ngh
Bi 15. Chng minh rng vi mi n N, phn s
21n + 1
14n + 3
ti gin.
Bi 16. Cho ba s t nhin a; b; c i mt nguyn t cng nhau. Chng
minh rng (ab +bc +ca; abc) = 1.
Bi 17. Cho a; b N

. Chng minh rng tn ti v s n N sao cho


(a +n; b +n) = 1.
Bi 18. Gi s m; n N(m n) tha mn (199k1; m) = (19931; n).
Chng minh rng tn ti t(t N) sao cho m = 1993
t
.n.
Bi 19. Chng minh rng nu a; m N; a > 1 th
_
a
m
1
a 1
; a 1
_
=
(m; a 1).
Bi 20. Tm s nguyn dng n nh nht cc phn s sau ti gin:
a.
1
n
1996
+ 1995n + 2
,
b.
2
n
1996
+ 1995n + 3
,
c.
1994
n
1996
+ 1995n + 1995
,
d.
1995
n
1996
+ 1995n + 1996
.
Bi 21. Cho 20 s t nhin khc 0 l a
1
; a
2
; . . . a
n
c tng bng S
v UCLN bng d. Chng minh rng UCLN ca S a
1
; S
a
2
; . . . ; S a
n
bng tch ca d vi mt c no ca n 1.
Din n Ton hc Chuyn S hc
Chng
2
S Nguyn T
2.1 Mt s kin thc c bn v s
nguyn t 9
2.2 Mt s bi ton c bn v s nguyn
t 13
2.3 Bi tp 19
2.4 Ph lc: Bn nn bit 24
Nguyn Trung Hiu (nguyentrunghieua)
Phm Quang Ton (Phm Quang Ton)
2.1 Mt s kin thc c bn v s nguyn t
2.1.1 nh ngha, nh l c bn
nh ngha 2.1 S nguyn t l nhng s t nhin ln hn 1, ch c 2
c s l 1 v chnh n.
nh ngha 2.2 Hp s l s t nhin ln hn 1 v c nhiu hn 2
c.
Nhn xt. Cc s 0 v 1 khng phi l s nguyn t cng khng phi
l hp s. Bt k s t nhin ln hn 1 no cng c t nht mt c
s nguyn t.
nh l 2.1 Dy s nguyn t l dy s v hn.
9
10 2.1. Mt s kin thc c bn v s nguyn t
Chng minh. Gi s ch c hu hn s nguyn t l p
1
; p
2
; p
3
; ...; p
n
;
trong p
n
l s ln nht trong cc nguyn t.
Xt s N = p
1
p
2
...p
n
+ 1 th N chia cho mi s nguyn t p
i
(i = 1, n)
u d 1 (*)
Mt khc N l mt hp s (v n ln hn s nguyn t ln nht l p
n
)
do N phi c mt c nguyn t no , tc l N chia ht cho mt
trong cc s p
i
(**).
Ta thy (**) mu thun (*). Vy khng th c hu hn s nguyn t.
nh l 2.2 Mi s t nhin ln hn 1 u phn tch c ra tha
s nguyn t mt cch duy nht (khng k th t cc tha s).
Chng minh. * Mi s t nhin ln hn 1 u phn tch c ra tha
s nguyn t:
Tht vy: gi s iu khng nh trn l ng vi mi s m tho mn:
1 < m < n ta chng minh iu ng n n.
Nu n l nguyn t, ta c iu phi chng minh.
Nu n l hp s, theo nh ngha hp s, ta c: n = a.b (vi a, b < n)
Theo gi thit quy np: a v b l tch cc tha s nh hn n nn n l
tch cu cc tha s nguyn t.
* S phn tch l duy nht:
Gi s mi s m < n u phn tch c ra tha s nguyn t mt
cch duy nht, ta chng minh iu ng n n:
Nu n l s nguyn t th ta c iu phi chng minh. Nu n l hp
s: Gi s c 2 cch phn tch n ra tha s nguyn t khc nhau:
n = p.q.r....
n = p

.q

.r

....
Trong p, q, r..... v p

, q

, r

.... l cc s nguyn t v khng c s


nguyn t no cng c mt trong c hai phn tch (v nu c s
tho mn iu kin nh trn, ta c th chia n cho s lc thng
s nh hn n, thng ny c hai cch phn tch ra tha s nguyn t
khc nhau, tri vi gi thit ca quy np).
Khng mt tnh tng qut, ta c th gi thit p v p

ln lt l cc s
nguyn t nh nht trong phn tch th nht v th hai.
V n l hp s nn n > p
2
v n > p
2
. Do p ,= p n > p.p

Din n Ton hc Chuyn S hc


2.1. Mt s kin thc c bn v s nguyn t 11
Xt m = n pp

< n c phn tch ra tha s nguyn t mt cch


duy nht ta thy:
p[n p[n pp

hay p[m
Khi phn tch ra tha s nguyn t ta c: m = npp

= p

p.P.Q... vi
P, Q P ( P l tp cc s nguyn t).
pp

[n pp

[p.q.r... p[q.r... p l c nguyn t ca q.r...


M p khng trng vi mt tha s no trong q, r... (iu ny tri vi
ga thit quy np l mi s nh hn n u phn tch c ra tha s
nguyn t mt cch duy nht).
Vy, iu gi s khng ng. nh l c chng minh.
2.1.2 Cch nhn bit mt s nguyn t
Cch 1
Chia s ln lt cho cc nguyn t t nh n ln: 2; 3; 5; 7...
Nu c mt php chia ht th s khng nguyn t.
Nu thc hin php chia cho n lc thng s nh hn s chia m cc
php chia vn c s d th s l nguyn t.
Cch 2
Mt s c hai c s ln hn 1 th s khng phi l s nguyn t.
Cho hc sinh lp 6 hc cch nhn bit 1 s nguyn t bng phng
php th nht (nu trn), l da vo nh l c bn:
c s nguyn t nh nht ca mt hp s A l mt s khng vt
qu

A.
Vi quy tc trn trong mt khon thi gian ngn, vi cc du hiu chia
ht th ta nhanh chng tr li c mt s c hai ch s no l
Chuyn S hc Din n Ton hc
12 2.1. Mt s kin thc c bn v s nguyn t
nguyn t hay khng.
H qu 2.1 Nu c s A > 1 khng c mt c s nguyn t no t
2 n

A th A l mt nguyn t.
2.1.3 S cc c s v tng cc c s ca 1 s
Gi s: A = p
x
1
1
.p
x
2
2
......p
n
x
n
; trong : p
i
P; x
i
N; i = 1, n
Tnh cht 2.1 S cc c s ca A tnh bng cng thc:
T(A) = (x
1
+ 1)(x
2
+ 1).....(x
n
+ 1)
V d 2.1. 30 = 2.3.5 th T(A) = (1 + 1)(1 + 1)(1 + 1) = 8. Kim tra:
(30) = 1; 2; 3; 5; 6; 10; 15; 30 nn (30) c 8 phn t.
Tnh cht 2.2 Tng cc c mt s ca A tnh bng cng thc:
(A) =
n

i=1
p
x
i
+1
i
1
p
i
1
2.1.4 Hai s nguyn t cng nhau
nh ngha 2.3 Hai s t nhin c gi l nguyn t cng nhau khi
v ch khi chng c c chung ln nht (CLN) bng 1.
Tnh cht 2.3 Hai s t nhin lin tip lun nguyn t cng nhau.
Tnh cht 2.4 Hai s nguyn t khc nhau lun nguyn t cng nhau.
Tnh cht 2.5 Cc s a, b, c nguyn t cng nhau khi v ch khi (a, b, c)
= 1.
nh ngha 2.4 Nhiu s t nhin c gi l nguyn t snh i khi
chng i mt nguyn t cng nhau.
Din n Ton hc Chuyn S hc
2.2. Mt s bi ton c bn v s nguyn t 13
2.1.5 Mt s nh l c bit
nh l 2.3 (Dirichlet) Tn ti v s s nguyn t p c dng:
p = ax +b (x, a, b N, a, b l 2 s nguyn t cng nhau).
Vic chng minh nh l ny kh phc tp, tr mt s trng hp c
bit, chng hn c v s s nguyn t dng: 2x1; 3x1; 4x+3; 6x+
5; . . .
nh l 2.4 (Tchebycheff-Betrand) Trong khong t s t nhin
n n s t nhin 2n c t nht mt s nguyn t (n > 2).
nh l 2.5 (Vinogradow) Mi s l ln hn 3
3
l tng ca 3 s
nguyn t.
2.2 Mt s bi ton c bn v s nguyn t
2.2.1 C bao nhiu s nguyn t dng ax + b
V d 2.2. Chng minh rng: c v s s nguyn t c dng 3x 1.
Li gii. Mi s t nhin khng nh hn 2 c 1 trong 3 dng: 3x; 3x+1
hoc 3x 1
Nhng s c dng 3x (vi x > 1) l hp s
Xt 2 s c dng 3x + 1: l s 3m+ 1 v s 3n + 1.
Xt tch (3m + 1)(3n + 1) = 9mn + 3m + 3n + 1. Tch ny c
dng: 3x + 1
Ly mt s nguyn t p bt c dng 3x 1, ta lp tch ca p
vi tt c cc s nguyn t nh hn p ri tr i 1 ta c: M =
2.3.5.7....p 1 = 3(2.5.7....p) 1 th M c dng 3x 1.
C 2 kh nng xy ra:
1. Kh nng 1: M l s nguyn t, l s nguyn t c dng
3x 1 > p, bi ton c chng minh.
Chuyn S hc Din n Ton hc
14 2.2. Mt s bi ton c bn v s nguyn t
2. Kh nng 2: M l hp s: Ta chia M cho 2, 3, 5, ...., p u tn
ti mt s d khc 0 nn cc c nguyn t ca M u ln
hn p, trong cc c ny khng c s no c dng 3x+1 (
chng minh trn). Do t nht mt trong cc c nguyn
t ca M phi c dng 3x (hp s) hoc 3x + 1
V nu tt c c dng 3x+1 th M phi c dng 3x+1 ( chng
minh trn). Do , t nht mt trong cc c nguyn t ca M
phi c dng 3x 1, c ny lun ln hn p.
Vy: C v s s nguyn t dng 3x 1.
V d 2.3. Chng minh rng: C v s s nguyn t c dng 4x+3.
Li gii. Nhn xt. Cc s nguyn t l khng th c dng 4x hoc
4x + 2. Vy chng ch c th tn ti di 1 trong 2 dng 4x + 1 hoc
4x + 3.
Ta s chng minh c v s s nguyn t c dng 4x + 3.
Xt tch 2 s c dng 4x + 1 l: 4m+ 1 v 4n + 1.
Ta c: (4m+1)(4n+1) = 16mn+4m+4n+1 = 4(4mn+m+n)+1.
Vy tch ca 2 s c dng 4x + 1 l mt s cng c dng 4x + 1.
Ly mt s nguyn t p bt k c dng 4x+3, ta lp tch ca 4p
vi tt c cc s nguyn t nh hn p ri tr i 1 khi ta c:
N = 4(2.3.5.7.....p) 1. C 2 kh nng xy ra
1. N l s nguyn t N = 4(2.3.5.7....p) 1 c dng 4x 1.
Nhng s nguyn t c dng 4x 1 cng chnh l nhng s
c dng 4x + 3 v bi ton c chng minh.
2. N l hp s. Chia N cho 2, 3, 5, ...., p u c cc s d
khc 0. Suy ra cc c nguyn t ca N u ln hn p.
Cc c ny khng th c dng 4x hoc 4x +2 (v l hp s).
Cng khng th ton cc c c dng 4x + 1 v nh th N phi
c dng 4x + 1. Nh vy trong cc c nguyn t ca N c t
nht 1 c c dng 4x 1 m c ny hin nhin ln hn p.
Din n Ton hc Chuyn S hc
2.2. Mt s bi ton c bn v s nguyn t 15
Vy: C v s s nguyn t c dng 4x 1 (hay c dng 4x + 3).
Trn y l mt s bi ton chng minh n gin ca nh l Dirichlet:
C v s s nguyn t dng ax +b trong a, b, x N, (a, b) = 1.
2.2.2 Chng minh s nguyn t
V d 2.4. Chng minh rng: (p 1)! chia ht cho p nu p l hp s,
khng chia ht cho p nu p l s nguyn t.
Li gii. Xt trng hp p l hp s: Nu p l hp s th p l tch
ca cc tha s nguyn t nh hn p v s m cc lu tha ny
khng th ln hn s m ca chnh cc lu tha y cha trong
(p 1)!. Vy: (p 1)!
.
.
.p (pcm).
Xt trng hp p l s nguyn t: V p P p nguyn t cng
nhau vi mi tha s ca (p 1)! (pcm).
V d 2.5. Cho 2
m
1 l s nguyn t. Chng minh rng m cng l
s nguyn t.
Li gii. Gi s m l hp s m = p.q (p, q N; p, q > 1)
Khi : 2
m
1 = 2
pq
1 = (2
p
)
q
1 = (2
p
1)((2
p
)
q1
+(2
p
)
q2
+.....+1)
v p > 1 2
p
1 > 1 v (2
p
)
q1
+ (2
p
)
q2
+..... + 1 > 1
Dn n 2
m
1 l hp s :tri vi gi thit 2
m
1 l s nguyn t.
Vy m phi l s nguyn t (pcm)
V d 2.6. Chng minh rng: mi c nguyn t ca 1994! 1 u ln
hn 1994.
Li gii. Gi p l c s nguyn t ca 1994! 1
Gi s p 1994 1994.1993.....3.2.1
.
.
.p 1994!
.
.
.p.
M 1994! 1
.
.
.p 1
.
.
.p (v l)
Vy: p > 1994 (pcm).
V d 2.7. Chng minh rng: n >2 th gia n v n! c t nht 1 s
nguyn t (t suy ra c v s s nguyn t).
Chuyn S hc Din n Ton hc
16 2.2. Mt s bi ton c bn v s nguyn t
Li gii. V n > 2 nn k = n! 1 > 1, do k c t nht mt c s
nguyn t p. Tng t bi tp 3, ta chng minh c mi c nguyn
t p ca k u ln hn k.
Vy: p > n n < p < n! 1 < n! (pcm)
2.2.3 Tm s nguyn t tha mn iu kin cho trc
V d 2.8. Tm tt c cc gi tr ca s nguyn t p : p + 10 v
p + 14 cng l s nguyn t.
Li gii. Nu p = 3 th p + 10 = 3 + 10 = 13 v p + 14 = 3 + 14 = 17
u l cc s nguyn t nn p = 3 l gi tr cn tm.
Nu p > 3 p c dng 3k + 1 hoc dng 3k 1
Nu p = 3k + 1 th p + 14 = 3k + 15 = 3(k + 5)
.
.
.3
Nu p = 3k 1 th p + 10 = 3k + 9 = 3(k + 3)
.
.
.3
Vy nu p > 3 th hoc p +10 hoc p +14 l hp s : khng tha mn
bi. Vy p = 3.
V d 2.9. Tm k N trong 10 s t nhin lin tip:
k + 1; k + 2; k + 3; ....k + 10
c nhiu s nguyn t nht.
Li gii. Nu k = 0: t 1 n 10 c 4 s nguyn t: 2; 3; 5; 7.
Nu k = 1: t 2 n 11 c 5 s nguyn t: 2; 3; 5; 7; 11.
Nu k > 1: t 3 tr i khng c s chn no l s nguyn t. Trong 5
s l lin tip, t nht c 1 s l bi s ca 3 do , dy s c t hn 5
s nguyn t.
Vy vi k = 1, dy tng ng: k + 1; k + 2, .....k + 10 c cha nhiu s
nguyn t nht (5 s nguyn t).
V d 2.10. Tm tt c cc s nguyn t p : 2
p
+p
2
cng l s nguyn
t.
Li gii. Xt 3 trng hp:
Din n Ton hc Chuyn S hc
2.2. Mt s bi ton c bn v s nguyn t 17
p = 2 2
p
+p
2
= 2
2
+ 2
2
= 8 , P
p = 3 2
p
+p
2
= 2
3
+ 3
2
= 17 P
p > 3 p ,
.
.
.3. Ta c 2
p
+p
2
= (p
2
1) + (2
p
+ 1).
V p l 2
p
+ 1
.
.
.3 v p
2
1 = (p + 1)(p 1)
.
.
.3 2
p
+p
2
, P
Vy c duy nht 1 gi tr p = 3 tho mn.
V d 2.11. Tm tt c cc s nguyn t p sao cho: p[2
p
+ 1.
Li gii. V p P : p[2
p
+ 1 p > 2 (2; p) = 1
Theo nh l Fermat, ta c: p[2
p1
1. M
p[2
p
+ 1 p[2(2
p1
1) + 3 p[3 p = 3
Vy: p = 3.
2.2.4 Nhn bit s nguyn t
V d 2.12. Nu p l s nguyn t v 1 trong 2 s 8p + 1 v 8p 1 l
s nguyn t th s cn li l s nguyn t hay hp s?
Li gii. Nu p = 2 8p + 1 = 17 P; 8p 1 = 15 , P
Nu p = 3 8p 1 = 23 P; 8p 1 = 25 , P
Nu p > 3, xt 3 s t nhin lin tip: 8p 1; 8p v 8p +1. Trong
3 s ny t c 1 s chia ht cho 3. Nn mt trong hai s 8p + 1
v 8p 1 chia ht cho 3.
Kt lun: Nu p P v 1 trong 2 s 8p + 1 v 8p 1 l s nguyn t
th s cn li phi l hp s.
V d 2.13. Nu p 5 v 2p + 1 l cc s nguyn t th 4p + 1 l
nguyn t hay hp s?
Li gii. Xt 3 s t nhin lin tip: 4p; 4p + 1; 4p + 2. Trong 3 s t
c mt s l bi ca 3.
M p 5; p P nn p c dng 3k + 1 hoc 3k + 2
Nu p = 3k + 1 th 2p + 1 = 6k + 3
.
.
.3: (tri vi gi thit)
Chuyn S hc Din n Ton hc
18 2.2. Mt s bi ton c bn v s nguyn t
Nu p = 3k+2. Khi 4p+1 = 4(3k+2)+1 = 12k+9
.
.
.3 4p+1
l hp s
V d 2.14. Trong dy s t nhin c th tm c 1997 s lin tip
nhau m khng c s nguyn t no hay khng ?
Li gii. Chn dy s: (a
i
) : a
i
= 1998! + i + 1 (i = 1, 1997) a
i
.
.
.i +
1 i = 1, 1997
Nh vy: Dy s a
1
; a
2
; a
3
; .....a
1997
gm c 1997 s t nhin lin tip
khng c s no l s nguyn t.
V d 2.15 (Tng qut bi tp 2.14). Chng minh rng c th tm
c 1 dy s gm n s t nhin lin tip (n > 1) khng c s no
l s nguyn t ?
Li gii. Ta chn dy s sau: (a
i
) : a
i
= (n+1)! +i +1 a
i
.
.
.i +1 i =
1, n.
Bn c hy t chng minh dy (a
i
) trn s gm c n s t nhin
lin tip trong khng c s no l s nguyn t c.
2.2.5 Cc dng khc
V d 2.16. Tm 3 s nguyn t sao cho tch ca chng gp 5 ln tng
ca chng.
Li gii. Gi 3 s nguyn t phi tm l a, b, c. Ta c: abc = 5(a +b +
c) abc
.
.
.5
V a, b, c c vai tr bnh ng nn khng mt tnh tng qut, gi s:
a
.
.
.5 a = 5
Khi : 5bc = 5(5 +b +c) 5 +b +c = bc (c 1)(b 1) = 6
Do vy:
_

_
_
b 1 = 1
c 1 = 6

_
b = 2
c = 7
chn
_
b 1 = 2
c 1 = 3

_
b = 3
c = 4
loi
Vy b s (a; b; c) cn tm l hon v ca (2; 5; 7).
V d 2.17. Tm p, q P sao cho p
2
= 8q + 1.
Din n Ton hc Chuyn S hc
2.3. Bi tp 19
Li gii. Ta c:
p
2
= 8q + 1 8q = p
2
1 = (p + 1)(p 1) (2.1)
Do p
2
= 8q + 1 : l p
2
: l p : l. t p = 2k + 1.
Thay vo (2.1) ta c:
8q = 2k(2k + 2) 2q = k(k + 1) (2.2)
Nu q = 2 4 = k(k + 1) khng tm c k N
Vy q > 2. V q P (2, q) = 1.
T (2.2) ta c:
a) k = 2 v q = k + 1 k = 2; q = 3. Thay kt qu trn vo (2.2)
ta c: p = 2.2 + 1 = 5
b) q = k v 2 = k + 1 q = 1 :loi.
Vy (q; p) = (5; 3).
2.3 Bi tp
2.3.1 Bi tp c hng dn
Bi 1. Ta bit rng c 25 s nguyn t nh hn 100. Tng ca 25 s
nguyn t nh hn 100 l s chn hay s l?
HD :Trong 25 s nguyn t nh hn 100 c cha mt s nguyn
t chn duy nht l 2, cn 24 s nguyn t cn li l s l. Do
tng ca 25 s nguyn t l s chn.
Bi 2. Tng ca 3 s nguyn t bng 1012. Tm s nguyn t nh nht
trong ba s nguyn t .
HD: V tng ca 3 s nguyn t bng 1012, nn trong 3 s
nguyn t tn ti t nht mt s nguyn t chn. M s
nguyn t chn duy nht l 2 v l s nguyn t nh nht. Vy
s nguyn t nh nht trong 3 s nguyn t l 2.
Chuyn S hc Din n Ton hc
20 2.3. Bi tp
Bi 3. Tng ca 2 s nguyn t c th bng 2003 hay khng? V sao?
HD: V tng ca 2 s nguyn t bng 2003, nn trong 2 s
nguyn t tn ti 1 s nguyn t chn. M s nguyn t
chn duy nht l 2. Do s nguyn t cn li l 2001. Do
2001 chia ht cho 3 v 2001 > 3. Suy ra 2001 khng phi l s
nguyn t.
Bi 4. Tm s nguyn t p, sao cho p + 2; p + 4 cng l cc s nguyn
t.
Bi 5. Cho p v p + 4 l cc s nguyn t (p > 3). Chng minh rng
p + 8 l hp s.
HD: V p l s nguyn t v p > 3, nn s nguyn t p c 1
trong 2 dng:
Nu p = 3k +2 th p +4 = 3k +6 = 3(k +2) p +4
.
.
.3 v
p + 4 > 3. Do p + 4 l hp s: tri bi.
Nu p = 3k +1 th p +8 = 3k +9 = 3(k +3) p +8
.
.
.3 v
p + 8 > 3. Do p + 8 l hp s.
Bi 6. Chng minh rng mi s nguyn t ln hn 2 u c dng 4n+1
hoc 4n 1.
Bi 7. Tm s nguyn t, bit rng s bng tng ca hai s nguyn
t v bng hiu ca hai s nguyn t.
HD: Gi s a, b, c, d, e l cc s nguyn t v d > e. Theo
bi:
a = b +c = d e ()
T (*) a > 2 nn a l s nguyn t l b +c; d e l s l.
Do b, d l cc s nguyn t b, d l s l c, e l s chn.
c = e = 2 (do c, el s nguyn t) a = b + 2 = d 2
d = b + 4.
Vy ta cn tm s nguyn t b sao cho b + 2 v b + 4 cng l
cc s nguyn t.
Din n Ton hc Chuyn S hc
2.3. Bi tp 21
Bi 8. Tm tt c cc s nguyn t x, y sao cho: x
2
6y
2
= 1.
Bi 9. Cho p v p + 2 l cc s nguyn t (p > 3). Chng minh rng
p + 1
.
.
.6.
2.3.2 Bi tp khng c hng dn
Bi 1. Tm s nguyn t p sao cho cc s sau cng l s nguyn t:
a) p + 2 v p + 10.
b) p + 10 v p + 20.
c) p + 10 v p + 14.
d) p + 14 v p + 20.
e) p + 2 v p + 8.
f) p + 2 v p + 14.
g) p + 4 v p + 10.
h) p + 8 v p + 10.
Bi 2. Tm s nguyn t p sao cho cc s sau cng l s nguyn t:
a) p + 2, p + 8, p + 12, p + 14
b) p + 2, p + 6, p + 8, p + 14
c) p + 6, p + 8, p + 12, p + 14
d) p + 2, p + 6, p + 8, p + 12, p + 14
e) p + 6, p + 12, p + 18, p + 24
f) p + 18, p + 24, p + 26, p + 32
g) p + 4, p + 6, p + 10, p + 12, p + 16
Bi 3. Cho trc s nguyn t p > 3 tha
a) p + 4 P. Chng minh rng: p + 8 l hp s.
b) 2p + 1 P. Chng minh rng: 4p + 1 l hp s.
c) 10p + 1 P. Chng minh rng: 5p + 1 l hp s.
Chuyn S hc Din n Ton hc
22 2.3. Bi tp
d) p + 8 P. Chng minh rng: p + 4 l hp s.
e) 4p + 1 P. Chng minh rng: 2p + 1 l hp s.
f) 5p + 1 P. Chng minh rng: 10p + 1 l hp s.
g) 8p + 1 P. Chng minh rng: 8p 1 l hp s.
h) 8p 1 P. Chng minh rng: 8p + 1 l hp s.
i) 8p
2
1 P. Chng minh rng: 8p
2
+ 1 l hp s.
j) 8p
2
+ 1 P. Chng minh rng: 8p
2
1 l hp s.
Bi 4. Chng minh rng:
a) Nu p v q l hai s nguyn t ln hn 3 th p
2
q
2
.
.
.24.
b) Nu a, a +k, a +2k(a, k N

) l cc s nguyn t ln hn
3 th k
.
.
.6.
Bi 5. a) Mt s nguyn t chia cho 42 c s d r l hp s. Tm s
d r.
b) Mt s nguyn t chia cho 30 c s d r. Tm s d r bit
rng r khng l s nguyn t.
Bi 6. Tm s nguyn t c ba ch s, bit rng nu vit s theo
th t ngc li th ta c mt s l lp phng ca mt s
t nhin.
Bi 7. Tm s t nhin c 4 ch s, ch s hng nghn bng ch s
hng n v, ch s hng trm bng ch s hng chc v s
vit c di dng tch ca 3 s nguyn t lin tip.
Bi 8. Tm 3 s nguyn t l cc s l lin tip.
Bi 9. Tm 3 s nguyn t lin tip p, q, r sao cho p
2
+q
2
+r
2
P.
Bi 10. Tm tt c cc b ba s nguyn t a, b, c sao cho abc < ab +
bc +ca.
Bi 11. Tm 3 s nguyn t p, q, r sao cho p
q
+q
p
= r.
Din n Ton hc Chuyn S hc
2.3. Bi tp 23
Bi 12. Tm cc s nguyn t x, y, z tho mn x
y
+ 1 = z.
Bi 13. Tm s nguyn t abcd tha ab, ac l cc s nguyn t v b
2
=
cd +b c.
Bi 14. Cho cc s p = b
c
+ a, q = a
b
+ c, r = c
a
+ b(a, b, c N

) l
cc s nguyn t. Chng minh rng 3 s p, q, r c t nht hai s
bng nhau.
Bi 15. Tm tt c cc s nguyn t x, y sao cho:
a) x
2
12y
2
= 1
b) 3x
2
+ 1 = 19y
2
c) 5x
2
11y
2
= 1
d) 7x
2
3y
2
= 1
e) 13x
2
y
2
= 3
f) x
2
= 8y + 1
Bi 16. Chng minh rng iu kin cn v p v 8p
2
+1 l cc s
nguyn t l p = 3.
Bi 17. Chng minh rng: Nu a
2
b
2
l mt s nguyn t th a
2
b
2
=
a +b.
Bi 18. Chng minh rng mi s nguyn t ln hn 3 u c dng 6n+1
hoc 6n 1.
Bi 19. Chng minh rng tng bnh phng ca 3 s nguyn t ln hn
3 khng th l mt s nguyn t.
Bi 20. Cho s t nhin n 2. Gi p
1
, p
2
, ..., p
n
l nhng s nguyn t
sao cho p
n
n+1. t A = p
1
.p
2
...p
n
. Chng minh rng trong
dy s cc s t nhin lin tip: A + 2, A + 3, ..., A + (n + 1),
khng cha mt s nguyn t no.
Bi 21. Chng minh rng: Nu p l s nguyn t th 2.3.4...(p 3)(p
2) 1
.
.
.p.
Chuyn S hc Din n Ton hc
24 2.4. Ph lc: Bn nn bit
Bi 22. Chng minh rng: Nu p l s nguyn t th 2.3.4...(p 2)(p
1) + 1
.
.
.p.
2.4 Ph lc: Bn nn bit
Mi s nguyn t c 93 ch s lp thnh cp s cng
Sau y l mt s nguyn t gm 93 ch s:
100996972469714247637786655587969840329509324689190041
803603417758904341703348882159067229719
K lc ny do 70 nh ton hc lp c nm 1998 tht kh m nh
bi c. H mt nhiu thng tnh ton mi tm c mi s nguyn
t to thnh mt cp s cng.
T mc tr chi trong 1 tp ch khoa hc, hai nh nghin cu trng
i hc Lyonl (Php) o su tng: Tm 6 s nguyn t sao cho
hiu 2 s lin tip lun lun nh nhau. iu l d i vi cc chuyn
gia nhng h mun i xa hn. Cng khng c vn g kh khn i
vi mt dy 7 s. H cn s h tr mt cht t c 8 s, mt s
h tr hn na t ti 9 s. Cui cng thng 3 nm 1998 c 70 nh
ton hc t khp trn th gii cng vi 200 my in ton hot ng
lin tc tm ra 10 s, mi s c 93 ch s, m hiu s ca 2 s lin
tip lun lun l 210. T s nguyn t trn ch cn thm vo 210 l
c s nguyn t th 2....
K lc c l dng : Theo c tnh ca cc nh khoa hc mun tm
c 1 dy 11 s nguyn t th phi mt hn 10 t nm.
Sinh ba rt t, phi chng sinh i li rt nhiu
Ta bit rng cc s nguyn t c th xa nhau tu iu ny th hin
bi tp:
Din n Ton hc Chuyn S hc
2.4. Ph lc: Bn nn bit 25
Bi ton 2.1. Cho trc s nguyn dng n tu . Chng minh rng
tn ti n s t nhin lin tip m mi s trong chng u l hp s.
Vy nhng, cc s nguyn t cng c th rt gn nhau. Cp s (2, 3)
l cp s t nhin lin tip duy nht m c hai bn u l s nguyn
t. Cp s (p, q)c gi l cp s sinh i, nu c 2 u l s nguyn
t v q = p + 2. B 3 s (p, q, r) gi l b s nguyn t sinh ba nu
c 3 s p,q,r u l cc s nguyn t v q = p + 2; r = q + 2.
Bi ton 2.2. Tm tt c cc b s nguyn t sinh ba?
y l mt bi ton d, dng phng php chng minh duy nht ta
tm ra b (3, 5, 7) l b ba s nguyn t sinh ba duy nht, cc b 3 s
l ln hn 3 lun c 1 s l hp s v n chia ht cho 3.
T bi ton 2.2 th bi ton sau tr thnh mt gi thuyt ln ang ch
cu tr li.
D on 2.1 Tn ti v hn cp s sinh i.
S hon ho (hon ton) ca nhng ngi Hy Lp c i
Ngi Hy Lp c i c quan nim thn b v cc s. H rt th v
pht hin ra cc s hon ho, ngha l cc s t nhin m tng cc c
s t nhin thc s ca n (cc c s nh hn s ) bng chnh n.
Chng hn:
6 = 1 + 2 + 3 28 = 1 + 2 + 4 + 7 + 14
Ngi Hy Lp c i bit tm tt c cc s hon ho chn ngha l
h lm c bi ton sau y:
Bi ton 2.3. Mt s t nhin chn n ,= 0 l s hon ho nu v ch
nu: n = 2
m+1
(2
m
1). Trong m l s t nhin khc 0 sao cho
2
m
1 l s nguyn t.
T ta c gi thuyt
Chuyn S hc Din n Ton hc
26 2.4. Ph lc: Bn nn bit
D on 2.2 Khng tn ti s hon ho l.
bi ton 2.3 trn, s nguyn t dng 2
m
1 gi l s nguyn t
Merseme. Cc s nguyn t Merseme c vai tr rt quan trng. Cho
n nay ngi ta vn cha bit c hu hn hay v hn s nguyn t
Merseme.
D on 2.3 Tn ti v hn s nguyn t Merseme.
Nm 1985 s nguyn t ln nht m ngi ta bit l s 2
132049
1 gm
39751 ch s ghi trong h thp phn. Gn y 2 sinh vin M tm
ra mt s nguyn t ln hn na l s 2
216091
1 gm 65050 ch s.
Ta bit rng vi hc sinh lp 6 th xem s A c t hn 20 ch s
c l s nguyn t khng bng cch th xem A c chia ht cho s no
nh hn A hay khng, th tm ht cc s nguyn t vi chic my
siu in ton cn hng th k !!!
David SlowinSky son mt phn mm, lm vic trn my siu in
ton Gray-2 , sau 19 gi ng tm ra s nguyn t 2
756839
1. S ny
vit trong h thp phn s c 227832 ch s- vit ht s ny cn 110
trang vn bn bnh thng. Hoc nu vit hng ngang nhng s trn
phng ch .VnTime Size 14 th ta cn khong 570 m.
Li Kt
Thng qua ti ny, chng ta c th khng nh rng: Ton hc c
mt trong mi cng vic, mi lnh vc ca cuc sng quanh ta, n
khng th tch ri v lng qun c, nn chng ta phi hiu bit v
nm bt c n mt cch t gic v hiu qu.
Mc ch ca ti ny l trang b nhng kin thc c bn c o
su c nng cao v rn luyn t duy ton hc cho hc sinh, to ra nn
tng tin cy cc em c vn kin thc nht nh lm hnh trang cho
Din n Ton hc Chuyn S hc
2.4. Ph lc: Bn nn bit 27
nhng nm hc tip theo.
Vi iu kin c nhiu hn ch v thi gian, v nng lc trnh nn
trong khun kh ti ny phn chia dng ton, loi ton ch c tnh
tng i. ng thi cng mi ch a ra li gii ch cha c phng
php, thut lm r rng. Tuy c c gng nhiu nhng chnsg ti t
thy trong ti ny cn nhiu hn ch. Chng ti rt mong nhn
c nhng kin ng gp ca cc thy c gio cng bn c ton
hc tht s c ngha cao p nh cu ngn ng Php vit:
Ton hc l Vua ca cc khoa hc
S hc l N hong
Chuyn S hc Din n Ton hc
Chng
3
Bi ton chia ht
3.1 L thuyt c bn 29
3.2 Phng php gii cc bi ton chia
ht 31
Phm Quang Ton (Phm Quang Ton)
Chia ht l mt ti quan trng trong chng trnh S hc ca bc
THCS. i km theo l cc bi ton kh v hay. Bi vit ny xin
gii thiu vi bn c nhng phng php gii cc bi ton chia ht:
phng php xt s d, phng php quy np, phng php ng d,
v.v...
3.1 L thuyt c bn
3.1.1 nh ngha v chia ht
nh ngha 3.1 Cho hai s nguyn a v b trong b ,= 0, ta lun tm
c hai s nguyn q v r duy nht sao cho
a = bq +r
vi 0 r < b.
Trong , ta ni a l s b chia, b l s chia, q l thng, r l s d.
Nh vy, khi a chia cho b th c th a ra cc s d r 0; 1; 2; ; [b[.
c bit, vi r = 0 th a = bq, khi ta ni a chia ht cho b (hoc a l
bi ca b, hoc b l c ca a). Ta k hiu b [ a. Cn khi a khng chia
29
30 3.1. L thuyt c bn
ht cho b, ta k hiu b a.
Sau y l mt s tnh cht thng dng, chng minh c suy ra trc
tip t nh ngha.
3.1.2 Tnh cht
Sau y xin gii thiu mt s tnh cht v chia ht, vic chng minh
kh l d dng nn s dnh cho bn c. Ta c vi a, b, c, d l cc s
nguyn th:
Tnh cht 3.1 Nu a ,= 0 th a [ a, 0 [ a.
Tnh cht 3.2 Nu b [ a th b [ ac.
Tnh cht 3.3 Nu b [ a v c [ b th c [ a.
Tnh cht 3.4 Nu c [ a v c [ b th c [ (ax by) vi x, y nguyn.
Tnh cht 3.5 Nu b [ a v a [ b th a = b hoc a = b.
Tnh cht 3.6 Nu c [ a v d [ b th cd [ ab.
Tnh cht 3.7 Nu b [ a, c [ a th BCNN(b; c) [ a.
Tnh cht 3.8 Nu c [ ab v UCLN(b, c) = 1 th c [ a.
Tnh cht 3.9 Nu p [ ab, p l s nguyn t th p [ a hoc p [ b.
T tnh cht trn ta suy ra h qu
H qu 3.1 Nu p [ a
n
vi p l s nguyn t, n nguyn dng th
p
n
[ a
n
.
Din n Ton hc Chuyn S hc
3.2. Phng php gii cc bi ton chia ht 31
3.1.3 Mt s du hiu chia ht
Ta t N = a
n
a
n1
. . . a
1
a
0
Du hiu chia ht cho 2; 5; 4; 25; 8; 125
2 [ N 2 [ a
0
a
0
0; 2; 4; 6; 8
5 [ N 5 [ a
0
a
0
0; 5
4; 25 [ N 4; 25 [ a
1
a
0
8; 125 [ N 8; 125 [ a
2
a
1
a
0
Du hiu chia ht cho 3 v 9
3; 9 [ N 3; 9 [ (a
0
+a
1
+ +a
n1
+a
n
)
Mt s du hiu chia ht khc
11 [ N 11 [ [(a
0
+a
2
+ ) (a
1
+a
3
+ )]
101 [ N 101 [ [(a
1
a
0
+a
5
a
4
+ ) (a
3
a
2
+a
7
a
6
+ )]
7; 13 [ N 7; 37 [ [(a
2
a
1
a
0
+a
8
a
7
a
6
+ ) (a
5
a
4
a
3
+a
11
a
10
a
9
+ )]
37 [ N 37 [ (a
2
a
1
a
0
+a
5
a
4
a
3
+ +a
n
a
n1
a
n2
)
19 [ N 19 [
_
a
n
+ 2a
n1
+ 2
2
a
n2
+ + 2
n
a
0
_
3.2 Phng php gii cc bi ton chia ht
3.2.1 p dng nh l Fermat nh v cc tnh cht ca chia
ht
nh l Fermat nh
nh l 3.1 (nh l Fermat nh) Vi mi s nguyn a v s
nguyn t p th a
p
p (mod p).
Chng minh. 1. Nu p [ a th p [ (a
5
a).
2. Nu p a th 2a, 3a, 4a, , (p 1)a cng khng chia ht cho p.
Gi r
1
, r
2
, , r
p1
ln lt l s d khi chia a, 2a, 3a, , (p1)a
cho p. th chng s thuc tp 1; 2; 3; ; p 1 v i mt khc
nhau (v chng hn nu r
1
= r
3
th p [ (3a a) hay p [ 2a,
Chuyn S hc Din n Ton hc
32 3.2. Phng php gii cc bi ton chia ht
ch c th l p = 2, m p = 2 th bi ton khng ng). Do
r
1
r
2
r
p1
= 1 2 3 (p 1). Ta c
a r
1
(mod p)
2a r
2
(mod p)

(p 1)a r
p1
(mod p)
Nhn v theo v ta suy ra
123 (p1)a
p1
r
1
r
2
r
p1
(mod p) a
p1
1 (mod p)
V UCLN(a, p) = 1 nn a
p
a (mod p).
Nh vy vi mi s nguyn a v s nguyn t p th a
p
a (mod p).
Nhn xt. Ta c th chng minh nh l bng quy np. Ngoi ra, nh
l cn c pht biu di dng sau:
nh l 3.2 Vi mi s nguyn a, p l s nguyn t, UCLN(a, p) =
1 th a
p1
1 (mod p).
Phng php s dng tnh cht chia ht v p dng nh l
Fermat nh
C s: S dng cc tnh cht chia ht v nh l Fermat nh gii
ton.
V d 3.1. Cho a v b l hai s t nhin. Chng minh rng 5a
2
+15ab
b
2
chia ht cho 49 khi v ch khi 3a +b chia ht cho 7.
Li gii. ) Gi s 49 [ 5a
2
+ 15ab b
2
7 [ 5a
2
+ 15ab b
2
7 [
(14a
2
+ 21ab) (5a
2
+ 15ab b
2
) 7 [ (9a
2
+ 6ab + b
2
) 7 [
(3a +b)
2
7 [ 3a +b.
) Gi s 7 [ 3a +b. t 3a +b = 7c (c Z. Khi b = 7c 3a. Nh
vy
5a
2
+ 15ab b
2
= 5a
2
+ 15a(7c 3a) (7c 3a)
2
= 49(c
2
+ 3ac a
2
)
Din n Ton hc Chuyn S hc
3.2. Phng php gii cc bi ton chia ht 33
chia ht cho 49.
Vy 5a
2
+ 15ab b
2
chia ht cho 49 khi v ch khi 3a +b chia ht cho
7.
V d 3.2. Cho 11 [ (16a + 17b)(17a + 16b) vi a, b l hai s nguyn.
Chng minh rng 121 [ (16a + 17b)(17a + 16b).
Li gii. Ta c theo u bi, v 11 nguyn t nn t nht mt trong
hai s 16a + 17b v 17a + 16b chia ht cho 11. Ta li c (16a + 17b) +
(17a + 16b) = 33(a + b) chia ht cho 11. Do nu mt trong hai s
16a +17b v 17a +16b chia ht cho 11 th s cn li cng chia ht cho
11. Cho nn 121 [ (16a + 17b)(17a + 16b).
V d 3.3. Chng minh rng A = 1
30
+ 2
30
+ + 11
30
khng chia ht
cho 11.
Li gii. Vi mi a = 1, 2, , 10 th (a, 10) = 1. Do theo nh l
Fermat b th a
10
1 (mod 11) a
30
1 (mod 11) vi mi a =
1, 2, , 10 v 11
30
0 (mod 11). Nh vy
A 1 + 1 + + 1
. .
10 s 1
+0 (mod 11)
10 (mod 11) 11 A
V d 3.4. Cho p v q l hai s nguyn t phn bit. Chng minh rng
p
q1
+q
p1
1 chia ht cho pq.
Li gii. V q nguyn t nn theo nh l Fermat nh th
p
q1
1 (mod q)
Do
p
q1
+q
p1
1 (mod q)
V q v p c vai tr bnh ng nn ta cng d dng suy ra
q
p1
+p
q1
1 (mod p).
Cui cng v UCLN(q, p) = 1 nn p
q1
+ q
p1
1 (mod pq) hay
p
q1
+q
p1
1 chia ht cho pq.
Chuyn S hc Din n Ton hc
34 3.2. Phng php gii cc bi ton chia ht
Bi tp ngh
Bi 1. Chng minh rng 11a+2b chia ht cho 19 khi v ch khi 18a+5b
chia ht cho 19 vi a, b l cc s nguyn.
Bi 2. Chng minh rng 2a + 7 chia ht cho 7 khi v ch khi 3a
2
+
10ab 8b
2
.
Bi 3. Cho p l s nguyn t ln hn 5. Chng minh rng nu n l s
t nhin c p 1 ch s v cc ch s u bng 1 th n chia
ht cho p.
Bi 4. Gi s n N, n 2. Xt cc s t nhin a
n
= 11 1 c vit
bi n ch s 1. Chng minh rng nu a
n
l mt s nguyn t
th n l c ca a
n
1.
Bi 5. Gi s a v b l cc s nguyn dng sao cho 2a 1, 2b 1 v
a +b u l s nguyn t. Chng minh rng a
b
+b
a
v a
a
+b
b
u khng chia ht cho a +b.
Bi 6. Chng minh rng vi mi s nguyn t p th tn ti s nguyn
n sao cho 2
n
+ 3
n
+ 6
n
1 chia ht cho p.
3.2.2 Xt s d
C s: chng minh A(n) chia ht cho p, ta xt cc s n dng
n = kp +r vi r 0; 1; 2; ; p 1.
Chng hn, vi p = 5 th s nguyn n c th vit li thnh 5k; 5k +
1; 5k + 2; 5k + 3; 5k + 4. Ta th mi dng ny vo cc v tr ca n ri
l lun ra p s. Sau y l mt s v d
V d 3.5. Tm k N tn ti n N sao cho
4 [ n
2
k
vi k 0; 1; 2; 3.
Li gii. Gi s tn ti k N tn ti n N tha mn 4 [ n
2
k.
Ta xt cc Trng hp: (m N

)
Din n Ton hc Chuyn S hc
3.2. Phng php gii cc bi ton chia ht 35
1. Nu n = 4m th n
2
k = 16m
2
k chia ht cho 4 khi v ch khi
4 [ k nn k = 0.
2. Nu n = 4m1 th n
2
k = 16m
2
8m+ 1 k chia ht cho 4
khi v ch khi 4 [ 1 k nn k = 1.
3. Nu n = 4m2 th n
2
k = 16m
2
16m+4 k chia ht cho 4
khi v ch khi 4 [ k nn k = 0.
Vy k = 0 hoc k = 1.
V d 3.6. Chng minh rng vi mi n N th 6 [ n(2n+7)(7n+1).
Li gii. Ta thy mt trong hai s n v 7n + 1 l s chn n N. Do
2 [ n(2n+7)(7n+1). Ta s chng minh 3 [ n(2n+7)(7n+1). Tht
vy, xt
1. Vi n = 3k th 3 [ n(2n + 7)(7n + 1).
2. Vi n = 3k + 1 th 2n + 7 = 6k + 9 chia ht cho 3 nn 3 [
n(2n + 7)(7n + 1).
3. Vi n = 3k + 2 th 7n + 1 = 21k + 15 chia ht cho 3 nn 3 [
n(2n + 7)(7n + 1).
Do 3 [ n(2n+7)(7n+1) m (2, 3) = 1 nn 6 [ n(2n+7)(7n+1) n
N.
V d 3.7. (HSG 9, Tp H Ch Minh, vng 2, 1995) Cho x, y, z l cc
s nguyn tha mn
(x y)(y z)(z x) = x +y +z (3.1)
Chng minh rng 27 [ (x +y +z).
Li gii. Xt hai trng hp sau
Chuyn S hc Din n Ton hc
36 3.2. Phng php gii cc bi ton chia ht
1. Nu ba s x, y, z chia ht cho 3 c cc s d khc nhau th cc
hiu xy, yz, zx cng khng chia ht cho 3. M 3 [ (x+y+z)
nn t (3.1) suy ra v l .
2. Nu ba s x, y, z ch c hai s chia cho 3 c cng s d th trong ba
hiu xy, yz, zx c mt hiu chia ht cho 3. M 3 (x+y+z)
nn t (3.1) suy ra v l.
Vy x, y, z chia cho 3 c cng s d, khi x y, y z, z x u chia
ht cho 3. T (3.1) ta suy ra 27 [ (x +y +z), ta c pcm.
Bi tp ngh
Bi 1. i) Tm s t nhin n 7 [ (2
n
1).
ii) Chng minh rng 7 (2
n
+ 1) n N.
Bi 2. Chng minh rng vi mi s nguyn a th a(a
6
1) chia ht
cho 7.
Bi 3. Tm n 13 [ 3
2n
+ 3
n
+ 1.
Bi 4. Chng minh rng vi mi a, b N th ab(a
2
b
2
)(4a
2
b
2
) lun
lun chia ht cho 5.
Bi 5. Chng minh rng 24 [ (p 1)(p + 1) vi p l s nguyn t ln
hn 3.
Bi 6. Chng minh rng khng tn ti s nguyn a a
2
+1 chia ht
cho 12.
Bi 7. Chng minh rng vi mi s nguyn x, y, z nu 6 [ x+y +z th
6 [ x
3
+y
3
+z
3
.
Bi 8. Cho ab = 2011
2012
, vi a, b N. Hi tng a +b c chia ht cho
2012 hay khng ?
Bi 9. S 3
n
+2003 trong n l s nguyn dng c chia ht cho 184
khng ?
Din n Ton hc Chuyn S hc
3.2. Phng php gii cc bi ton chia ht 37
Bi 10. Cho cc s nguyn dng x, y, z tha mn x
2
+y
2
= z
2
. Chng
minh rng xyz chia ht cho 60.
Bi 11. Cho cc s nguyn dng x, y, z tha mn x
2
+y
2
= 2z
2
. Chng
minh rng x
2
y
2
chia ht cho 84.
Bi 12. Cho n > 3, (n N). Chng minh rng nu 2
n
= 10a+b, (0 <
b < 9) th 6 [ ab.
3.2.3 Phn tch
Phn tch thnh tch
C s: chng minh A(n) chia ht cho p, ta phn tch A(n) = D(n)p,
cn nu trong ta khng th a ra cch phn tch nh vy, ta c th
vit p = kq.
Nu (k, q) = 1 th ta chng minh A(n) cng chia ht cho k v q.
Nu (k, q) ,= 1 th ta vit A(n) = B(n)C(n) v chng minh B(n)
chia ht cho k, C(n) chia ht cho q.
V d 3.8. Cho n l mt s nguyn dng. Chng minh rng
2
n
[ (n + 1) (n + 2) (2n) .
Li gii. Ta c
(n + 1) (n + 2) (2n) =
(2n)!
n!
=
(1.3.5...(2n 1)) (2.4.6...2n)
n!
= 1.3.5...(2n 1).2
n
.
n!
n!
= 1.3.5...(2n 1).2
n
.
Do 2
n
[ (n + 1) (n + 2) (2n) .
Chuyn S hc Din n Ton hc
38 3.2. Phng php gii cc bi ton chia ht
V d 3.9. Chng minh rng vi mi s nguyn n th 6 [ n
3
n.
Li gii. Phn tch
n
3
n = n(n
2
1) = n(n 1)(n + 1)
Biu thc l tch ba s nguyn lin tip nn tn ti t nht mt trong
ba s mt s chia ht cho 2 v mt s chia ht cho 3. M (2, 3) = 1 nn
6 [ n
3
n.
V d 3.10. Chng minh rng n
6
n
4
n
2
+1 chia ht cho 128 vi n
l.
Li gii. Ta c
n
6
n
4
n
2
+ 1 = (n
2
1)
2
(n + 1) = (n 1)
2
(n + 1)
2
V n l nn t n = 2k, k N, suy ra
(n
2
1)
2
=
_
(2k + 1)
2
1

= (4k
2
+ 4k)
2
= [4k(k + 1)]
2
Vy 64 [ (n
2
1)
2
. V n l nn 2 [ n + 1, suy ra pcm.
V d 3.11. Cho ba s nguyn dng khc nhau x, y, z. Chng minh
rng (xy)
5
+(y z)
5
+(xz)
5
chia ht cho 5(xy)(y z)(xz).
Li gii. Ta c
(x y)
5
+ (y z)
5
+ (x z)
5
= (x z +z y)
5
+ (y z)
5
+ (z x)
5
= (x z)
5
+ 5(x z)
4
(z y) + 10(x z)
3
(z y)
2
+10(x z)
4
(z y) + 10(x z)
3
(z y)
2
+10(x z)
2
(z y)
3
+ 5(x z)(z y)
4
= 5(x z)(z y)

_
(x z)
3
+ 2(x z)
2
(z y) + 2(x z)(z y)
2
+ (z y)
3

.
Din n Ton hc Chuyn S hc
3.2. Phng php gii cc bi ton chia ht 39
Nhng ta cng c:
(x z)
3
+ 2(x z)
2
(z y) + 2(x z)(z y)
2
+ (z y)
3
= (x y +y z)
3
+ 2(x y +y z)
2
(z y)
+2(x y +y z)(z y)
2
+ (z y)
3
= (x y)
3
+ 2(x y)
2
(y z) + 3(x y)(y z)
2
+(y z)
3
+ 2(x y)
2
(z y)
+4(x y)(y z)(z y) + 2(y z)
2
(z y)
+2(x y)(z y)
2
+ 2(y z)(z y)
2
+ (z y)
3
= (x y)
3
+ 3(x y)
2
(y z) + 3(x y)(y z)
2
+2(x y)
2
(z y) + 4(x y)(y z)(z y) + 2(x y)(z y)
2
,
Biu thc cui cng c nhn t chung (x y). Ta suy ra iu phi
chng minh.
Bi tp ngh
Bi 1. Chng minh rng nu a, k l cc s nguyn, a l th 2
k+1
[
(a
2
k
1).
Bi 2. Chng minh rng n
5
n chia ht cho 30 vi mi n Z.
Bi 3. Chng minh rng 3n
4
14n
3
+ 21n
2
10n chia ht cho 24 vi
mi n Z.
Bi 4. Chng minh rng n
5
5n
3
+4n chia ht cho 120 vi mi n Z.
Bi 5. Chng minh rng n
3
3n
2
n + 3 chia ht cho 48 vi mi n
l, n Z.
Bi 6. Chng minh rng n
8
n
6
n
4
+n
2
chia ht cho 1152 vi mi
s nguyn n l.
Bi 7. Chng minh rng n
4
4n
3
4n
2
+16n chia ht cho 348 vi mi
n l s nguyn chn.
Bi 8. Chng minh rng n
4
14n
3
+ 71n
2
154n + 120 chia ht cho
24 vi mi s t nhin n.
Chuyn S hc Din n Ton hc
40 3.2. Phng php gii cc bi ton chia ht
Bi 9. Cho x, y, z l cc s nguyn khc 0. Chng minh rng nu
x
2
yz = a, y
2
zx = b, z
2
xy = c th tng (ax + by + cz)
chia ht cho tng (a +b +c).
Bi 10. Cho m, n l hai s chnh phng l lin tip. Chng minh rng
mn mn + 1 chia ht cho 192.
Bi 11. (HSG 9 TQ 1970) Chng minh rng n
12
n
8
n
4
+1 chia ht
cho 512 vi mi s t nhin n l.
Bi 12. (HSG 9 TQ 1975) Chng minh rng n
4
+6n
3
+11n
2
+6n chia
ht cho 24 vi mi s nguyn dng n.
Tch tng
C s: chng minh A(n) chia ht cho p, ta bin i A(n) thnh
tng nhiu hng t ri chng minh mi hng t u chia ht cho p.
Ta c th s dng mt s hng ng thc p dng vo chia ht, v d
nh:
Cho a, b l cc s thc v n l s nguyn dng. Khi ta
c
a
n
b
n
= (a b)(a
n1
+a
n2
b + +ab
n2
+b
n1
)
Ta s c h qu l:
H qu 3.2 Nu a b ,= 0 th a
n
b
n
chia ht cho a b.
H qu 3.3 Nu a +b ,= 0 v n l th a
n
+b
n
chia ht cho a +b.
H qu 3.4 Nu a +b ,= 0 v n chn th a
n
b
n
chia ht cho a +b
V d 3.12. Chng minh rng ax
2
+bx+c Z, x Z khi v ch khi
2a, a +b, c Z
Din n Ton hc Chuyn S hc
3.2. Phng php gii cc bi ton chia ht 41
Li gii. Phn tch
ax
2
+bx +c = ax
2
ax + (a +b)x +c
= 2a.
x(x 1)
2
+ (a +b)x +c Z, x Z.
V d 3.13. Chng minh rng 6 [ (a
3
+ 5a) a N.
Li gii. Phn tch a
3
+5a = (a
3
a)+6a. Hin nhin ng v 6 [ n
3
n
(chng minh v d Equation 4.27).
Nhn xt. T v d Equation 4.27 ta cng c th a ra cc bi ton
sau, chng minh cng bng cch vn dng phng php tch tng:
Bi ton 3.1. Cho m, n Z. Chng minh rng 6 [ m
2
n
2
(mn).
Bi ton 3.2. Cho a, b, c Z. Chng minh rng 6 [ (a
3
+ b
3
+ c
3
) khi
v ch khi 6 [ (a +b +c)
Bi ton 3.3. Cho a Z. Chng minh rng
a
3
+
a
2
2
+
a
3
6
Z
Bi ton 3.4. Vit s 2011
2012
thnh tng cc s nguyn dng. em
tng lp phng tt c cc s hng chia cho 3 th c d l bao
nhiu ?
V d 3.14. Cho m, n l cc s nguyn tha mn:
m
n
= 1
1
2
+
1
3

1
4
+
1
1334
+
1
1335
Chng minh rng 2003 [ m.
Chuyn S hc Din n Ton hc
42 3.2. Phng php gii cc bi ton chia ht
Li gii. rng 2003 l s nguyn t. Ta c
m
n
= 1
1
2
+
1
3

1
4
+
1
1334
+
1
1335
=
_
1 +
1
2
+
1
3
+ +
1
1335
_
2
_
1
2
+
1
4
+
1
6
+ +
1
1334
_
=
_
1 +
1
2
+
1
3
+ +
1
1335
_

_
1 +
1
2
+
1
3
+ +
1
667
_
=
1
668
+
1
669
+ +
1
1335
=
_
1
668
+
1
1335
_
+
_
1
669
+
1
1334
_
+ +
_
1
1001
+
1
1002
_
= 2003
_
1
668.1335
+
1
669.1334
+ +
1
1001.1002
_
= 2003.
p
q
y p l s nguyn cn q = 668 669 1335. V 2003 nguyn t nn
(q, 2003) = 1.
Do t () suy ra 2003pn = mq.
V p, n nguyn nn suy ra 2003[mq m (q, 2003) = 1 nn 2003[m.
V d 3.15. Chng minh rng vi mi s t nhin n th A = 2005
n
+
60
n
1897
n
168
n
chia ht cho 2004.
Li gii. Ta c 2004 = 12 167. V (12, 167) = 1 nn chng minh
A chia ht cho 2004 ta chng minh A chia ht cho 12 v 167.
p dng tnh cht a
n
b
n
chia ht cho a b vi mi n t nhin v
ab ,= 0 suy ra 2005
n
1897
n
chia ht cho 20051897 = 108 = 129,
hay 2005
n
1897
n
chia ht cho 12. Tng t th 168
n
60
n
chia ht
cho 12. Vy A chia ht cho 12.
Tip tc phn tch
A = (2005
n
168
n
) (1897
n
60
n
).
Lp lun tng t nh trn th 2005
n
168
n
v 1897
n
60
n
chia ht
cho 167, tc A chia ht cho 167. Vy ta c iu phi chng minh.
Din n Ton hc Chuyn S hc
3.2. Phng php gii cc bi ton chia ht 43
V d 3.16. ( thi tuyn sinh HKHTN-HQG H Ni, vng 1, nm
2007-2008) Cho a, b l hai s nguyn dng v a +1, b +2007 u chia
ht cho 6. Chng minh rng 4
a
+a +b chia ht cho 6.
Li gii. Phn tch
4
a
+a +b = (4
a
+ 2) + (a + 1) + (b + 2007) 2010
4
a
+ 2 = 4
a
1 + 3 = (4 1)(4
a1
+ 1) + 3
Nh vy 3 [ 4
a
+ 2. Do 4
a
+a +b l tng ca cc s nguyn dng
chia ht cho 6 nn 4
a
+a +b chia ht cho 6.
Bi tp ngh
Bi 1. a ra cc m rng t bi tp ngh ca phng php phn
tch thnh tch thnh cc bi ton vn dng phng php tch
tng (ging nh cch m rng ca v d 1.9).
Bi 2. (Hungary MO 1947) Chng minh rng 46
n
+ 296.13
n
chia ht
cho 1947 vi mi s t nhin n l.
Bi 3. Chng minh rng 20
n
+ 16
n
3
n
1 chia ht cho 323 vi mi
s t nhin n chn.
Bi 4. Chng minh rng 2903
n
803
n
464
n
+261
n
chia ht cho 1897
vi mi s t nhin n.
Bi 5. Chng minh rng vi mi s nguyn n > 1 ta c n
n
+ 5n
2

11n + 5 chia ht cho (n 1)


2
.
Bi 6. (HSG 9 Tp H Ni, vng 2, 1998) Chng minh rng 1997 [ m
vi m, n N tha mn
m
n
= 1
1
2
+
1
3

1
4
+ +
1
1329

1
1330
+
1
1331
.
Bi 7. Chng minh rng 3
2n+1
+ 2
n+2
chia ht cho 7 vi mi n N.
Chuyn S hc Din n Ton hc
44 3.2. Phng php gii cc bi ton chia ht
Bi 8. Chng minh rng 2003
2005
+ 2017
2015
chia ht cho 12.
Bi 9. Cho p l s t nhin l v cc s nguyn a, b, c, d, e tha mn
a + b + c + d + e v a
2
+ b
2
+ c
2
+ d
2
+ e
2
u chia ht cho p.
Chng minh rng s a
5
+b
5
+c
5
+d
5
+e
5
5abcde cng chia
ht cho p.
Bi 10. (Canada Training for IMO 1987)
K hiu:
1 3 5 (2n 1) = (2n 1)!!
2 4 6 (2n) = (2n)!!.
Chng minh rng (1985)!! + (1986)!! chia ht cho 1987.
Bi 11. Chng minh rng s 2222
5555
+ 5555
2222
chia ht cho 7.
Bi 12. Cho k l s nguyn dng sao cho s p = 3k + 1 l s nguyn
t v
1
1 2
+
1
3 4
+ +
1
(2k 1)2k
=
m
n
vi hai s nguyn dng nguyn t cng nhau m v n.Chng
minh m chia ht cho p.
(Tp ch Mathematics Reflections, ng bi T.Andreescu)
3.2.4 Xt ng d
nh ngha v mt s tnh cht
nh ngha 3.2 Cho a, b l cc s nguyn v n l s nguyn dng. Ta
ni, a ng d vi b theo modun n v k hiu a b (mod n) nu a v
b c cng s d khi chia cho n.
Nh vy a n (mod n) n [ (a b). V d: 2012 2 (mod 5).
Tnh cht (bn c t chng minh)
Cho a, b, c, d, n l cc s nguyn.
Tnh cht 3.10
a a (mod n),
a b (mod n) b a (mod n),
a b (mod n), b c (mod n) a c (mod n).

Din n Ton hc Chuyn S hc


3.2. Phng php gii cc bi ton chia ht 45
Tnh cht 3.11
_
a b (mod n)
c d (mod n)

_
a c b d (mod n)
ac bd (mod n)

Tnh cht 3.12 a b (mod n) a


k
b
k
(mod n), k 1.
Tnh cht 3.13 a b (mod n) ac bc (mod mc), c > 0
Tnh cht 3.14 (a +b)
n
b
n
(mod a), (a > 0).
Tnh cht 3.15 Nu d l c chung dng ca a, b v m th a b
(mod m) th
a
d

b
d
(mod
m
d
).
Tnh cht 3.16 a b (mod m), c l c chung ca a v b, (c, m) = 1
th
a
c

b
c
(mod m).
Phng php ng d thc gii cc bi ton chia ht
C s: S dng cc tnh cht v nh ngha trn gii cc bi ton
chia ht.
V d 3.17. Chng minh rng vi mi s t nhin n th 7 [ 8
n
+ 6.
Li gii. Ta c 8
n
1 (mod 7) = 8
n
+ 6 7 0 (mod 7).
V d 3.18. Chng minh rng 19 [ 7 5
2n
+ 12 6
n
. vi mi s nguyn
dng n.
Li gii. Ta c 5
2
= 25 6 (mod 19) = (5
2
)
n
6
n
(mod 19) =
7 5
2n
7 6
n
(mod 19) = 7 5
2n
+12 6
n
19 6
n
0 (mod 19).
V d 3.19. Vit lin tip cc s 111, 112, , 888 c s A =
111112 888. Chng minh rng 1998 [ A.
Chuyn S hc Din n Ton hc
46 3.2. Phng php gii cc bi ton chia ht
Li gii. Ta thy A chn nn 2 [ A. Mt khc
A = 111 1000
777
+ 112 1000
776
+ + 888.
Do 1000
k
1 (mod 999), k N nn
A 111 + 112 + + 888 0 (mod 999).
Suy ra 999 [ A, v (999, 2) = 1 nn 1998 [ A.
V d 3.20. Chng minh rng 7 [ 5555
2222
+ 2222
5555
.
Li gii. Ta c
2222 4 (mod 7) = 2222
5555
(4)
5555
(mod 7)
5555 4 (mod 7) = 5555
2222
4 (mod 7)
= 5555
2222
+ 2222
5555
4
5555
+ 4
2222
(mod 7)
Li c
4
5555
+ 4
2222
= 4
2222
_
4
3333
1
_
= 4
2222
_
64
1111
1
_
V 64 1 (mod 7) = 64
1111
1 0 (mod 7).
Do 7 [ 5555
2222
+ 2222
5555

Bi tp ngh
Bi 1. Mt s bi tp phng php phn tch c th gii bng phng
php ng d thc.
Bi 2. Chng minh rng 333
555
777
+ 777
555
333
chia ht cho 10.
Bi 3. Chng minh rng s 11
10
1967
1 chia ht cho 10
1968
.
Bi 4. Cho 9 [ a
3
+b
3
+c
3
, a, b, c Z. Chng minh rng 3 [ a b c.
Bi 5. Chng minh rng 222
333
+ 333
222
chia ht cho 13.
Din n Ton hc Chuyn S hc
3.2. Phng php gii cc bi ton chia ht 47
Bi 6. Chng minh rng 9
n
+ 1 khng chia ht cho 100, n N.
Bi 7. Chng minh rng vi mi s nguyn khng m n th 2
5n+3
+
5
n
3
n+1
chia ht cho 17.
Bi 8. Tm n N sao cho 2n
3
+ 3n = 19851986.
Bi 9. Vit lin tip 2000 s 1999 ta c s X = 19991999 1999.
Tm s d trong php chia X cho 10001.
Bi 10. Chng minh rng 100 [ 7
7
7
7
7
7
7
7
.
Bi 11. Cho b
2
4ac v b
2
+ 4ac l hai s chnh phng vi a, b, c N.
Chng minh rng 30 [ abc.
3.2.5 Quy np
C s : chng minh mnh ng vi mi s t nhin n p, ta
lm nh sau:
Kim tra mnh ng vi n = p.
Gi s mnh ng vi n = k. Ta i chng minh mnh cng
ng vi n = k + 1.
V d 3.21. Chng minh rng A = 4
n
+ 15 1 chia ht cho 9 vi mi
n N

.
Li gii. Vi n = 1 = A = 18 chia ht cho 9.
Gi s bi ton ng vi n = k. Khi 9 [ 4
k
+15
k
1, hay 4
k
+15
k
1 =
9q vi q N

. Suy ra 4
k
= 9q 15k + 1.
Ta i chng minh bi ton ng vi n = k+1, tc 9 [ 4
k+1
+15(k+1)1.
Tht vy:
4
k+1
+ 15(k + 1) 1 = 4 4
k
+ 15k + 14
= 4 (9q 15k + 1) + 15k + 14
= 36q 45k + 18
chia ht cho 9. Ta c pcm.
Chuyn S hc Din n Ton hc
48 3.2. Phng php gii cc bi ton chia ht
V d 3.22. (HSG 9 TQ 1978)Chng minh rng s c to bi 3
n
ch
s ging nhau th chia ht cho 3
n
vi 1 n, n N.
Li gii. Vi n = 1, bi ton hin nhin ng.
Gi s bi ton ng vi n = k, tc 3
k
[ aa a
. .
3
n
s a
.
Vi n = k + 1 ta c:
aa a
. .
3
k+1
= aa a
. .
3
k
aa a
. .
3
k
aa a
. .
3
k
= aa a
. .
3
k
1 00 0
. .
3
k
1
00 0
. .
3
k
1
1
chia ht cho 3
k+1
. Ta c pcm.
V d 3.23. Chng minh rng vi mi n N

, k l s t nhin l th
2
n+2
[ k
2
n
1
Li gii. Vi n = 1 th k
2
n
1 = k
2
1 = (k + 1)(k 1). Do k l,nn
t k = 2m+ 1 vi m N

, th khi (k + 1)(k 1) = 4k(k + 1) chia


ht cho 2
3
= 8.
Gi s bi ton ng vi n = p, tc 2
p+2
[ k
2
p
1 hay k
2
p
= q 2
p+2
+1
vi q N

.
Ta chng minh bi ton ng vi n = p + 1. Tht vy
A = k
2
p+1
1 = k
22
p
1 =
_
k
2
p
_
2
1
=
_
k
2
p
1
_ _
k
2
p
+ 1
_
= q 2
p+2

_
2 +q 2
p+2
_
= q 2
p+3

_
1 +q 2
p+1
_
chia ht cho 2
p+3
. Ta c pcm.
Din n Ton hc Chuyn S hc
3.2. Phng php gii cc bi ton chia ht 49
Bi tp ngh
Bi 1. Mt s bi ton cc phng php nu trn c th gii bng
phng php quy np.
Bi 2. Chng minh rng 255 [ 16
n
15n 1 vi n N.
Bi 3. Chng minh rng 64 [ 3
2n+3
+ 40n 27 vi n N.
Bi 4. Chng minh rng 16 [ 3
2n+2
+ 8n 9 vi n N.
Bi 5. Chng minh rng 676 [ 3
3n+3
16n 27 vi n N, n 1.
Bi 6. Chng minh rng 700 [ 29
2n
140n 1 vi n N.
Bi 7. Chng minh rng 270 [ 2002
n
138n 1 vi n N.
Bi 8. Chng minh rng 22 [ 3
2
4n+1
+ 2
3
4n+1
+ 5 vi n N.
Bi 9. Chng minh rng s 2
3
n
+ 1 chia ht cho 3
n
nhng khng chia
ht cho 3
n+1
vi n N.
Bi 10. Chng minh rng s 2001
2
n
1 chia ht cho 2
n+4
nhng khng
chia ht cho 2
n+5
vi n N.
Bi 11. Chng minh rng vi mi s t nhin n 2, tn ti mt s t
nhin m sao cho 3
n
[ (m
3
+ 17), nhng 3
n+1
(m
3
+ 17).
Bi 12. C tn ti hay khng mt s nguyn dng l bi ca 2007 v
c bn ch s tn cng l 2008.
Bi 13. Chng minh rng tn ti mt s c 2011 ch s gm ton ch
s 1 v 2 sao cho s chia ht cho 2
2011
.
Bi 14. Tm phn d khi chia 3
2
n
cho 2
n+3
, trong n l s nguyn
dng.
Bi 15. Cho n N, n 2. t A = 7
7
.
.
.
(ly tha n ln). Chng minh
rng A
n
+ 17 chia ht cho 20.
Chuyn S hc Din n Ton hc
50 3.2. Phng php gii cc bi ton chia ht
3.2.6 S dng nguyn l Dirichlet
Ni dung: Nht 5 con th vo 3 chung th tn ti chung cha t nht
2 con.
nh l 3.3 Nht m = nk + 1 con th vo k chung (k < n) th tn
ti chung cha t nht n + 1 con th.
Chng minh. Gi s khng c chung no cha t nht n+1 con th,
khi mi chung cha nhiu nht n con th, nn k chung cha nhiu
nht kn con th, mu thun vi s th l nk + 1.
nh l 3.4 (p dng vo s hc) Trong m = nk + 1 s c t
nht n + 1 s chia cho k c cng s d.
Tuy nguyn l c pht biu kh n gin nhng li c nhng ng
dng ht sc bt ng, th v. Bi vit ny ch xin nu mt s ng dng
ca nguyn l trong vic gii cc bi ton v chia ht.
V d 3.24. Chng minh rng lun tn ti s c dng
20112011 201100 0
chia ht cho 2012.
Li gii. Ly 2013 s c dng
2011; 20112011, , 20112011 2011
. .
2012 s 2011
.
Ly 2013 s ny chia cho 2012. Theo nguyn l Dirichlet th tn ti hai
s c cng s d khi chia cho 2012.
Gi s hai s l 20112011 2011
. .
m s 2011
v 20112011 2011
. .
n s 2011
(m > n >
0).
= 2012 [ 20112011 2011
. .
m s 2011
20112011 2011
. .
n s 2011
Din n Ton hc Chuyn S hc
3.2. Phng php gii cc bi ton chia ht 51
= 2012 [ 20112011 2011
. .
mn s 2011
00 00
. .
n s 2011
Vy tn ti s tha mn bi.
V d 3.25. Chng minh rng trong 101 s nguyn bt k c th tm
c hai s c 2 ch s tn cng ging nhau.
Li gii. Ly 101 s nguyn cho chia cho 100 th theo nguyn l
Dirichlet tn ti hai s c cng s d khi chia cho 100. Suy ra trong
101 s nguyn cho tn ti hai s c ch s tn cng ging nhau.
V d 3.26 (Tuyn sinh 10 chuyn HSPHN, 1993). Cho 5 s nguyn
phn bit ty a
1
, a
2
, a
3
, a
4
, a
5
. Chng minh rng tch
P = (a
1
a
2
)(a
1
a
3
)(a
1
a
4
)(a
1
a
5
)(a
2
a
3
)
(a
2
a
4
)(a
2
a
5
)(a
3
a
4
)(a
3
a
5
)(a
4
a
5
)
chia ht cho 288.
Li gii. Phn tch 288 = 2
5
3
2
.
1. Chng minh 9 [ P: Theo nguyn l Dirichlet th trong 4 s
a
1
, a
2
, a
3
c hai s c hiu chia ht cho 3. Khng mt tnh tng
qut, gi s: 3 [ a
1
a
2
. Xt 4 s a
2
, a
3
, a
4
, a
5
cng c hai s c
hiu chia ht cho 3. Nh vy P c t nht hai hiu khc nhau
chia ht cho 3, tc 9 [ p.
2. Chng minh 32 [ P: Theo nguyn l Dirichlet th tng 5 s cho
tn ti t nht 3 s c cng tnh chn l. Ch c th c hai kh
nng sau xy ra:
Nu c t nht 4 s c cng tnh chn l, th t bn s c th
lp thnh su hiu khc nhau chia ht cho 2. Do 32 [ P.
Chuyn S hc Din n Ton hc
52 3.2. Phng php gii cc bi ton chia ht
Nu c 3 s c cng tnh chn l. Khng mt tnh tng qut,
gi s ba s l a
1
, a
2
, a
3
. Khi a
4
, a
5
cng cng tnh
chn l nhng li khc tnh chn l ca a
1
, a
2
, a
3
. Khi
cc hiu sau chia ht cho 2: a
1
a
2
, a
1
a
3
, a
2
a
3
, a
4
a
5
.
Mt khc, trong 5 s cho c t nht hai hiu chia ht cho
4, cho nn trong 4 hiu a
1
a
2
, a
1
a
3
, a
2
a
3
, a
4
a
5
c
t nht mt hiu chia ht cho 4. Vy 32 [ P.
Ta c pcm.
V d 3.27. Cho 2012 s t nhin bt k a
1
, a
2
, , a
2012
. Chng minh
rng tn ti mt s chia ht cho 2012 hoc tng mt s s chia ht cho
2012.
Li gii. Xt 2012 s
S
1
= a
2
S
2
= a
1
+a
2

S
2012
= a
1
+a
2
+ +a
2012
Trng hp 1: Nu tn ti s S
i
(i = 1, 2, , 2012) chia ht cho
2012 th bi ton chng minh xong.
Trng hp 2: Nu 2012 S
i
vi mi i = 1, 2, , 2012. em 2012
s ny chia cho 2012 nhn c 2012 s d. Cc s d nhn gi
tr thuc tp 1; 2; ; 2011. V c 2012 s d m ch c 2011
gi tr nn theo nguyn l Dirichlet chc chn c hai s d bng
nhau. Ga s gi hai s l S
m
v S
n
c cng s d khi chia
cho 2012 (m, n N, 1 n < m 2012) th hiu
S
m
S
n
= a
n+1
+a
n+2
+ +a
m
chia ht cho 2012.
Din n Ton hc Chuyn S hc
3.2. Phng php gii cc bi ton chia ht 53
Nhn xt. Ta c th rt ra bi ton tng qut v bi ton m rng
sau:
Bi ton 3.5 (Bi ton tng qut). Cho n s a
1
, a
2
, , a
n
. Chng
minh rng trong n s trn tn ti mt s chia ht cho n hoc tng mt
s s chia ht cho n.
Bi ton 3.6 (Bi ton m rng). (Tp ch Ton Tui Th s 115)
Cho n l mt s chuyn dng v n s nguyn dng a
1
, a
2
, , a
n
c
tng bng 2n 1. Chng minh rng tn ti mt s s trong n s
cho c tng bng n.
Bi tp ngh
Bi 1. Chng minh rng c v s s chia ht cho 2013
11
356
m trong
biu din thp phn ca cc s khng c cc ch s 0, 1, 2, 3.
Bi 2. (HSG 9 H Ni, 2006) Chng minh rng tn ti s t nhin
n ,= 0 tha mn 3
13579
[ (13579
n
1).
Bi 3. Chng minh rng trong 52 s nguyn dng bt k lun lun
tm c hai s c tng hoc hiu chia ht cho 100.
Bi 4. Cho 10 s nguyn dng a
1
, a
2
, , a
10
. Chng minh rng tn
ti cc s c
i
0, 1, 1, (i = 1, 10) khng ng thi bng
0 sao cho
A = c
1
a
1
+c
2
a
2
+ +c
10
a
10
chia ht cho 1032.
Bi 5. Chng minh rng tn ti s t nhin k sao cho 2002
k
1 chia
ht cho 2003
10
.
Bi 6. Bit rng ba s a, a +k, a +2k u l cc s nguyn t ln hn
3. Chng minh rng khi k chia ht cho 6.
Chuyn S hc Din n Ton hc
54 3.2. Phng php gii cc bi ton chia ht
3.2.7 Phn chng
C s: chng minh p A(n), ta lm nh sau:
Gi s ngc li p [ A(n).
Chng minh iu ngc li sai.
V d 3.28. Chng minh rng vi mi s nguyn n th n
2
+n+1 khng
chia ht cho 9.
Li gii. Gi s 9 [ (n
2
+n+1). Khi n
2
+n+1 = (n+2)(n1) +3
chia ht cho 3. Suy ra 3 [ n + 2 v 3 [ n 1. Nh vy (n + 2)(n 1)
chia ht cho 9, tc n
2
+n + 1 chia 9 d 3, mu thun. Ta c pcm.
Nhn xt. Bi ton ny vn c th gii theo phng php xt s d.
V d 3.29. Gi s p = k.2
t
+ 1 l s nguyn t l, t l s nguyn
dng v k l s t nhin l. Gi thit x v y l cc s t nhin m
p [
_
x
2
t
+y
2
t
_
. Chng minh rng khi x v y ng thi chia ht cho
p.
Li gii. Gi s tri li p x, suy ra p y.
Do p l s nguyn t nn theo nh l Fermat nh ta c
_
x
p1
1 (mod p)
y
p1
1 (mod p)
Theo gi thit th p 1 = k.2
t
, do
_
x
k.2
t
1 (mod p)
y
k.2
t
1 (mod p)
T ta c
x
k.2
t
+y
k.2
t
2 (mod p). (i)
Theo gi thit th
x
2
t
+y
2
t
0 (mod p).
Din n Ton hc Chuyn S hc
3.2. Phng php gii cc bi ton chia ht 55
Do k l nn
x
k.2
t
+y
k.2
t
=
_
x
2
t
_
k
+
_
y
2
t
_
k
.
.
.
_
x
2
t
+y
2
t
_

_
x
k.2
t
+y
k.2
t
_
0 (mod p) (ii)
T (i) v (ii) suy ra iu mu thun. Vy gi thit phn chng sai. Do
x, y ng thi chia ht cho p.
Bi tp ngh
Bi 1. Chng minh n
2
+n + 2 khng chia ht cho 15 vi mi n Z.
Bi 2. Chng minh n
2
+3n+5 khng chia ht cho 121 vi mi n N.
Bi 3. Chng minh 9n
3
+ 9n
2
+ 3n 16 khng chia ht cho 343 vi
mi n N.
Bi 4. Chng minh 4n
3
6n
2
+ 3n + 37 khng chia ht cho 125 vi
mi n N.
Bi 5. Chng minh n
3
+3n38 khng chia ht cho 49 vi mi n N.
Chuyn S hc Din n Ton hc
Chng
4
Phng trnh nghim
nguyn
4.1 Xt tnh chia ht 57
4.2 S dng bt ng thc 74
4.3 Nguyn tc cc hn, li v hn 86
Trn Nguyn Thit Qun (L Lawliet)
Phm Quang Ton (Phm Quang Ton)
Trong chng trnh THCS v THPT th phng trnh nghim nguyn
vn lun l mt ti hay v kh i vi hc sinh. Cc bi ton nghim
nguyn thng xuyn xut hin ti cc k thi ln, nh, trong v ngoi
nc. Trong bi vit ny ti ch mun cp n cc vn c bn ca
nghim nguyn (cc dng, cc phng php gii) ch khng i nghin
cu su sc v n. Ti cng khng cp ti phng trnh Pell, phng
trnh Pythagore, phng trnh Fermat v n c nhiu trong cc sch,
cc chuyn khc.
4.1 Xt tnh chia ht
4.1.1 Pht hin tnh chia ht ca 1 n
V d 4.1. Gii phng trnh nghim nguyn
13x + 5y = 175 (4.1)
57
58 4.1. Xt tnh chia ht
Li gii. Gi s x, y l cc s nguyn tha mn phng trnh (4.1). Ta
thy 175 v 5y u chia ht cho 5 nn 13x
.
.
.5 x
.
.
.5 (do GCD(13; 5) = 1).
t x = 5t (t Z). Thay vo phng trnh (4.1), ta c
13.5t + 5y = 175 13t +y = 35 y = 35 13t
Do , phng trnh (4.1) c v s nghim nguyn biu din di dng
(x; y) = (5t; 35 13t), (t Z)
Bi tp ngh
Bi 1. Gii phng trnh nghim nguyn 12x 19y = 285
Bi 2. Gii phng trnh nghim nguyn 7x + 13y = 65
Bi 3. Gii phng trnh nghim nguyn 5x + 7y = 112
4.1.2 a v phng trnh c s
V d 4.2. Tm nghim nguyn ca phng trnh
3xy + 6x +y 52 = 0 (4.2)
Li gii. Nhn xt. i vi phng trnh ny, ta khng th p dng
phng php trn l pht hin tnh chia ht, vy ta phi gii nh th
no?
Ta gii nh sau:
(4.2) 3xy +y + 6x + 2 54 = 0
y (3x + 1) + 2 (3x + 1) 54 = 0
(3x + 1) (y + 2) = 54
Nh vy, n y ta c x v y nguyn nn 3x + 1 v y + 2 phi l c
ca 54. Nhng nu nh vy th ta phi xt n hn 10 trng hp sao?
V:
4 = 1.54 = 2.27 = 3.18 = 6.9
= (1).(54) = (2).(27) = (3).(18) = (6).(9)
Din n Ton hc Chuyn S hc
4.1. Xt tnh chia ht 59
C cch no khc khng? Cu tr li l c! Nu ta mt cht n
tha s 3x + 1, biu thc ny chia cho 3 lun d 1 vi mi x nguyn.
Vi lp lun trn, ta c:
_

_
_
3x + 1 = 1
y + 2 = 54

_
x = 0
y = 52
_
3x + 1 = 2
y + 2 = 54

_
x = 1
y = 56
V d 4.3. Gii phng trnh nghim nguyn sau:
2x + 5y + 3xy = 8 (4.3)
Li gii. Ta c
(4.3) x(2 + 3y) + 5y = 8
3x(2 + 3y) + 15y = 24
3x(2 + 3y) + 5(2 + 3y) = 34
(3x + 5)(3y + 3) = 34
n y phn tch 34 = 1 34 = 2 17 ri xt cc trng hp. Ch
rng 3x + 5, 3y + 2 l hai s nguyn chia 3 d 2, vn dng iu ny ta
c th gim bt s trng hp cn xt.
V d 4.4. Gii phng trnh nghim nguyn
x
2
y
2
= 2011 (4.4)
Li gii. (4.4) (x y)(x + y) = 2011. V 2011 l s nguyn t nn
c nguyn ca 2011 ch c th l 1, 2011. T suy ra nghim
(x; y) l (1006; 1005); (1006; 1005); (1006; 1005); (1006; 1005).
V d 4.5. Tm cc s nguyn x, y tho mn iu kin
x
2
+y
2
= (x y)(xy + 2) + 9 (4.5)
Chuyn S hc Din n Ton hc
60 4.1. Xt tnh chia ht
Li gii. t a = x y, b = xy. Khi (4.5) tr thnh
a
2
+ 2b = a(b + 2) + 9 (a 2)(a b) = 9 (4.6)
V x, y Z nn a, , a 2, a b u l cc s nguyn. T (4.6) ta c cc
trng hp sau:

_
a 2 = 9
a b = 1

_
a = 11
b = 10

_
x y = 11
xy = 10
(4.7)

_
a 2 = 3
a b = 3

_
a = 5
b = 2

_
x y = 5
xy = 2
(4.8)

_
a 2 = 1
a b = 9

_
a = 3
b = 6

_
x y = 3
xy = 6
(4.9)

_
a 2 = 1
a b = 9

_
a = 1
b = 10

_
x y = 1
xy = 10
(4.10)

_
a 2 = 3
a b = 3

_
a = 1
b = 2

_
x y = 1
xy = 2
(4.11)

_
a 2 = 3
a b = 3

_
a = 1
b = 2

_
x y = 1
xy = 2
(4.12)
D thy cc h (4.7),(4.8),(4.10) khng c nghim nguyn, h (4.9) v
nghim, h (4.11) c hai nghim nguyn (1; 2) v (2; 1), h (4.12)
c hai nghim nguyn (1; 6) v (6; 1).
Tm li phng trnh (4.5) c cc cp nghim nguyn (x; y) l (1; 2);
(2; 1); (1; 6); (6; 1).
V d 4.6. Tm nghim nguyn ca phng trnh:
_
x
2
+ 1
_ _
y
2
+ 1
_
+ 2 (x y) (1 xy) = 4 (1 +xy) (4.13)
Din n Ton hc Chuyn S hc
4.1. Xt tnh chia ht 61
Li gii. Phng trnh (4.13) tng ng vi:
x
2
y
2
+x
2
+y
2
+ 1 + 2x 2x
2
y 2y + 2xy
2
= 4 + 4xy
(x
2
+ 2x + 1)y
2
2(x
2
+ 2x + 1)y + (x
2
+ 2x + 1) = 4
(x + 1)
2
(y 1)
2
= 4

_
(x + 1)(y 1) = 2
(x + 1)(y 1) = 2
Vi (x + 1)(y 1) = 2 m x, y Z nn ta c cc trng hp sau:

_
x + 1 = 1
y 1 = 2

_
x = 0
y = 3

_
x + 1 = 2
y 1 = 1

_
x = 1
y = 2

_
x + 1 = 2
y 1 = 1

_
x = 3
y = 0

_
x + 1 = 1
y 1 = 2

_
x = 2
y = 1
Vi (x + 1)(y 1) = 2 , tng t ta cng suy ra c:

_
x + 1 = 1
y 1 = 2

_
x = 2
y = 3

_
x + 1 = 1
y 1 = 2

_
x = 0
y = 1

_
x + 1 = 2
y 1 = 1

_
x = 1
y = 0

_
x + 1 = 2
y 1 = 1

_
x = 3
y = 2
Vy phng trnh cho c cc cp nghim nguyn:
(x; y) = (0; 3); (1; 2); (3; 0); (2; 1); (2; 3); (0; 1); (1; 0); (3; 2)
V d 4.7. Tm nghim nguyn ca phng trnh
x
6
+ 3x
3
+ 1 = y
4
(4.14)
Chuyn S hc Din n Ton hc
62 4.1. Xt tnh chia ht
Li gii. Nhn hai v ca phng trnh (4.14) cho 4, ta c:
4x
6
+ 12x
3
+ 4 = 4y
4
(4x
6
+ 12x
3
+ 9) 4y
4
= 5
(2x
3
+ 3)
2
4y
4
= 5
(2x
3
2y
2
+ 3)(2x
3
+ 2y
2
+ 3) = 5.
Vi lu rng 5 = 1.5 = 5.1 = (1).(5) = (5).(1) v x, y Z nn
ta suy ra c cc trng hp sau:

_
2x
3
2y
2
+ 3 = 1
2x
3
+ 2y
2
+ 3 = 5

_
x
3
y
2
= 1
x
3
+y
2
= 1

_
x
3
= 0
y
2
= 1

_
_
x = 0
y = 1
_
x = 0
y = 1

_
2x
3
2y
2
+ 3 = 1
2x
3
+ 2y
2
+ 3 = 5

_
x
3
y
2
= 2
x
3
+y
2
= 4

_
x
3
= 3
y
2
= 1
(loi)

_
2x
3
2y
2
+ 3 = 5
2x
3
+ 2y
2
+ 3 = 1

_
x
3
y
2
= 1
x
3
+y
2
= 1

_
x
3
= 0
y
2
= 1
(loi)

_
2x
3
2y
2
+ 3 = 5
2x
3
+ 2y
2
+ 3 = 1

_
x
3
y
2
= 4
x
3
+y
2
= 2

_
x
3
= 3
y
2
= 1
(loi)
Vy phng trnh cho c cc cp nghim nguyn:
(x; y) = (0; 1); (0; 1)
Nhn xt. Bi ton ny cng c th gii bng phng php kp.
V d 4.8. Gii phng trnh nghim nguyn dng:
1
x
+
1
y
=
1
p
(4.15)
trong p l s nguyn t.
Din n Ton hc Chuyn S hc
4.1. Xt tnh chia ht 63
Li gii.
(4.15) xy = px +py (x y)(y p) = p
2
.
V p l s nguyn t nn c s nguyn ca p
2
ch c th l 1, p, p
2
.
Th ln lt vi cc c trn ta d tm c kt qu. Phn trnh by
xin dnh cho bn c.
Nhn xt. Phng php ny cn hai bc chnh: Phn tch thnh c
s v xt trng hp tm kt qu. Hai bc ny c th ni l khng
qu kh i vi bn c, nhng xin ni mt s lu thm v bc xt
trng hp. Trong mt s bi ton, hng s nguyn v phi sau khi
phn tch l mt s c nhiu c, nh vy i hi xt trng hp v
tnh ton rt nhiu. Mt cu hi t ra l: Lm th no gim s
trng hp b xt y? V tr li c cu hi , ta s tham kho
v d di y.
V d 4.9. Tm nghim nguyn ca phng trnh:
x
2
+ 12x = y
2
. (4.16)
Li gii. (thng thng) Phng trnh (4.16) cho tng ng vi:
(x + 6)
2
y
2
= 36 (x + 6 +y)(x + 6 y) = 36
Suy ra x + y + 6, x + 6 y l c ca 36. M s 36 c tt c 18 c
nn ta phi xt 18 trng hp tng ng vi
x + 6 +y 1; 2; 3; 4; 6; 9; 12; 18; 36
. Kt qu l ta tm c cc cp nghim nguyn (x; y) l
(0; 0); (12; 0); (16; 8); (16; 8); (4; 8); (4; 8)
.
Nhn xt. ng nh vn m ta nu ra trn, s c qu nhiu
xt. Cho nn ta s c cc nhn xt sau thc hin thao tc "siu
phm" chuyn t con s 18 xung ch cn 2!
Chuyn S hc Din n Ton hc
64 4.1. Xt tnh chia ht
V y c s m chn trong phng trnh nn c th gi s y 0. Khi
x + 6 y x + 6 + y, do vy ta loi c tm trng hp v cn
li cc trng hp sau:
_
x + 6 +y = 9
x + 6 y = 4
,
_
x + 6 +y = 9
x + 6 y = 4
,
_
x +y + 6 = 1
x +y 6 = 36
,
_
x +y + 6 = 36
x y + 6 = 1
,
_
x +y + 6 = 2
x y + 6 = 18
,
_
x +y + 6 = 18
x y + 6 = 2
,
_
x +y + 6 = 3
x y + 6 = 12
,
_
x +y + 6 = 12
x y + 6 = 3
,
_
x +y + 6 = 6
x y + 6 = 6
,
_
x +y + 6 = 6
x +y 6 = 6
.
By gi ta c 10 trng hp, ta s tip tc lc b. Nhn thy
(x +y + 6) (x + 6 y) = 2y nn x + 6 y v x + 6 +y c cng tnh
chn l, do ta loi thm 6 trng hp, ch cn
_
x +y + 6 = 18
x +y 6 = 2
,
_
x +y + 6 = 2
x +y 6 = 18
,
_
x +y + 6 = 6
x y + 6 = 6
,
_
x +y + 6 = 6
x +y 6 = 6
.
Tip tc xt hai phng trnh
_
x +y + 6 = 6
x y + 6 = 6
v
_
x +y + 6 = 6
x +y 6 = 6
,
hai phng trnh ny u tm c y = 0. Vy sao khng n gin
hn, ta xt y = 0 ngay t u. Phng trnh c dng x(x + 12) = y
2
,
xt hai kh nng:
Nu y = 0 th x = 0 hoc x = 12.
Nu y ,= 0 th x+6+y > x+6y, p dng hai nhn xt trn ta ch
c hai trng hp:
_
x +y + 6 = 2
x y + 6 = 18
v
_
x +y + 6 = 18
x y + 6 = 2
.

Din n Ton hc Chuyn S hc


4.1. Xt tnh chia ht 65
Phng trnh cho c 6 nghim nguyn
(x; y) = (16; 8), (0; 0), (12; 0), (16; 8), (4; 8), (4; 8)
Nhn xt. Nh vy bi ton ngn gn, chnh xc nh linh hot trong
vic xt tnh chn l, gii hn hai s gim s trng hp cn xt.
Ngoi cc cch nh gi trn ta cn c th p dng xt s d tng v
nh gi (y cng l mt phng php gii phng trnh nghim
nguyn).
Bi tp ngh
Bi 1. Th bin i cc bi ton gii phng trnh nghim nguyn
phng php Biu th mt n theo n cn li bng phng
php a v c s.
Bi 2. Tm di cnh mt tam gic vung sao cho tch hai cnh
huyn gp ba ln chu vi tam gic .
Bi 3. Gii phng trnh nghim nguyn x y + 2xy = 6
Bi 4. Gii phng trnh nghim nguyn 2x + 5y + 2xy = 8
Bi 5. (Thi HSG lp 9 tnh Qung Ngi nm 2011-2012) Gii phng
trnh nghim nguyn 6x + 5y + 18 = 2xy
Bi 6. Tm nghim nguyn (xy 7)
2
= x
2
+y
2
Bi 7. Tm x, y Z tha mn 2x
2
2xy = 5x y 19.
Bi 8. Tm nghim nguyn ca phng trnh x
2
+6xy+8y
2
+3x+6y =
2.
Bi 9. Tm nghim nguyn dng ca phng trnh x
3
y
3
= xy +61
Bi 10. Tm nghim nguyn ca phng trnh 4x
2
y
2
= 22 +x(1 +x) +
y(1 +y)
Bi 11. Gii phng trnh nghim nguyn x(x + 1)(x + 7)(x + 8) = y
2
.
Chuyn S hc Din n Ton hc
66 4.1. Xt tnh chia ht
Bi 12. Tm nghim nguyn dng ca phng trnh 6x
3
xy(11x +
3y) + 2y
3
= 6 (Tp ch TTT2 s 106).
Bi 13. Tm nghim nguyn dng ca phng trnh x(x+2y)
3
y(y +
2x)
3
= 27 (tp ch THTT s 398).
Bi 14. Tm nghim nguyn ca phng trnh

9x
2
+ 16x + 96 = 3x
16y 24.
Bi 15. Tm nghim nguyn dng ca phng trnh
2 +

x +
1
2
+
_
x +
1
4
= y
.
Bi 16. Tm s nguyn x x
2
4x 52 l s chnh phng.
Bi 17. Gii phng trnh nghim nguyn x
2
+2y
2
+3xy 2x y = 6.
Bi 18. Gii phng trnh nghim nguyn x
2
+3xy y
2
+2x 3y = 5.
Bi 19. Gii phng trnh nghim nguyn 2x
2
+3y
2
+xy 3x 3 = y.
Bi 20. (Tuyn sinh vo lp 10 THPT chuyn trng KHTN H Ni
nm hc 2012-2013) Tm tt c cc cp s nguyn x, y tha
mn ng thc (x +y + 1)(xy +x +y) = 5 + 2(x +y).
Bi 21. Gii phng trnh nghim nguyn x
4
2y
4
x
2
y
2
4x
2
7y
2

5 = 0.
(Thi HSG lp 9 tnh Hng Yn nm 2011-2012)
Bi 22. (Romanian 1999) Chng minh rng phng trnh sau khng c
nghim nguyn
x
5
x
4
y 13x
3
y
2
+ 13x
2
y
3
+ 36xy
4
36y
5
= 1937
Din n Ton hc Chuyn S hc
4.1. Xt tnh chia ht 67
4.1.3 Biu th mt n theo n cn li ri s dng tnh chia
ht
V d 4.10. Tm nghim nguyn ca phng trnh
2x xy + 3 = 0 (4.17)
Li gii. Nhn xt. phng trnh ny ta khng th p dng cc cch
bit, vy ta phi lm sao? Ch hn mt xu na ta thy c th
biu din y theo x c ri vn dng kin thc tm gi tr nguyn
lp 8 tm nghim nguyn ca phng trnh, th lm theo tng
xem sao.
(4.17) xy = 2x + 3
Nu x = 0 th phng trnh (4.17) cho v nghim nguyn y.
Nu x ,= 0 th
(4.17) y =
2x + 3
x
= 2 +
3
x
Nh vy mun y nguyn th ta cn
3
x
nguyn hay ni cch khc x l
c ca 3. Vi mi gi tr nguyn x ta tm c mt gi tr y nguyn.
T , ta c b nghim ca (4.17) l
(x; y) = (3; 1); (1; 1); (1; 5); (3; 3)
V d 4.11 (Thi HSG lp 9 Qung Ngi nm 2011-2012). Tm cc s
nguyn dng x, y sao cho
6x + 5y + 18 = 2xy (4.18)
Nhn xt. Hng phn tch v nh hng li gii. xc nh c
phng php ca dng ny th by gi ta s biu din n x theo y.
Khng kh vit thnh x =
5y 18
6 2y
. Ta dng nh nhn thy biu
thc ny rt kh phn tch nh biu thc v d u. Tuy nhin, nu
k s thy bn mu l 2y v t l 5y, do ta mnh dn nhn 2
vo t xut hin 2y ging nh mu.
Chuyn S hc Din n Ton hc
68 4.1. Xt tnh chia ht
Li gii. Ta c
(4.18) x =
5y 18
6 2y
2x =
10y 36
6 2y
2x =
66 + 5(6 2y)
6 2y
=
66
6 2y
+ 5
2x =
33
3 y
+ 5
Nh vy mun x l s nguyn dng th 3 y l phi l c ca 33.
Hay 3 y 1; 3; 11, 33. Li rng v y 1 nn 3 y 2.
Do ch c th 3 y 1; 3; 11; 33. Ta c bng sau:
3 y 1 1 3 11 33
y 2 4 6 14 36
x 14 19 8 4 3
Th li thy cc cp (x; y) nguyn dng tha mn (4.18) l (x; y) =
(19; 4), (8; 6), (4; 14), (3; 36).
Nhn xt. Bi ny ta cng c th s dng phng php a v phng
trnh c s. Cng xin ch vi bn rng li gii trn th ta nhn
2 x bin i, do phi c mt bc th li coi gi tr x, y tm
c c tha mn (4.18) hay khng ri mi c th kt lun.
Bi tp ngh
Bi 1. Gii phng trnh nghim nguyn x
2
xy = 6x 5y 8.
Bi 2. Gii phng trnh nghim nguyn x
2
+x + 1 = 2xy +y.
Bi 3. Gii phng trnh nghim nguyn x
3
x
2
y + 3x 2y 5 = 0.
Bi 4. (Vo 10 chuyn THPT HKHTN H Ni nm 2001-2002) Tm
gi tr x, y nguyn tha mn ng thc (y 2)x
2
+ 1 = y
2
.
Din n Ton hc Chuyn S hc
4.1. Xt tnh chia ht 69
Bi 5. (Vo 10 chuyn THPT HKHTN H Ni nm 2000-2001) Tm
cp s nguyn (x, y) tha mn ng thc y(x 1) = x
2
+ 2.
Bi 6. Tm s nh nht trong cc s nguyn dng l bi ca 2007 v
c 4 ch s cui cng l 2008.
Bi 7. Tm nghim nguyn ca phng trnh 5x 3y = 2xy 11.
4.1.4 Xt s d tng v
C s phng php. c ngay tiu phng php th chc bn s
hiu ngay phng php ny ni n vic xt s d tng v cho cng
mt s. Vy, ti sao li phi xt v xt nh vy c li ch g trong "cng
cuc" gii ton? Hy cng tm hiu qua v d u sau:
V d 4.12. Tm nghim nguyn ca phng trnh
x
2
+y
2
= 2011 (4.19)
Li gii. Ta c x
2
; y
2
chia 4 c th d 0 hoc 1 nn tng chng chia 4
ch c th d 0; 1 hoc 2. Mt khc 2011 chia 4 da 3 nn phng trnh
(4.19) v nghim nguyn.
Nhn xt. Qua v d u ny th ta thy r s d khi chia cho 4 ca
hai s khc nhau th phng trnh v nghim. Do ta li cng hiu
thm mc ch ca phng php ny. Bt m thm t na th phng
php ny ch yu dng cho cc phng trnh khng c nghim nguyn.
Cho nn, nu bn bt gp mt phng trnh bt k m bn khng th
tm ra c nghim cho phng trnh , th hy ngh n phng
php ny u tin. Cn by gi ta tip tc n vi v d sau:
V d 4.13 (Balkan MO 1998). Tm nghim nguyn ca phng trnh
x
2
= y
5
4 (4.20)
Li gii. Ta c: x
2
0; 1; 3; 4; 5; 9 (mod 11). Trong khi y
5
4
6; 7; 8 (mod 11): v l. Vy phng trnh (4.20) v nghim nguyn.
Chuyn S hc Din n Ton hc
70 4.1. Xt tnh chia ht
Nhn xt. Mt cu hi na li le ln trong u ta: Lm th no li c
th tm c con s 11 m xt ng d c nh? p n ca cu
hi ny cng chnh l ci ct li bn vn dng phng php ny, v
cng l nhng kinh nghim sau:
1. i vi phng trnh nghim nguyn c s tham gia ca cc bnh
phng th ta thng xt ng d vi 3, 4, 5, 8. C th l:
a
2
0, 1 (mod 3)
a
2
0, 1 (mod 4)
a
2
0, 1, 4 (mod 5)
a
2
0, 1, 4 (mod 8)
2. i vi cc phng trnh nghim nguyn c s tham gia ca cc
s lp phng th ta thng xt ng d vi 9, v x
3
0; 1; 8
(mod 9) v ng d vi 7, v x
3
0, 1, 6 (mod 7).
3. i vi phng trnh nghim nguyn c s tham gia ca cc
ly tha bc 4 th ta thng xt ng d vi 8, nh: z
4
0, 1
(mod 8).
4. Mt vn cui cng l nh l Fermat: i vi phng trnh
nghim nguyn c s tham gia ca cc ly tha c s m l mt
s nguyn t hay l mt s m khi cng 1 vo s ta c mt
s nguyn t th ta thng s dng nh l nh Fermat xt
ng d.
Trn y l mt s kinh nghim bn thn, cn nu cc bn mun vn
dng c phng php xt s d ny, yu cu hy ghi nh kinh nghim
trn v tm cch chng minh n. Ngoi ra, nu bn mun m rng tm
hiu bit hn na, bn c th tm cc ng d vi ly tha khc nhau
(chng hn qua v d 2 ta rt ra c moun 11 cho ly tha bc
hai, bc nm). Cn by gi, hy th xem kinh nghim trn c hiu qu
khng nh!
V d 4.14 (Bi ton trong tun - diendantoanhoc.net). Chng minh
rng phng trnh sau khng c nghim nguyn
x
10
+y
10
= z
10
+ 199
Din n Ton hc Chuyn S hc
4.1. Xt tnh chia ht 71
Nhn xt. Thng thng cc bi ton khi t cu hi phng trnh
c nghim hay khng th thng c cu tr li l khng. Do
chng minh phng trnh trn khng c nghim, th ta s tm mt con
s sao cho khi chia VT v VP cho con s ny th c hai s d khc
nhau.
Nh vy, cng vic by gi ca ta l tm con s . n s m 10
th s khin ta lin tng con s 11 l s nguyn t. Nh vy li gii
ca ta s p dng nh l Fermat nh cho s 11 chng minh hai v
phng trnh chia cho 11 khng cng s d.
Li gii. p dng nh l Fermat nh th
_

_
x
10
0, 1 (mod 11)
y
10
0, 1 (mod 11)
z
10
0, 1 (mod 11)
.
Do x
10
+y
10
z
10
0, 1, 2, 10 (mod 11) m 199 8 (mod 11) nn
phng trnh v nghim nguyn.
V d 4.15 ( thi chn HSG ton quc gia nm 2003 - Bng B).
H phng trnh sau c tn ti nghim nguyn hay khng:
x
2
+y
2
= (x + 1)
2
+u
2
= (x + 2)
2
+v
2
= (x + 3)
2
+t
2
(4.21)
Nhn xt. Ta d on phng trnh trn cng s v nghim. Do
cn tm mt s v khi chia c 5 v c cc s d khc nhau. bi
ton ny c bnh phng nn ta ngh ti vic s dng cc tnh cht
nh: a
2
0, 1 (mod 3), a
2
0, 1 (mod 4), a
2
0, 1, 4 (mod 5), a
2

0, 1, 4 (mod 8). bi ton ny, ta s chn 8. By gi ch cn xt tnh


d khi chia cho 8.
Li gii. Gi s phng trnh (4.21) c nghim nguyn (x
0
, y
0
, u
0
, v
0
, t
0
),
tc l:
x
2
0
+y
2
0
= (x
0
+ 1)
2
+u
2
0
= (x
0
+ 2)
2
+v
2
0
= (x
0
+ 3)
2
+t
2
0
(4.22)
Vi a Z th a
2
0, 1, 4 (mod 8). Ta xt cc kh nng sau:
Chuyn S hc Din n Ton hc
72 4.1. Xt tnh chia ht
1. Nu x
0
0 (mod 4) th x
2
0
+y
2
0
0, 1, 4 (mod 8). V
x
0
+ 1 1 (mod 8) (x
0
+ 1)
2
1 (mod 8)
(x
0
+ 1)
2
+u
2
0
1, 2, 5 (mod 8)
x
0
+ 2 2 (mod 4) (x
0
+ 2)
2
4 (mod 8)
(x
0
+ 2)
2
+v
2
0
0, 4, 5 (mod 8)
x
0
+ 3 3 (mod 4) (x
0
+ 3)
2
1 (mod 8)
(x
0
+ 3)
2
+t
2
0
1, 2, 5 (mod 8)
Nhn thy 0, 1, 4 1, 2, 5 0, 4, 5 1, 2, 5 = 0 nn do
phng trnh khng c nghim nguyn vi x 0 (mod 4).
2. Tng t vi x
0
1 (mod 4), x
0
2 (mod 4) v x
0
3 (mod 4)
ta cng thc hin tng t v cng cho kt qu phng trnh
khng c nghim nguyn.
Vy phng trnh (4.21) cho khng c nghim nguyn.
Nhn xt. V d 4 ta c th tng qut ln:
V d 4.16. Tm s nguyn dng n ln nht sao cho h phng trnh
(x + 1)
2
+y
2
1
= (x + 2)
2
+y
2
2
= . . . = (x +n)
2
+y
2
n
c nghim nguyn.
y cng chnh l thi chn i tuyn HSG quc gia ton nm 2003
- Bng A. Li gii xin ginh cho bn c. Cng xin ni thm mt tha
nhn rng, phng php xt s d tng v ny, chng ta c tng
chng nh n gin, nhng thc cht khng phi th. Dn chng l
cc v d trn, u l cc bi ton hay v kh ly t khc cuc thi
trong nc v ngoi nc.
Bi tp ngh
Bi 1. Cho a thc f(x) c cc h s nguyn. Bit rng f(1).f(2) l s
l. Chng minh rng phng trnh f(0) = 0 khng c nghim
nghim nguyn.
Din n Ton hc Chuyn S hc
4.1. Xt tnh chia ht 73
Bi 2. Tn ti hay khng nghim nguyn ca phng trnh x
12
+y
12
+
z
12
= 2
_
37
2012
+ 2014
1995
_
.
Bi 3. Gii phng trnh nghim nguyn 31
2x
+ 12
2x
+ 1997
2x
= y
2
.
Bi 4. Gii phng trnh nghim nguyn dng 7
z
= 2
x
3
y
1
Bi 5. Gii phng trnh nghim nguyn dng 2
x
3
y
= 1 + 5
z
Bi 6. Gii phng trnh nghim t nhin 19
x
+5
y
+1890 = 1975
4
30
+
1993.
Bi 7. Gii phng trnh nghim nguyn x
3
+y
3
+z
3
= 1012
Bi 8. (Tuyn sinh vo lp 10 chuyn Trn Ph, Hi Phng nm hc
2012-2013) x
4
+y
4
+z
4
= 2012
Bi 9. [x y[ +[y z[ +[z x[ =
10
n
1
9
vi mi n N
Bi 10. Tm nghim nguyn ca phng trnh (2
x
+ 1)(2
x
+ 2)(2
x
+
3)(2
x
+ 4) 5
y
= 11879
Bi 11. Tm nghim nguyn ca phng trnh x
2
+(x+1)
2
+(x+2)
2
=
y
2
.
Bi 12. (Tuyn sinh vo THPT chuyn HKHTN H Ni nm 2011-
2012) Chng minh rng khng tn ti b ba s nguyn (x; y; z)
tha mn x
4
+y
4
= 7z
4
+ 5.
Bi 13. Gii phng trnh nghim nguyn x
4
1
+x
4
2
+ = x
4
13
+20122015.
Bi 14. Cho p l s nguyn t l. Chng minh rng phng trnh x
p
+
y
p
= p [(p 1)!]
p
khng c nghim nguyn
Bi 15. Tm nghim nguyn ca phng trnh x
2012
y
2010
= 7.
Bi 16. Chng minh rng khng tn ti s nguyn x, y tha mn x
5
+
y
5
+ 1 = (x + 2)
5
+ (y 3)
5
.
Chuyn S hc Din n Ton hc
74 4.2. S dng bt ng thc
4.2 S dng bt ng thc
4.2.1 Sp th t cc n
V d 4.17. Gii phng trnh nghim nguyn dng sau
1
x
+
1
y
+
1
z
= 1 (4.23)
Li gii. Khng mt tnh tng qut, ta c th gi s
1 x y z
1
x
+
1
y
+
1
z
= 1
3
x
x 3
Vi x = 1 th (4.23) khng c nghim nguyn dng.
Vi x = 2 th
1
2
+
1
y
+
1
z
= 1
1
y
+
1
z
=
1
2

2
y
y 4 Mt
khc, y x = 2 y 2; 3; 4. Ta th ln lt cc gi tr ca y
Vi y = 2 th (4.23) v nghim nguyn.
Vi y = 3 th z = 6.
Vi y = 4 th z = 4.
Vi x = 3, ta c
1
3
+
1
y
+
1
z
= 1
1
y
+
1
z
=
2
3

2
y
y 3 Mt
khc, do y x = 3 y = 3 z = 3
Vy nghim nguyn (x; y; z) ca (4.23) l hon v ca cc b (2; 3; 6);
(2; 4; 4); (3; 3; 3).
Nhn xt. Phng php ny c s dng ch sp th t cc n
1 x y z ri gii hn nghim gii.
Ta ch s dng phng php sp th t cc n khi vai tr cc n l
bnh ng vi nhau. D khi vn dng phng php ny cc bn cn
ch trnh nhm ln. C th, ta s n vi v d sau:
V d 4.18. Gii phng trnh nghim nguyn dng
x +y + 1 = xyz (4.24)
Din n Ton hc Chuyn S hc
4.2. S dng bt ng thc 75
Li gii (Li gii sai). Khng mt tnh tng qut, gi s 1 x y
z. Khi x+y+1 3z hay xyz 3z, suy ra xy 3. M z y x 1
nn x = y = z = 1.
Nhn xt. Ci li sai li gii ny l do x, y, z khng bnh ng, nn
khng th sp th t cc n nh trn. Sau y l li gii ng:
Li gii. Khng mt tnh tng qut, gi s 1 x y. Ta xt trng
hp:
Nu x = y th
(4.24) 2y + 1 = y
2
z
y(z 2) = 1

_
y = 1
yz 2 = 1

_
y = 1
z = 3
Nu x < y th t (4.24) suy ra 2y + 1 > xyz. 2y xyz
xz 2 xz 1; 2.
Vi xz = 1 x = z = 1, thay vo (4.24) suy ra y + 2 = y
(v nghim).
Vi xz = 2
_
x = 1
z = 2
hoc
_
x = 2
z = 1
. T y ta tm
c nghim x = 1, y = 2, z = 2 hoc x = 1, y = 3, z = 1.
Vy phng trnh c nghim nguyn dng l (1; 1; 3), (1; 2; 2), (2; 1; 2),
(2; 3; 1), (3; 2; 1).
Nhn xt. By gi bn hiu v cch sp xp cc n nh th no.
Nhng ti sao bi ny li xt x = y v x < y m li khng i vo phn
Chuyn S hc Din n Ton hc
76 4.2. S dng bt ng thc
tch lun nh bi trc. Nu bn rng nu khng phn chia thnh
hai trng hp nhu trn th phng trnh (4.24) s thnh 2y+1 y
2
z,
rt kh tip tc phn tch ra nghim. Do vic xt nhu trn l
hp l.
Bi tp ngh
Bi 1. Gii phng trnh nghim nguyn dng 2(x+y+z)+9 = 3xyz.
Bi 2. Gii phng trnh nghim nguyn dng xyz = 3(x +y +z).
Bi 3. Gii phng trnh nghim nguyn dng 5(x+y +z +t) +10 =
2xyzt
Bi 4. Gii phng trnh nghim nguyn dng x! +y! = (x +y)!
(K hiu x! l tch cc s t nhin lin tip t 1 n x).
Bi 5. Tm nghim nguyn dng ca phng trnh x
3
+7y = y
3
+7x.
Bi 6. Tm nghim nguyn dng ca phng trnh x
1
+x
2
+ +x
12
=
x
1
x
2
x
12
.
Bi 7. Tm tt c cc nghim nguyn dng ca phng trnh
x
y
2
z
2
+
y
z
2
x
2
+
z
x
2
y
2
= t.
Bi 8. Tm nghim nguyn dng ca phng trnh x! +y! +z! = u!.
4.2.2 S dng bt ng thc
Nhn xt. gii phng trnh ny, ta thng s dng cc bt ng
thc quen thuc nh gi mt v ca phng trnh khng nh hn
(hoc khng ln hn) v cn li. Mun cho hai v bng nhau th bt
ng thc phi tr thnh ng thc.
C th, ta c mt s bt ng thc c bn thng dng:
1. Bt ng thc Cauchy (hay cn gi l bt ng thc AM-GM):
Nu a
1
, a
2
, , a
n
l cc s thc khng m th
a
1
+a
2
+ +a
n
n

n

a
1
a
2
a
n
Din n Ton hc Chuyn S hc
4.2. S dng bt ng thc 77
Du ng thc xy ra khi v ch khi a
1
= a
2
= = a
n
.
2. Bt ng thc Bunhiacopxki (hay cn c gi l bt ng thc
Cauchy - Bunyakovsky - Schwarz): Vi hai b s thc bt k
(a
1
, a
2
, , a
n
) v (b
1
, b
2
, , b
n
), ta c
_
a
2
1
+a
2
2
+ +a
2
n
_ _
b
2
1
+b
2
2
+ +b
2
n
_

(a
1
b
1
+a
2
b
2
+ +a
n
b
n
)
2
.
ng thc xy ra khi v ch khi tn ti s thc k sao cho a
i
= kb
i
vi mi i = 1, 2, , n.
3. Bt ng thc Trebusep (hay cn vit l bt ng thc Chebyshev):
Cho dy hu hn cc s thc c sp theo th t a
1
a
2

a
n
v b
1
b
2
b
n
. Khi ta c:
n(a
1
b
1
+a
2
b
2
+ +a
n
b
n
) (a
1
+a
2
+ +a
n
)(b
1
+b
2
+ +b
n
)
Du ng thc xy ra khi v ch khi
_
a
1
= a
2
= = a
n
b
1
= b
2
= = b
n
.
By gi ta s cng xem xt mt s v d sau:
V d 4.19. Gii phng trnh nghim nguyn dng sau:
x
6
+z
3
15x
2
z = 3x
2
y
2
z (y
2
+ 5)
3
(4.25)
Li gii. Nhn xt. phng trnh ny khi mi nhn vo hn a s
cc bn s c phn ri, khng xc nh c phng php lm, khng
vn dng c cc phng php hc. Tuy nhin nu k mt
x th ta thy x
6
= (x
2
)
3
iu ny c g c bit? Ta thy (x
2
)
3
, z
3
v
(y
2
+ 5)
3
u c cng bc ba v bi cho nguyn dng nn ta
ngh ngay n mt Bt ng thc kinh in: Bt ng thc Cauchy
hay cn gi l bt ng thc AM-GM.
Ta gii nh sau
(4.25) (x
2
)
3
+ (y
2
+ 5)
3
+z
3
= 3x
2
z(y
2
+ 5)
Chuyn S hc Din n Ton hc
78 4.2. S dng bt ng thc
p dng Bt ng thc AM-GM cho b ba s dng (x
2
)
3
, z
3
v (y
2
+
5)
3
ta c:
(x
2
)
3
+(y
2
+5)
3
+z
3
3
3
_
(x
2
)
3
.(y
2
+ 5)
3
.z
3
= 3x
2
z(y
2
+5) = V P(4.25)
Du bng ch xy ra khi x
2
= y
2
+ 5 = 5.
Mt khc ta c:
x
2
= y
2
+ 5 (x y)(x +y) = 5
y l mt dng phng trnh nghim nguyn quen thuc ta hc,
ti tin chc cc bn u c th d dng gii phng trnh trn, v t
x; y trn ta c th tm c z mt cch d dng.
p s: Nghim nguyn ca phng trnh (4.25) l (x; y; z) = (3; 2; 9).
V d 4.20. Tm nghim nguyn ca phng trnh
(x +y +z)
2
= 3(x
2
+y
2
+ 1)
Li gii. p dng bt ng thc Bunyakovsky cho hai b s (x, y, 1)
v (1, 1, 1) ta c
(x +y + 1)
2
(1
2
+ 1
2
+ 1
2
)(x
2
+y
2
+ 1) = 3(x
2
+y
2
+ 1)
ng thc xy ra khi v ch khi x = y = 1.
Vy phng trnh c nghim nguyn l (x, y) = (1, 1).
Nhn xt. Cc bi Ton v phng trnh nghim nguyn m gii bng
cch s dng Bt ng thc rt t dung v rt d b l dng nu
ngi ra khng kho lo. Tuy nhin, ta vn phi thnh tho phng
php ny khng c xem thng n trnh nhng sai lm ng tic
khng th sa c.
Bi tp ngh
Bi 1. Tm nghim nguyn dng x, y tha mn phng trnh (x
2
+
1)(x
2
+y
2
) = 4x
2
y
Bi 2. Tm nghim nguyn ca phng trnh
xy
z
+
yz
x
+
zx
y
= 3.
Din n Ton hc Chuyn S hc
4.2. S dng bt ng thc 79
Bi 3. ( thi tuyn sinh vo i hc Vinh) Tm nghim nguyn ca
phng trnh
(x
2
+ 1)(y
2
+ 4)(z
2
+ 9) = 48xyz
Bi 4. Gii phng trnh nghim nguyn
4

x 2
+
1

y 1
+
25

z 5
= 16

x 2
_
y 1

z 5
Bi 5. Tm nghim nguyn ca h phng trnh
_

_
x
2
+z
2
= 9
y
2
+t
2
= 16
xt +yz = 12
Bi 6. Tm nghim nguyn dng ca phng trnh x
3
+y
3
6xy+8 =
0.
Bi 7. Tm nghim nguyn ca h phng trnh
_
xy +yz +zx = 12
x
4
+y
4
+z
4
= 48
.
Bi 8. Cho phng trnh x
3
+y
3
+z
3
= nxyz.
a, Chng minh rng khi m = 1 v m = 2 th phng trnh
khng c nghim nguyn dng.
b, Gii phng trnh nghim nguyn dng khi m = 3.
Bi 9. Gii phng trnh nghim nguyn dng (x
3
+y
3
)+4(x
2
+y
2
)+
4(x +y) = 16xy.
Bi 10. Gii phng trnh nghim nguyn dng
3(x
4
+y
4
+x
2
+y
2
+ 2) = 2(x
2
x + 1)(y
2
y + 1)
Bi 11. Gii phng trnh nghim nguyn dng vi x, y, z l cc s i
mt khc nhau
x
3
+y
3
+z
3
= (x +y +z)
2
Chuyn S hc Din n Ton hc
80 4.2. S dng bt ng thc
4.2.3 Ch ra nghim
Nhn xt. Phng php ny dnh cho nhng bi ton gii phng
trnh nghim nguyn khi m ta tm c chnh xc nghim nguyn
v mun chng minh phng trnh ch c duy nht cc nghim nguyn
m thi.
V d 4.21. Tm nghim nguyn dng ca phng trnh
2
x
+ 3
x
= 5
x
(4.26)
Li gii. Chia 2 v ca phng trnh (4.26) cho s dng 5
x
, ta c:
(4.26)
_
2
5
_
x
+
_
3
5
_
x
= 1
Vi x = 1 th ta c
2
5
+
3
5
= 1:ng nn x = 1 l 1 nghim ca
(4.26).
Vi x > 1 th
_
2
5
_
x
+
_
3
5
_
x
>
2
5
+
3
5
= 1
Do mi gi tr x > 1 u khng l nghim ca (4.26). Vy nghim
nguyn dng ca (4.26) l x = 1.
Nhn xt. v d trn, ta d nhn thy x = 1 l nghim duy nht
ca phng trnh nn ch cn chng minh vi x > 1 th phng trnh
v nghim. Ngoi ra, t bi ton trn ta c th m rng thnh hai bi
ton mi:
Bi ton 4.1. Tm nghim nguyn dng ca phng trnh
(

3)
x
+ (

4)
x
= (

5)
x
Bng cch gii tng t ta cng tm c nghim duy nht ca phng
trnh trn l x = 4.
Din n Ton hc Chuyn S hc
4.2. S dng bt ng thc 81
Bi ton 4.2. Tm nghim nguyn dng ca phng trnh
3
x
+ 4
y
= 5
z
Bi ton 4.2 r rng c nng cao ln r rt, nhng li gii ca bi
ton ny l s dng phng php xt s d hc. Sau y l li gii
rt p ca khanh3570883 hin l iu hnh vin THPT ca VMF:
Li gii. Xt theo module 3 ta c:
5
z
(1)
z
(mod 3) 4
y
(1)
z
(mod 3) z = 2h(h N)
(5
h
2
y
)(5
h
+ 2
y
) = 3
x
Do 5
h
2
y
v 5
h
+2
y
khng ng thi chia ht cho 3 nn 5
h
+2
y
= 3
x
v 5
h
2
y
= 1.
Ta c 5
h
+2
y
(1)
h
+(1)
y
= 0 (mod 3) v 5
h
2
y
(1)
h
(1)
y
=
1 (mod 3) h l v y chn.
Nu y > 2 th 5
h
+ 2
y
1 (mod 4) 3
x
1 (mod 4) 3
x
1
(mod 8).
Mt khc 5 5
h
+ 2
y
(mod 8) 5 3
x
(mod 8) 5 1 (mod 8):
v l.
Do y = 2. Suy ra x = y = z = 2.
Phng php ny thng hay s dng cho cc phng trnh c n s
m v cc phng trnh c nghim nh.
4.2.4 S dng ca phng trnh bc 2
Nhn xt. Vit phng trnh di dng phng trnh bc hai i vi
mt n, dng iu kin. 0 hoc l s chnh phng. Ta s ty
trng hp chn mt trong hai cch xt vo vic gii ton.
V d 4.22. Gii phng trnh nghim nguyn
3x
2
+ (3y 1)x + 3y
2
8y = 0 (4.27)
Chuyn S hc Din n Ton hc
82 4.2. S dng bt ng thc
Li gii. Coi (4.27) l phng trnh bc 2 n x. Xt
x
= 27y
2
+
9y + 1.
(4.27) c nghim x th

x
0 27y
2
+ 9y + 1 0 0, 01 y 3, 3 y 0; 1; 2; 3
Nu y = 0 3x
2
x = 0 x = 0 v x Z.
Nu y = 1 3x
2
+ 2x 5 = 0 x = 1 v x Z.
Nu y = 2 hoc y = 3 th khng tm c x nguyn nn loi.
Vy (4.27) c nghim nguyn (x; y) = (0; 0); (1; 1).
V d 4.23. Gii phng trnh nghim nguyn
3x
2
y
2
2xy 2x 2y + 8 = 0 (4.28)
Li gii. Ta c
(4.28) y
2
+ 2(x + 1)y (3x
2
2x + 8) = 0

y
= (x + 1)
2
+ 3x
2
2x + 8 = 4x
2
+ 9
(4.28) c nghim th

y
= 4x
2
+9 l s chnh phng. t 4x
2
+9 =
k
2
vi k N, ta a v phng trnh c s v tm c x 2; 0; 2.
Vi x = 2 ta c y
2
+ 6y 16 = 0 nn y 8; 2.
Vi x = 0 th y
2
+ 2y 8 = 0 nn y 4; 2.
Vi x = 2 th y
2
2y 24 = 0 nn y 6; 4.
Kt lun. Vy phng trnh (4.28) c nghim (x; y) l (2; 8), (2; 2),
(0; 4), (0; 2), (2; 6), (2; 4).
Nhn xt. Hai bi ton trn u c th s dng phng php a v
phng trnh c s gii.
Bi tp ngh
Bi 1. Tm cc phng php trc (nht l phng php a v
phng trnh c s) cc bi ton gii bng phng php
ny.
Din n Ton hc Chuyn S hc
4.2. S dng bt ng thc 83
Bi 2. Tm nghim nguyn ca phng trnh x +xy +y = x
2
+y
2
.
Bi 3. Gii phng trnh nghim nguyn 10x
2
+ 5y
2
+ 38 12xy +
16y 36x = 0.
Bi 4. Tm nghim nguyn phng trnh 9x
2
+x
2
+4y
2
+34 12xy +
20y 36x = 0.
Bi 5. Tm nghim nguyn dng ca x + 2y
2
+ 3xy + 3x + 5y = 14.
Bi 6. Tm nghim nguyn phng trnh x
2
xy6y
2
+2x6y10 = 0.
Bi 7. Tm nghim nguyn ca phng trnh x
2
+2y62+3xy+3x+5y =
15.
Bi 8. Tm nghim nguyn ca phng trnh 2x
2
+6y
2
+7xyxy =
25.
Bi 9. Tm nghim nguyn ca phng trnh 9x
2
10y
2
9xy +3x
5y = 9.
Bi 10. Tm nghim nguyn ca phng trnh 12x
2
+6xy+3y
2
= 28(x+
y).
(Thi vo lp 10 chuyn, HKHTN-HQGHN nm 1994)
Bi 11. Tm nghim nguyn ca phng trnh 3(x
2
+xy +y
2
) = x+8y.
Bi 12. Tm nghim nguyn ca phng trnh 7(x
2
+xy +y
2
) = 39(x+
y).
Bi 13. Tm nghim nguyn ca phng trnh 2x
2
+ y
2
+ 3xy + 3x +
2y + 2 = 0.
Bi 14. Tm nghim nguyn ca phng trnh x
2
+2y
2
+3xyxy+3 =
0.
Bi 15. Tm nghim nguyn ca phng trnh 3x
2
+4y
2
+12x+3y+5 =
0.
Chuyn S hc Din n Ton hc
84 4.2. S dng bt ng thc
4.2.5 Phng php kp
Nhn xt. S dng tnh cht ly tha cng bc ca s nguyn lin
tip hoc tch cc s nguyn lin tip ... a phng trnh nghim
nguyn cn gii v dng phng trnh khc t n hn v quen thuc
hn. Phng php ny cn c cch gi khc l phng php kh n.
Ta thng vn dng cc nhn xt sau:
1. X
n
Y
n
(X + a)
n
(a N

) th Y
n
= (X + a i)
n
vi
i = 0; 1; 2; ; a.
V d vi n = 2 th:
Khng tn ti x Z a
2
< x
2
< (a + 1)
2
vi a Z.
Nu a
2
< x
2
< (a + 2)
2
th x
2
= (a + 1)
2
2. X(X +1) (X +n) Y (Y +1) (Y +n) (X +a)(X +a +
1) (X + a + n) th Y (Y + 1) (Y + n) = (X + i)(X + 1 +
i) (X +a +i) vi i = 0; 1; 2; ; a.
V d:
Khng tn ti b Z a(a + 1) < b(b + 1) < (a + 1)(a + 2)
vi a Z.
Vi a(a + 1) < b(b + 1) < (b + 2)(b + 3) th b(b + 1) =
(b + 2)(b + 3).
V d 4.24. Tm cc s nguyn dng x biu thc sau l s chnh
phng
A = x
4
+ 2x
3
+ 2x
2
+x + 3 (4.29)
Li gii. V A l s chnh phng nn ta c th t
A = x
4
+ 2x
3
+ 2x
2
+x + 3 = y
2
(y N)
Ta thy
y
2
= (x
4
+ 2x
3
+x
2
) +x
2
+x + 3
= (x
2
+x)
2
+
_
x +
1
2
_
2
+
11
4
> (x
2
+x)
2
y
2
> (x
2
+x)
2
, (i)
Din n Ton hc Chuyn S hc
4.2. S dng bt ng thc 85
Nu x = 1 A = 9: l s chnh phng nn tha .
Nu x > 1 th xt hiu
(x
2
+x+1)
2
y
2
= x
2
+x2 = (x+2)(x1) > 0 y
2
< (x
2
+x+1)
2
, (ii)
T (i) v (ii), ta c
(x
2
+x)
2
< y
2
< (x
2
+x + 1)
2
Suy ra, khng tn ti y N y
2
= A khi x > 1.
Vy x = 1 l gi tr cn tm.
V d 4.25. Gii phng trnh nghim nguyn
x
4
+x
2
+ 4 = y
2
y (4.30)
Li gii. Ta c nh gi sau
x
2
(x
2
+ 1) < x
4
+x
2
+ 4 < (x
2
+ 2)(x
2
+ 3) (4.31)
T (4.30) v (4.31) suy ra
x
2
(x
2
+ 1) < y(y 1) < (x
2
+ 2)(x
2
+ 3). (4.32)
V x, y, z nguyn nn t (4.32) suy ra
y(y 1) = (x
2
+ 1)(x
2
+ 2) (4.33)
T (4.30) v (4.33) th
x
4
+x
2
+ 4 = (x
2
+ 1)(x
2
+ 2) x
2
= 1 x = 1
T y d tm c y = 1 hoc y = 3.
Vy pt cho c bn cp nghim
(x, y) = (1, 2), (1, 3), (1, 2), (1, 3)
Bi tp ngh
Tm nghim nguyn ca cc phng trnh sau:
Chuyn S hc Din n Ton hc
86 4.3. Nguyn tc cc hn, li v hn
Bi 1. x
4
+x
2
+ 1 = y
2
Bi 2. 3(x
4
+y
4
+x
2
+y
2
+ 2) = 2(x
2
x + 1)(y
2
y + 1)
Bi 3. 2x
4
+ 3x
2
+ 1 y
2
= 0
Bi 4. x
2
+ (x +y)
2
= (x + 9)
2
Bi 5. y
3
x
3
= 2x + 1
Bi 6. x
4
y
4
+z
4
+ 2x
2
z
2
+ 3x
2
+ 4z
2
+ 1 = 0
Bi 7. x
3
y
3
2y
2
3y 1 = 0
Bi 8. x
4
+ (x + 1)
4
= y
2
+ (y + 1)
2
Bi 9. 9
x
3
x
= y
4
+ 2y
3
+y
2
+ 2y
Bi 10. x
4
+x
2
y
2
+y + 10 = 0
Bi 11. x
6
4y
3
4y
4
= 2 + 3y + 6y
2
Bi 12. (x 2)
4
x
4
= y
3
Bi 13. x
3
+ 8x
2
6x + 8 = y
3
4.3 Nguyn tc cc hn, li v hn
4.3.1 Li v hn
V d 4.26 (Korea 1996). Gii phng trnh nghim nguyn sau:
x
2
+y
2
+z
2
= 2xyz (4.34)
Li gii. Gi s (x
0
; y
0
; z
0
) l b nghim nguyn ca (4.34) th ta c
x
2
0
+y
2
0
+z
2
0
= 2x
0
y
0
z
0
R rng VT (4.34) chn do VP (4.34) chn nn c 2 trng hp xy
ra:
Din n Ton hc Chuyn S hc
4.3. Nguyn tc cc hn, li v hn 87
Trng hp 1. Trong x
0
; y
0
; z
0
, c 2 s l, 1 s chn. Khng mt tnh
tng qut, gi s x
0
; y
0
l cn z
0
chn. Xt theo module 4 th
V T(4.34) 2 (mod 4), V P(4.34) 0 (mod 4) : v l!
Vy trng hp ny khng xy ra.
Trng hp 2. x
0
; y
0
; z
0
u chn. t x
0
= 2x
1
; y
0
= 2y
1
; z
0
= 2z
1
vi x
1
; y
1
; z
1
Z. Thay vo (4.34) v rt gn, ta thu c
x
2
1
+y
2
1
+z
2
1
= 4x
1
y
1
z
1
Lp lun nh trn, ta li c x
1
; y
1
; z
1
u chn.
Qu trnh din ra tip tc nn x
0
; y
0
; z
0
.
.
.2
k
vi k t nhin ty .
iu ch xy ra khi v ch khi x
0
= y
0
= z
0
= 0.
4.3.2 Nguyn tc cc hn
nh ngha 4.1 Nguyn tc cc hn hay cn gi l nguyn l khi u
cc tr. V mt hnh thc th phng php ny khc vi phng php
li v hn nhng cch s dng u nh nhau u chng minh phng
trnh ch c nghim tm thng (nghim tm thng l nghim bng
0). Phng php gii nh sau:
Gi s (x
0
; y
0
; z
0
; ...) l nghim ca f(x; y; z; ...) vi mt iu kin no
rng buc b (x
0
; y
0
; z
0
; ...). Chng hn x
0
nh nht hoc x
0
+y
0
+
z
0
+ ... nh nht v sau bng cc php bin i s hc ta li tm
c 1 b nghim (x
1
; y
1
; z
1
; ...) tri vi nhng iu kin rng buc
trn. V d ta chn b (x
0
; y
0
; z
0
; ...) vi iu kin x
0
nh nht sau
ta li tm c 1 b (x
1
; y
1
; z
1
; ...) vi x
1
< x
0
dn n phng trnh
c nghim tm thng.
V d 4.27. Gii phng trnh nghim nguyn sau
8x
4
+ 4y
4
+ 2z
4
= t
4
(4.35)
Li gii. Gi s (x
0
; y
0
; z
0
; t
0
) l nghim nguyn khng tm thng ca
(4.35) vi x
0
nh nht.
Chuyn S hc Din n Ton hc
88 4.3. Nguyn tc cc hn, li v hn
T (4.35) suy ra t
0
chn. t t = 2t
1
(t
1
Z) th vo (4.35) v rt gn,
ta c
4x
4
o
+ 2y
4
o
+z
4
o
= 8t
4
1
Do vy z
0
chn. t z
0
= 2z
1
(z
1
Z), th vo v rt gn ta c
2x
4
o
+y
4
o
+ 8z
4
1
= 4t
4
1
Do vy y
0
chn. t y
0
= 2y
1
(y
1
Z), th vo v rt gn ta c
x
4
o
+ 8y
4
1
+ 4z
4
1
= 2t
4
1
Do vy x
0
chn. t x
0
= 2x
1
(x
1
Z), th vo phng trnh ta c
8x
4
1
+ 4y
4
1
+ 2z
4
1
= t
4
1
Suy ra (x
1
; y
1
; z
1
; t
1
) cng l nghim ca (4.35) . D thy x
1
< x
0
(v
l vi iu gi s). Do phng trnh c nghim nguyn duy nht l
(x; y; z; t) = (0; 0; 0; 0).
Bi tp ngh
Bi 1. Gii cc phng trnh nghim nguyn x
2
+y
2
= 3z
2
Bi 2. Gii cc phng trnh nghim nguyn x
3
+ 2y
3
= 4z
3
Bi 3. Gii cc phng trnh nghim nguyn 3x
2
+ 6y
2
+ 12z
2
= t
2
Bi 4. Gii cc phng trnh nghim nguyn x
2
+ 6y
2
+ 2z
2
= 4t
2
Bi 5. Gii phng trnh nghim nguyn x
2
+y
2
+z
2
+t
2
= x
2
y
2
z
2
.
Bi 6. Gii phng trnh nghim nguyn 5x
3
+ 11y
3
+ 13z
3
= 0.
Din n Ton hc Chuyn S hc
Chng
5
Phng trnh ng d
5.1 Phng trnh ng d tuyn tnh 89
5.2 Phng trnh ng d bc cao 90
5.3 H phng trnh ng d bc nht
mt n 90
5.4 Bc ca phng trnh ng d 95
5.5 Bi tp 95
5.6 ng dng nh l Euler gii
phng trnh ng d 96
5.7 Bi tp 101
Trn Trung Kin (Ispectorgadget)
Nguyn nh Tng (tungc3sp)
5.1 Phng trnh ng d tuyn tnh
nh ngha 5.1 Phng trnh ng d dng ax b (mod m) c gi
l phng trnh ng d tuyn tnh vi a, b, m l cc s bit.
x
0
l mt nghim ca phng trnh khi v ch khi ax
0
b (mod m).
Nu x
0
l mt nghim ca phng trnh th cc phn t thuc lp x
0
cng l nghim.
V d 5.1. Gii phng trnh ng d sau: 12x 7 (mod 23)
Li gii. Do (12; 23) = 1 nn phng trnh lun c nghim duy nht.
Ta tm mt s nguyn sao cho 7 + 23k chia ht cho 12. Chn k = 7
suy ra 12x 7.24 (mod 23) x 14 (mod 23)
89
90 5.2. Phng trnh ng d bc cao
V d 5.2. Gii phng trnh 5x 2 (mod 7)
Li gii. V (5; 2) = 1 nn tn ti s k = 4 sao cho 2 +7k chia ht cho
5. Khi y 5x 2 + 6.7 (mod 7) ta c nghim x
30
5
6 (mod 7)
hay x = 6 + 7k
V d 5.3. Gii phng trnh: 5x 4 (mod 11)
Li gii. Ta c:
_
5x 4 (mod 11)
4 4 (mod 11)
p dng tnh cht bc cu ta c: 5x 4 (mod 11) 5x = 11t + 4
Ta c th ly t = 1; x = 3. T phng trnh c nghim duy nht l
x 3 (mod 11)
Nhn xt. Cch xc nh nghim ny l n gin nhng ch dng c
trong trng hp a l mt s nh hoc d thy ngay s k.
5.2 Phng trnh ng d bc cao
V d 5.4. Gii phng trnh 2x
3
+ 4 0 (mod 5)
Li gii. Ta thy x = 2 suy ra 2x
3
4 (mod 5).
Nn x = 2 l nghim duy nht ca phng trnh cho.
5.3 H phng trnh ng d bc nht mt n
nh ngha 5.2 H phng trnh c dng sau c gi l h phng
trnh ng d bc nht mt n
_

_
x b
1
(mod m
1
)
x b
2
(mod m
2
)
....
x b
k
(mod m
k
)
Vi m
1
; m
2
; ...m
k
l nhng s nguyn ln hn 1 v b
1
; b
2
; ...; b
k
l nhng
s nguyn ty .
Din n Ton hc Chuyn S hc
5.3. H phng trnh ng d bc nht mt n 91
Nhn xt. Trong trng hp tng qut, chng ta c th chng
minh c rng: iu kin cn v h phng trnh (5.2) c
nghim l UCLN(m
i
; m
j
) chia ht b
i
b
j
vi i ,= j(1 i, j k).
Gi s m = p
1
1
p
a
2
2
...p
k
k
l phn tch tiu chun ca m. Khi
y phng trnh ng d f(x) 0 (mod m) tng ng vi h
phng trnh ng d f(x) 0 (mod p
1
i
), i = 1, 2, ..., k. T
suy ra rng nu x b
1
(mod p
1
1
) l mt nghim ca phng
trnh f(x) 0 (mod p
i
), i = 1, 2, ..., k th nghim ca h phng
trnh ca h phng trnh ng d
_

_
x b
1
(modp

1
1
)
x b
2
(modp

2
2
)
...
x b
k
_
modp

k
k
_
cho ta nghim ca phng trnh f(x) 0(modm).
Vy trong Trng hp tng qut gii mt phng trnh ng
d dn n gii h trn. Vi cc module m
1
, m
2
, ..., m
k
i mt
nguyn t cng nhau.
Phng php chung gii:
Trng hp 1: h 2 phng trnh
_
x b
1
(mod m
1
)
x b
2
(mod m
2
)
Vi gi thit d = (m
1
, m
2
) chia ht cho b
1
b
2
. Trc tin ta nhn
xt rng, mi s x = b
1
+m
1
t, t Z l nghim ca phng trnh
th nht. Sau ta tm cch xc nh t sao cho x nghim ng
phng trnh th hai, ngha l h hai phng trnh trn tng
ng vi h phng trnh
_
x = b
1
+m
1
t
b
1
+m
1
t b
2
(mod m
2
)
Chuyn S hc Din n Ton hc
92 5.3. H phng trnh ng d bc nht mt n
V gi thit d = (m
1
, m
2
) l c b
1
b
2
nn phng trnh: b
1
+
m
1
t b
2
(mod m
2
) tng ng vi phng trnh:
m
1
d
t
b
2
b
1
d
(mod
m
2
d
)
Nhng (
m
1
d
,
m
2
d
) = 1 nn phng trnh ng d ny cho ta
nghim t t
0
(mod
m
2
d
), l tp hp tt c cc s nguyn
t = t
0
+
m
2
d
u, u Z
Thay biu thc ca t vo biu thc tnh x ta c tp hp cc
gi tr ca x nghim ng c hai phng trnh ng d ang xt
l:
x = b
1
+ m
1
(t
0
+
m
2
d
u) = b
1
+ m
1
t
0
+
m
1
m
2
d
u, hay x = x
0
+ m
u
vi x
0
= b
1
+m
1
t
0
, m = BCNN(m
1
, m
2
).
Vy x x
0
(mod m) l nghim ca h hai phng trnh ng d
ang xt.
Trng hp 2: H gm n phng trnh. u tin gii h hai
phng trnh no ca h cho, ri thay trong h hai phng
trnh gii bng nghim tm thy, ta s c mt h gm n1
phng trnh tng ng vi vi h cho. Tip tc nh vy
sau n 1 bc ta s c nghim cn tm.
V d 5.5. Gii h phng trnh:
_

_
x 26 (mod 36)
x 62 (mod 60)
x 92 (mod 150)
x 11 (mod 231)

Li gii. H hai phng trnh:


_
x 26 (mod 36)
x 62 (mod 60)

_
x = 26 + 36t
26 + 36t 62
, t Z.
26 + 36t 62 (mod 60)
36t 36 (mod 60)
t 1 (mod 5)
Din n Ton hc Chuyn S hc
5.3. H phng trnh ng d bc nht mt n 93
Vy nghim ca h l: x 26 +36.1 (mod 180) hay x 62 (mod 180)
Do h phng trnh cho tng ng vi h:
_
_
_
x 62 (mod 180)
x 92 (mod 150)
x 11 (mod 231)
V d 5.6. Gii h phng trnh
_
x 62 (mod 180)
x 92 (mod 150)

_
x = 62 + 180t
62 + 180t 92 (mod 150)
, t Z.
Li gii. Ta c:
62 + 180t 92 (mod 1)50)
180t 30 (mod 150)
6t 1 (mod 5) t 1 (mod 5)
Vy nghim ca h l:
x 62 + 180.(1+) (mod 900) x 242 (mod 900)
H cho tng ng vi:
_
x 242 (mod900)
x 11 (mod231)
H ny c nghim x 242 (mod 69300) , v y cng l nghim ca
h cho cn tm.
V d 5.7. Tm s nguyn dng nh nht tha tnh cht: chia 7 d 5,
chia 11 d 7 v chia 13 d 3.
Li gii. Ta c: n
1
= 7; N
1
= 11.13 = 143; n
2
= 11; N
2
= 7.13 =
91; n
3
= 13; N
3
= 7.11 = 77.
Ta c N
1
b
1
3b
1
1 (mod 7) b
1
= 2. Tng t b
2
= 4; b
3
= 1
Vy a = 143(2)5 +(91)(4)(7) +(77)(1)(3) = 1430 +2548 231 =
887 vy cc s cn tm c dng b = 877 + 1001k.
Vy 877 l s cn tm.
Chuyn S hc Din n Ton hc
94 5.3. H phng trnh ng d bc nht mt n
V d 5.8 (Chn i tuyn KHTN). Xt h ng d gm 3 phng
trnh:
xy 1 (mod z) (5.1)
yz 1 (mod x) (5.2)
xz 1 (mod y) (5.3)
Hy tm s b (x, y, z)nguyn dng phn bit vi1 trong 3 s l 19.
Li gii. T ba phng trnh, theo tnh cht ng d ta ln lt c
xy + 1
.
.
.z v yz 1
.
.
.x v zx 1
.
.
.y
Suy ra
(xy + 1)(yz 1)(zx 1)
.
.
.xyz
x
2
y
2
z
2
x
2
yz xy
2
z +xyz
2
+xy yz zx + 1
.
.
.xyz
xy yz zx + 1
.
.
.xyz
Nhn thy do x, y, z nguyn dng cho nn xyz 1. Suy ra xy yz
zx + 1 2xyz
Mt khc yz +zx xy 1 2xyz (yz +zx xy 1) 2xyz
Do ta c bt phng trnh kp 2xyz xy yz zx + 1 2xyz
M xyyzzx+1
.
.
.xyz xyyzzx+1 = 2xyz, 1xyz, 0, 1xyz, 2xyz
Trng hp 1: xy yz zx + 1 = 2xyz xy 1 (mod z), yz 1
(mod x), zx 1 (mod y)
Cho nn ta ch cn tm nghim ca xy yz zx + 1 = 2xyz l xong.
V x, y, z c mt s bng 19 nn ta thay ln lt vo.
Nu x = 19 19y yz 19z + 1 = 38yz 39yz 19y + 19z = 1
(39y + 19)(39z 19) = 322 Vi y = 19 hoc z = 19 th tng t.
Trng hp 2,3,4,5: xyyzzx+1 = 1xyz, 0, 1xyz, 2xyz lm hon
ton tng t, ta y c v phng trnh c dng au+bv = ab+uv+x
vi x l hng s.
a v (a v)(b u) = x v gii kiu phng trnh c s. Bi ton
hon tt.
Din n Ton hc Chuyn S hc
5.4. Bc ca phng trnh ng d 95
Nhn xt. Bi ton ny m khng cho iu kin mt s bng 19 th
khng a c dng au +bv = ab +uv +x (a v)(b u) = x lc
suy ra v hn nghim.
5.4 Bc ca phng trnh ng d
nh ngha 5.3 Xt phng trnh ng d f(x) = 0 (mod m) vi
f(x) = a
0
x
n
+a
1
x
n1
+... +a
n
, a
i
N, i = 0, 1, ..., n
Nu a
0
khng ng d 0 (mod m) th ta ni n l bc ca phng trnh
ng d.
V d 5.9. Xc nh bc ca phng trnh 15x
6
8x
4
+x
2
+6x+8 0
(mod 3)
Li gii. Ta thy 15 0 (mod 3) nn bc ca phng trnh khng
phi l bc 6. Phng trnh trn tng ng vi 8x
4
+ x
2
+ 2 0
(mod 3)
V 8 , 0 (mod 3) nn bc phng trnh l n = 4.
5.5 Bi tp
Bi 1. Gii cc phng trnh sau: a) 7x 6 (mod 13) b) (a + b)x
a
2
+ b
2
(mod ab) vi (a, b) = 1 c) 17x 13 (mod 11) d) x
2
+
x 2 1 (mod 3)
Bi 2. Gii cc h phng trnh: a)
_

_
x 1 (mod 3)
x 4 (mod 4)
x 2 (mod 7)
x 9 (mod 11)
b)
_
_
_
5x 1 (mod 12)
5x 2 (mod 8)
7x 3 (mod 11)
Bi 3. Tm a nguyn h phng trnh sau c nghim
Chuyn S hc Din n Ton hc
96 5.6. ng dng nh l Euler gii phng trnh ng d
a)
_

_
x 3 (mod 3)
x 1 (mod 4)
x 11 (mod 7)
x a (mod 11)
b)
_
2x a (mod 3)
3x 4 (mod 10)
Bi 4. Mt lp gm 40 hc sinh ng thnh vng trn v quay mt
v trong vng trn chi bng. Mi hc sinh nhn c bng
phi nm qua mt 6 bn bn tay tri mnh. Chng minh rng
tt c hc sinh trong lp u nhn c bng nm ti mnh
sau 40 ln nm bng lin tip.
5.6 ng dng nh l Euler gii phng trnh
ng d
Qua bi vit ny ti xin gii thiu mt phng php gii phng
trnh ng d bng cch khai thc nh l Euler
Trc ht, xin nhc li vi kin thc quen thuc.
nh ngha 5.4 Hm Euler (m) vi s nguyn dng m l cc s t
nhin nh hn m l cc s nguyn t vi m.
5.6.1 nh l Euler.
nh l 5.1 (Euler) Cho m l s nguyn dng v (a, m) = 1 th
a
(m)
1 (mod m)
Hm c tnh cht sau:
(mn) = (m)(n) vi (m; n) = 1
Nu p nguyn t (p) = p 1; (p
n
) = p
n
p
n1
(n > 1)
Din n Ton hc Chuyn S hc
5.6. ng dng nh l Euler gii phng trnh ng d 97
Nu m = p

1
1
p

2
2
...p

k
k
, p
i
l cc s nguyn t th
(m) = m
_
1
1
p
1
__
1
1
p
2
_
...
_
1
1
p
k
_
By gi ta xt m = a.b trong (a; b) = 1 th c cc kt qu sau
nh l 5.2
a
(b)
+b
(a)
1 (mod ab) (5.4)
Chng minh. Theo nh l Euler ta c: a
(b)
1 (mod b) m b
(a)
0
(mod b)
Nn a
(b)
+b
(a)
1 (mod b).
Tng t ta c:a
(b)
+b
(a)
1 (mod a)
Theo tnh cht ng d th : a
(b)
+b
(a)
1 (mod ab)
nh l 5.3 Gi s c k(k 2) s nguyn dng m
1
; m
2
; . . . m
k
v
chng nguyn t vi nhau tng i mt. t M = m
1
.m
2
. . . m
k
= m
i
t
i
vi i = 1, 2, 3. . . , k ta c
t
(m
1
)
1
+t
(m
2
)
2
+... +t
(m
k
)
k
1 (mod M) (5.5)
Chng minh. T gi thit ta c (m
i
, t
i
) = 1 vi mi i = 1, 2, . . . , k nn
theo nh l Euler th
t
(m
1
)
1
1 (mod m
i
) (5.6)
Mt khc vi i; j thuc tp 1;2;. . . ;k v i ,= j th t
j
chia ht cho m
j
nn (t
j
; m
i
) = m
i
hay
t
(m
i
)
j
0 (mod m
i
) (5.7)
t S = t
(m
1
)
1
+t
(m
2
)
2
+... +t
(m
k
)
k
T (5.6) v (5.7) c S t
m
i
i
1 (mod m
i
)
V m
1
; m
2
; . . . m
k
nguyn t vi nhau tng i mt, nn theo tnh cht
ng d thc c
S 1 0 (mod m
1
.m
2
...m
k
) S 1 (mod M), tc l c (5.5).
Chuyn S hc Din n Ton hc
98 5.6. ng dng nh l Euler gii phng trnh ng d
Khi m rng (5.4) theo hng nng ln ly tha cc s hng ta c kt
qu sau.
nh l 5.4 Vi (a, b) = 1 v n, v l hai s nguyn dng no th
a
n(b)
+b
v(a)
1 (mod ab) (5.8)
Chng minh. tin lp lun t x = a
(b)
.
Theo nh l Euler th x = a
(b)
1 (mod b) x 1 0 (mod b)
ng thi x = a
(b)
0 (mod a).
T c x(x1) 0 (mod a) v x(x1) 0 (mod b) nn x(x1) 0
(mod ab)
T x
3
x
2
.x x.x x
2
x (mod ab) v c lp lun nh th c
x
n
x (mod ab) hay a
n(b)
a
(b)
(mod ab)
Tng t ta c: b
v(a)
b
(a)
(mod ab) nn theo (5.4) c a
n(b)
+
b
v(a)
b
(a)
+a
(b)
1 (mod ab).
(5.8) c chng minh.
H qu 5.1 Vi (a; b) = 1 th a
n(b)
+b
n(a)
1 (mod ab)
H qu ny c th chng minh trc tip khi nng hai v ca h thc
(5.4) ln ly tha bc n (s dng khi trin nh thc Newton) v ch
rng ab 0 (mod ab). Nn lu rng trong ng d thc th a , 0
(mod ab)!
Vi k hiu nh nh l 5.3 ta c t
i
.t
j
0 (mod M) vi i khc j v mi
i; j thuc tp 1,2,...,k (nhng t , 0 (mod M) vi mi i = 1, 2, 3, ...k)
T khi nng hai v ca (5.5) ln ly tha bc n ta c kt qu sau.
nh l 5.5 Vi cc gi thit nh nh l 5.3 ta c:
t
n(m
1
)
1
+t
n(m
2
)
2
+... +t
n(m
k
)
k
1 (mod M) (5.9)
Vi cc k hiu nh trn ta t a = m
i
v b = t
i
th theo (5.4) c
m
n(t
i
)
i
+t
n(m
i
)
i
1 (mod M) (5.10)
Cng tng v ca k ng thc dng (5.10) v s dng (5.5) ta c
kt qu sau:
Din n Ton hc Chuyn S hc
5.6. ng dng nh l Euler gii phng trnh ng d 99
nh l 5.6 Vi cc gi thit nh l 5.3 ta c:
m
(t
1
)
1
+m
(t
2
)
2
+... +m
n(t
k
)
k
k 1 (mod M) (5.11)
Khi nhn 2 v ca (??) vi m
i
ta c
m
1+(t
i
)
1
+m
i
.t
(m
i
)
i
+ m
i
(mod M) (5.12)
Do m
i
.t
(m
i
)
i
= m
i
.t
i
.t
(m
i
)1
i
= M.t
(m
i
)1
i
nn
m
1+(t
1
)
i
m
i
(mod M), i = 1, k (5.13)
Cng tng v k ng thc dng (5.13) ta c kt qu sau:
nh l 5.7 Vi cc gi thit nh nh l 5.3 ta c:
m
1+(t
1
)
1
+m
2+(t
2
)
2
+... +m
1+(t
k
)
k
m
1
+m
2
+... +m
k
(mod M)
(5.14)
Khi nhn 2 v ca (5.10) vi t
i
ta c
m
1+(t
1
)
1
+m
2+(t
2
)
2
+... +m
1+(t
k
)
k
m
1
+m
2
+... +m
k
(mod M)
(5.15)
t
1+(m
i
)
i
t
i
(mod M), i = 1, k (5.16)
Cng tng v ca k ng d dng (5.16) ta c kt qu sau
nh l 5.8 Vi cc gi thit nh nh l 5.3 ta c:
t
1+(m
1
)
1
+t
1+(m
2
)
2
+... +t
1+(m
k
)
k
t
1
+t
2
+... +t
k
(mod M) (5.17)
Ch rng t
i
.t
j
0 (mod M) nn khi nng ln ly tha bc n ca
tng t
1
+t
2
+... +t
k
ta c kt qu sau.
nh l 5.9 Vi cc gi thit nh nh l 5.3 ta c:
t
n
1
+t
n
2
+... +t
n
k
(t
1
+t
2
+... +t
k
)
n
(mod M) (5.18)
Chuyn S hc Din n Ton hc
100 5.6. ng dng nh l Euler gii phng trnh ng d
Kh nng tm ra cc h thc ng d mi cha phi ht mi bn
c nghin cu thm. nm r c nhng phn trn ta tm hiu
qua mt s v d sau y.
V d 5.10. Tm t nht bn nghim ca phng trnh ng d:
x
3
+y
7
1 (mod 30) (5.19)
Li gii. Do 30 = 5.6 v (6; 5) = 1 nn theo (5.4) c 5
(6)
+ 6
(5)
1
(mod 30)
v (6) = (2).(3) = 2 v (5) = 4; 6
2
6 (mod 30).
Tng t ta c: 25
7
25 (mod 30) v 6
3
6 (mod 30) nn 6
3
+25
7

26 + 6 1 (mod 30)
Nu phn tch 30 = 3.10 vi (3; 10) = 1 th theo (5.4) c 3
(10)
+10
(3)

1 (mod 30). Tnh ton tng t nh trn ta c 3


4
+10
2
1 (mod 30).
V 3
4
= 81 21 (mod 30) v 10
2
10 (mod 30) nn theo (5.8) c
(3
4
)
3
+ (10
2
)
7
1 (mod 30) v (3
4
)
7
+ (10
2
)
3
1 (mod 30)
Suy ra phng trnh trn c t nht bn nghim (x; y) l (25; 6); (6; 25);
(21; 10); (10; 21).
V d 5.11. Chng minh rng phng trnh ng d sau c nghim
(x; y; z; t) khc (0; 0; 0; 0):
x
4
+y
4
+z
4
+t
4
t
3
(mod 60).
Li gii. 60 = 3.4.5 v (5; 3) = 1; (5; 4) = 1; (3; 4) = 1 nn t m
1
=
3; m
2
= 4; m
3
= 5; t
1
= 15; t
2
= 1; t
3
= 20 theo (5.18)
15
4
+ 12
4
+ 20
4
(15 + 20 + 12)
4
1 (mod 60)
V d 5.12. Tm t nht mt nghim ca phng trnh ng d x
17
+
y
19
1 (mod 35)
Li gii. Ta c: 35 = 5.7 m (5; 7) = 1 nn theo (5.4): 5

7
+ 7

5
1
(mod 35))
V (5) = 4; (7) = 6 nn 5
4
+ 7
6
1 (mod 35)
Theo (5.8): 14
17
+ 30
19
14 + 30 1 (mod 35)
Vy phng trnh ng d c t nht mt nghim (x; y) = (14; 30)
Din n Ton hc Chuyn S hc
5.7. Bi tp 101
5.7 Bi tp
Bi 1. Chng minh rng phng trnh ng d sau c nghim (x; y; z; t)
khc (0; 0; 0; 0):
a) x
3
+y
3
+z
3
t
3
(mod 210)
b) x
5
+y
5
+z
5
t
5
(mod 1155)
Bi 2. Tm t nht mt nghim ca phng trnh ng d sau:
x
11
+y
13
1 (mod 45)
Bi 3. Chng t rng mi phng trnh sau c nghim nguyn dng.
a) 2
x
+ 3
y
+ 5
z
+ 7
t
3 (mod 210)
b) 3
x
+ 5
y
+ 7
z
2 (mod 105)
Chuyn S hc Din n Ton hc
Chng
6
H thng d v nh l
Thng d Trung Hoa
6.1 Mt s k hiu s dng trong bi
vit 103
6.2 H thng d 104
6.3 nh l thng d Trung Hoa 117
6.4 Bi tp ngh & gi p s 125
Nguyn nh Tng (tungc3sp)
Bi vit ny trnh by v H thng d v nh l Thng d Trung Hoa.
Mt s k hiu s dng c phc ha trong Phn 6.1. Phn 6.2 gii
thiu n bn c mt s kin thc c bn v H thng d y
v H thng d thu gn km theo bi tp ng dng. nh l Thng
d Trung Hoa km ng dng ca n gip gii quyt mt s dng ton
c trnh by trong Phn 6.3. Phn 6.4 kt thc bi vit bao gm
mt s bi tp ngh km gi hoc p s.
6.1 Mt s k hiu s dng trong bi vit
[x, y] : bi chung nh nht ca hai s nguyn dng x, y (nu
khng ni g thm).
(x, y) : c chung ln nht ca hai s nguyn x, y.
x y (mod p): x khng ng d vi y theo module p.
H: h thng d y .
103
104 6.2. H thng d
HTG: h thng d thu gn.
P: tp cc s nguyn t.
(n): hm le ca n.
[A[: s phn t ca tp A.
x: phn l ca s thc x, c xc nh nh sau: x = x[x],
trong [x] l phn nguyn ca s thc x (l s nguyn ln nht
khng vt qu x).

i=1
p
i
= p
1
p
2
...p
n
6.2 H thng d
6.2.1 Kin thc c bn
H thng d y
nh ngha 6.1 Cho tp A = a
1
; a
2
; ...; a
n
. Gi s r
i
, 0 r
i
n 1
l s d khi chia a
i
cho n. Nu tp s d r
1
; r
2
; ...; r
n
trng vi tp
0; 1; 2; ...; n 1 th ta ni A l mt h thng d y (gi tt l
H) mod n.
Nhn xt. T nh ngha, d thy:
Nu A = a
1
; a
2
; ...; a
n
lp thnh H (mod n) nu v ch nu:
i ,= j a
i
,= a
j
(mod n).
Nu A = a
1
; a
2
; ...; a
n
l H (mod n) th t nh ngha d
dng suy ra:
Vi mi m Z, tn ti duy nht a
i
A sao cho a
i
m
(mod n).
Vi mi a Z, tp a + A = a +a
1
; a +a
2
; ...; a +a
n
l
mt H (mod n).
Din n Ton hc Chuyn S hc
6.2. H thng d 105
Vi mi c Z v (c; n) = 1; tp cA = ca
1
; ca
2
; ...; ca
n
l
mt H (mod n).
Ch : tp A

= 0; 1; 2; 3; ...; n 1 l mt H (mod n) khng m


nh nht. S phn t ca tp A l [A[ = n.
V d 6.1. Cho hai H (mod n): A = a
1
; a
2
; ...; a
n
v
B = b
1
; b
2
; ...; b
n
.
a. Chng minh rng: Nu n chn th tp A + B = a
1
+ b
1
; a
2
+
b
2
; ...; a
n
+b
n
khng hp thnh H (mod n)
b. Kt lun cu a. s th no nu n l s l
Li gii. a. Ta c mt iu kin cn sau y i vi H (mod n),
khi n chn. Gi s C = c
1
; c
2
; ...; c
n
l mt H (mod n). Khi
theo nh ngha ta c:
c
1
+c
2
+... +c
n
(1 + 2 +... + (n 1))
n(n + 1)
2
(mod n)
Do n chn nn n = 2k, suy ra:
n(n + 1)
2
= k(2k + 1) ,
.
.
.n k(2k + 1) 0 (mod n)
c
1
+c
2
+... +c
n
0 (mod n) (6.1)
Ta c:
A+B = a
1
+b
1
; a
2
+b
2
; ...; a
n
+b
n

(a
1
+a
2
+... +a
n
) + (b
1
+b
2
+... +b
n
) (mod n)

_
n(n + 1)
2
+
n(n + 1)
2
_
(mod n)
[n(n + 1)] (mod n)
A+B 0 (mod n) (6.2)
( y ta cng s dng gi thit A v B l hai H mod n).
T (6.1) v (6.2) ta suy ra pcm.
Chuyn S hc Din n Ton hc
106 6.2. H thng d
b. Xt khi n l: Lc ny cha th kt lun g v tnh cht ca h
A+B.
Tht vy, ta xt n = 3; A = 1; 2; 3 ; B = 4; 5; 6.
Khi A+B = 5; 7; 9 l mt H mod 3.
Nhng, xt h A = 1; 2; 3 , B = 5; 4; 6.
Khi A+B = 6; 6; 9 khng phi l mt H mod 3.
H thng d thu gn
nh ngha 6.2 Cho tp B = b
1
; b
2
; ...; b
k
l mt tp hp gm k s
nguyn v (b
i
; n) = 1 vi mi i = 1; 2; ...; k.
Gi s: b
i
= q
i
n +r
i
vi 1 r
i
< n. Khi d thy (r
i
; n) = 1.
Nu tp r
1
; r
2
; ...; r
n
bng tp K gm tt c cc s nguyn dng
nh hn n v nguyn t cng nhau vi n th B c gi l h thng
d thu gn mod n, gi tt l HTG (mod n).
Nhn xt. Ta c th rt ra hai nhn xt:
D thy tp B = b
1
; b
2
; ...; b
k
gm k s nguyn lp thnh mt
HTG khi v ch khi
i. (b
i
; n) = 1
ii. b
i
,= b
j
(mod n) vi 1 i ,= j k
iii. [B[ = (n)
iu kin (iii) tng ng vi (iii

): vi mi x Z; (x; n) = 1
tn ti duy nht b
i
B sao cho x b
i
(mod n).
T nh ngha ta suy ra: cho tp B = b
1
; b
2
; ...; b
k
l HTG mod
n v c Z; (c; n) = 1 th tp cB = cb
1
; cb
2
; ...; cb
n
cng l HTG
mod n.
Din n Ton hc Chuyn S hc
6.2. H thng d 107
V d 6.2. Cho hai s nguyn dng m, n vi (m; n) = 1. Gi s A =
a
1
, a
2
, ..., a
h
; B = b
1
, b
2
, ..., b
k
tng ng l cc h thu gn mod m
v mod n. Xt tp hp C = a
i
n +b
j
m ; 1 i h; 1 j k.. Chng
minh rng C l mt h thu gn HTG mod mn.
Li gii. + Ta chng minh (a
i
n +b
j
m, mn) = 1 i = 1, h; j = 1, k
(iu kin (i)).
Gi s tn ti i, j v s nguyn t p l c chung ca a
i
n +b
j
m
v mn.
Ta c a
i
n +b
j
m
.
.
.p v mn
.
.
.p.
Do mn
.
.
.p m (m, n) = 1 nn c th gi s n
.
.
.p, suy ra
a
i
n
.
.
.p b
j
m
.
.
.p b
j
.
.
.p
Vy p l c nguyn t chung ca n v b
j
. iu ny mu thun
vi gi thit. Nn iu gi s l sai. Vy (a
i
n+b
j
m, mn) = 1 i =
1, h; j = 1, k.
+ Chng minh iu kin (ii).
Gi s tn ti a A; b B sao cho an + bm a

n + b

m
(mod mn)
an a

n (mod m) a a

(mod m) (do (m, n) = 1)


(iu ny mu thun).
Vy an +bm a

n +b

m (mod mn).
+ Chng minh iu kin (iii

).
Gi s (x, mn) = 1 (x, m) = 1; (x, n) = 1.
V (m, n) = 1 nn tp B = mb
1
, mb
2
, ..., mb
k
l mt HTG mod
n.
Vy tn ti duy nht b B x mb (mod n).
Chuyn S hc Din n Ton hc
108 6.2. H thng d
Tng t, tn ti duy nht a A x na (mod m).
T suy ra x na + mb (mod n) v x na + mb (mod m).
T kt hp vi (m, n) = 1 suy ra x na +mb (mod mn).
Nhn xt. T y, ta c th suy ra cng thc tnh hm le (n).
6.2.2 ng dng
Trong cc bi ton v a thc, dy s
V d 6.3. [THTT, s 340] Cho p l s nguyn t l v a thc Q(x) =
(p 1)x
p
x 1. Chng minh rng tn ti v hn s nguyn dng a
sao cho Q(a) chia ht cho p
p
.
Li gii. Thay cho vic chng minh tn ti v hn s nguyn dng a
sao cho Q(a) chia ht cho p
p
, ta s chng minh tp
H = Q(1); Q(2); ...; Q(p
p
)
l mt H mod p
p
.
Ta c nhn xt sau: trong tp s 1; 2; ...; p
p
gm p
p
s, gi s c hai
s u, v khc nhau th Q(u) Q(v) (mod p
p
).
Ta chng minh iu ny bng phn chng. Gi s c Q(u) Q(v)
(mod p
p
)
(p 1)u
p
u 1 (p 1)v
p
v 1 (mod p
p
)
(p 1)(u
p
v
p
) (u v) 0 (mod p) (6.3)
Theo nh l Ferma nh th u
p
u (mod p) v v
p
vp (mod p) vi p
l s nguyn t nn u
p
v
p
u v (mod p).
T (6.3) suy ra
(p 2)(u v) 0 (mod p) u v (mod p) (6.4)
Cng t (6.3) ta c:
(u v)((p 1)(u
p1
+u
p2
v +... +uv
p2
+v
p1
) 1) 0 (mod p
p
)
Din n Ton hc Chuyn S hc
6.2. H thng d 109
Kt hp vi (6.4) suy ra
(u v)((p 1).p.u
p1
1) 0 (mod p
p
) u v 0 (mod p
p
)
iu ny mu thun vi gi s u v (mod p
p
). Vy nhn xt c
chng minh.
T nhn xt trn suy ra H = Q(1); Q(2); ...; Q(p
p
) l mt H
mod p
p
. T suy ra trong tp s 1; 2; ...; p
p
gm p
p
s th tn
ti duy nht mt s a sao cho Q(a) 0 (mod p
p
) hay Q(a)
.
.
.p
p
.
Ta xt dy s hng a
k
= a +k.p
p
vi k = 0, 1, 2..., d thy rng:
Q(a
p
) Q(a) 0 (mod p
p
).
Ngha l tn ti v hn s a
k
(k = 0, 1, 2, ...) tha mn Q(a
k
)
.
.
.p
p
.

V d 6.4. Cho a thc P(x) = x


3
11x
2
87x+m. Chng minh rng
vi mi s nguyn m, tn ti s nguyn n sao cho P(n) chia ht cho
191.
Li gii. tng cng tng t V d 6.3, ta s s dng H. Trc
ht ta a ra b sau:
B 6.1 Cho p l s nguyn t, p 2 (mod 3). Khi ,vi mi s
nguyn x, y m x
3
y
3
(mod p) x y (mod p)
Chng minh. Tht vy:
Nu x 0 (mod p) y
3
0 (mod p) y 0 (mod p) x
y(modp)
Nu x, y cng khng chia ht cho p, do p 2(mod3) p =
3k + 2(k Z).
Chuyn S hc Din n Ton hc
110 6.2. H thng d
Theo nh l Ferma:
x
p1
= x
3k+1
1 (mod p)
y
p1
= y
3k+1
1 (mod p)
x
3k+1
y
3k+1
(mod p) (6.5)
M theo gi thit, x
3
y
3
mod p x
3k
y
3k
(mod p).
T suy ra x y (mod p). Vy b c chng minh.
Tr li bi ton, ta s chng minh P(n
1
) P(n
2
) (mod 191) vi
n
1
; n
2
Z th n
1
n
2
(mod 191).
Tht vy, v
27P(n
1
) = (3n
1
11)
3
11.191.n
1
+ 11
3
+ 27m
27P(n
2
) = (3n
2
11)
3
11.191.n
2
+ 11
3
+ 27m
nn
P(n
1
) P(n
2
) (mod 191)
27P(n
1
) 27P(n
2
) (mod 191)
(3n
1
11)
3
(3n
2
11)
3
(mod 191)
3n
1
11 3n
2
11 (mod 191)(suy ra t b )
n
1
n
2
(mod 191)
Vi mi n
1
, n
2
A = 1; 2; 3; ...; 1991 (A l mt H mod 191),
n
1
,= n
2
ta c P(n
1
) P(n
2
) (mod 191)
A

= P(1); P(2); ...; P(191) l mt H mod 191.


T suy ra n A = 1; 2; 3; ...; 191 sao cho
P(n) 191 (mod 191) P(n)
.
.
.191
.
V d 6.5. Cho p l mt s nguyn t. Chng minh rng vi mi s m
nguyn khng m bt k, lun tn ti mt a thc Q(x) c h s nguyn
sao cho p
m
l c chung ln nht ca cc s a
n
= (p + 1)
n
+Q(n); n =
1, 2, 3...
Din n Ton hc Chuyn S hc
6.2. H thng d 111
Li gii. Ta c b sau:
B 6.2 k N, k < m th tn ti b
k
Z sao cho b
k
p
m
+p
k
.
.
.k!
Chng minh. Gi s k! = p

k
M
k
vi (M
k
; p) = 1.
Khi e chy trong tp 0; 1; ...; M
k
1 th cc s
_
ep
mk
_
lp thnh
mt H modM
k
, thnh th tn ti b
k
Z sao cho b
k
p
mk
1
(mod M
k
)
(b
k
p
mk
+ 1)
.
.
.M
k
(b
k
p
m
+p
k
)
.
.
.p
k
.M
k
Mt khc

i=1
_
k
p
i
_
<

i=1
k
p
i
< k
Vy (b
k
p
m
+p
k
)
.
.
.p

k
.M
k
= k!. B c chng minh.
Tr v bi ton.
t f
i
(x) =
x(x 1)...(x i + 1)
i!
th f
i
(n) =
_
C
i
n
_
nu n i
_
0
_
nu n < i
_
.
t R(x) =
m1

i=0
f
i
(x)(b
i
p
m
+p
i
) th theo B 6.2, R(x) l a thc
c h s nguyn.
Ta c:
u
n
= (p + 1)
n
+R(n) =
n

i=0
C
i
n
p
i

m1

i=1
f
i
(n)p
i
p
m
m1

i=0
f
i
(n)b
i

i=0
f
i
(n)p
i

m1

i=1
f
i
(n)p
i
(mod p
m
)

i=0
f
i
(n)p
i
0 (mod p
m
) n = 1, 2, 3...
c bit u
1
= (p + 1) +R(1) = ep
m
Chuyn S hc Din n Ton hc
112 6.2. H thng d
Ta chng minh a thc Q(x) = R(x)+p
m
(1e) l a thc cn tm.Tht
vy,
a
n
= (p + 1)
n
+Q(n) = (p + 1)
n
+R(n) +p
m
(1 e)
= u
n
+p
m
(1 e)
.
.
.p
m
, n = 1, 2, 3... (6.6)
Mt khc
a
1
= (p + 1) +Q(1) = p + 1 +R(1) +p
m
(1 e) = ep
m
+p
m
(1 e)
.
.
.p
m
Do p
m
l CLN ca a
n
vi mi n = 1, 2, 3...
V d 6.6. Cho p 3 l mt s nguyn t v a
1
, a
2
, ..., a
p2
l mt dy
cc s nguyn dng sao cho p khng l c s ca a
k
v a
k
k
1 vi
mi k = 1, 2, 3, ..., p2. Chng minh rng tn ti mt s phn t trong
dy a
1
, a
2
, ..., a
p2
c tch ng d vi 2 module p.
Li gii. Ta c b sau:
B 6.3 Vi mi s nguyn k = 1, 2, ..., p 1 tn ti mt tp cc s
nguyn b
k,1
, b
k,2
, ..., b
k,k
tha mn hai iu kin sau:
1. Mi b
k,j
hoc bng 1, hoc bng tch ca mt s phn t trong dy
a
1
, a
2
, ..., a
p2
,
2. b
k,i
b
k,j
(mod p) vi 1 i ,= j k.
Chng minh. Vi k=2 chn b
21
= 1; b
22
= a
1
1 (mod p) (do a
1
1
1
khng chia ht cho p).
Gi s vi 2 k p 2 ta chn c tp b
k,1
, b
k,2
, ..., b
k,k
tha
mn hai tnh cht trn.
V a
k
,
.
.
.p nn hai phn t khc nhau bt k trong tp
a
k
b
k,1
, a
k
b
k,2
, ..., a
k
b
k,k

l phn bit theo mod p.


a
k
k
1(modp) (a
k
b
k,1
)(a
k
b
k,2
)...(a
k
b
k,k
) b
k,1
b
k,2
...b
k,k
(mod p)
Din n Ton hc Chuyn S hc
6.2. H thng d 113
T hai iu trn suy ra tn ti ch s j(1 j k) sao cho a
k
b
k,j
/
b
k,1
, b
k,2
, ..., b
k,k
.
Xt tp b
k,1
, b
k,2
, ..., b
k,k
, a
k
b
k,j
.
Sau khi nh s li cc phn t ta thu c tp
b
k+1,1
, b
k+1,2
, ..., b
k+1,k
, b
k+1,k+1

. Ta thy tp ny c k + 1 phn t tha mn hai tnh cht trn nn


theo nguyn l quy np, b c chng minh.
Quay li bi ton, p dng b 6.3, xt tp b
p1,1
, b
p1,2
, ..., b
p1,p1
,
ta thy tp ny l mt HTG mod p nn n cha ng mt phn t
ng d vi 2 mod p. V phn t ny khc 1 nn n phi ng d vi
tch ca mt s a
k
. Suy ra pcm.
Trong tp con tp s nguyn dng, bi ton s hc chia ht
V d 6.7. Cho p > 3 l s nguyn t c dng 3k + 2.
a. Chng minh rng tp A =
_
2
3
1; 3
3
1; 4
3
1; ...; p
3
1
_
l
HTG mod p.
b. Chng minh rng
p

i=1
(i
2
+i + 3) 3(modp).
Li gii. a. Ta s chng minh tp A tha mn 3 iu kin nu
nh ngha 6.2.
Hin nhin mi phn t ca A u khng chia ht cho p
(tha mn iu kin (i)).
Gi s tn ti 1 i < j p 1 sao cho
i
3
1 j
3
1 (mod p)
i
3
j
3
(mod p)
i
3k
j
3k
(mod p)
Mt khc, theo nh l Ferma, ta c: i
3k+1
j
3k+1
(mod p)
T suy ra i j (mod p) i = j (mu thun). Vy A
tha mn iu kin (ii).
Chuyn S hc Din n Ton hc
114 6.2. H thng d
V (p) = p 1 = [A[ nn iu kin (iii) tha mn.
Vy A l mt HTG mod p.
b. V B = 1; 2; 3; ...; p 1 l mt HTG mod p. M A cng l mt
HTG mod p (theo phn a.) nn ta c:
p

i=2
(i
3
1) (p 1)! (mod p)

i=2
(i
2
+i + 1) 1 (mod p)

i=1
(i
2
+i + 1) 3 (mod p)
Nhn xt. Ta c th m rng V d 6.7 nh sau:
V d 6.8. Cho p l s nguyn t l c dng mk + 2 (m, k l cc s
nguyn dng, m > 2). Tm s d ca php chia
T =
p

t=1
(t
m1
+t
m2
+... +t + 1)
cho p.
V d 6.9. Chng minh rng vi mi s nguyn dng n, tn ti s t
nhin n gm n ch s u l v n chia ht cho 5n.
Li gii. Xt s x
n
= a
1
a
2
...a
n
= 5
n
.a tha mn (vi a
i
Z
+
l vi
mi i = 1, 2, ..., n v a Z
+
)
Ta s chng minh bi ton bng phng php quy np ton hc.
Vi n = 1 a
1
= 5
.
.
.5
1
. Vy mnh ng vi n = 1.
Gi s mnh ng vi n x
n
= a
1
a
2
...a
n
= 5
n
.a, cn chng minh
mnh ng vi n + 1.
Xt 5 s sau y:
a
1
= 1a
1
a
2
...a
n
= 5
n
(1.2
n
+a)
a
2
= 3a
1
a
2
...a
n
= 5
n
(3.2
n
+a)
a
3
= 5a
1
a
2
...a
n
= 5
n
(5.2
n
+a)
a
4
= 7a
1
a
2
...a
n
= 5
n
(7.2
n
+a)
a
5
= 9a
1
a
2
...a
n
= 5
n
(9.2
n
+a)
Din n Ton hc Chuyn S hc
6.2. H thng d 115
Do B = 1, 3, 5, 7, 9 l mt H mod 5 cho nn
B

= 1.2
n
+ 1; 3.2
n
+a; 5.2
n
+a; 7.2
n
+a; 9.2
n
+a
cng l H mod 5 nn tn ti duy nht mt s trong B chia ht
cho 5.
Trong 5 s a
1
; a
2
; a
3
; a
4
; a
5
c duy nht mt s chia ht cho 5(n+1)
m s ny gm n + 1 ch s l. Vy mnh ng vi n + 1.
Theo nguyn l quy np, mnh ng vi mi n nguyn dng. Vy
vi mi s nguyn dng n, lun tn ti mt s t nhin gm n ch s
u l v chia ht cho 5n.
Trong mt s dng ton S hc khc
Ngoi cc ng dng nu trn, h thng d cn c dng trong nhiu
dng ton s hc khc, n biu nh trong cc bi ton lin quan ti
tnh tng, gii phng trnh nghim nguyn (phng trnh Diophant
bc nht). Sau y xin nu ra mt s v d.
V d 6.10. Vi mi cp s nguyn t cng nhau (p,q), t
S =
_
q
p
_
+
_
2q
p
_
+... +
_
(p 1)q
p
_
a. Chng minh rng: S =
(p 1)(q 1)
2
b. Xc nh gi tr ca p, q S l s nguyn t
Li gii. a. Ta c
_
kq
p
_
=
r
k
q
, y r
k
l s d trong php chia q
cho p (0 r
k
p 1).
Ta c:
S =
q
p
+
2q
p
+... +
(p 1)q
p

_
r
1
p
+
r
2
p
+... +
r
p1
p
_
V (p, q) = 1 r
k
,= 0 k = 1, 2, ..., p 1, t ta thy tp A =
r
1
; r
2
; ...; r
p1
chnh l mt hon v ca tp A = 1; 2; ...; p 1.
Chuyn S hc Din n Ton hc
116 6.2. H thng d
Tht vy, ngc li, gi s i, j 1; 2; ...; p 1 , i < j m
r
i
= r
j

_
1 j i p 2
(j i)q
.
.
.p

_
j i p 2
j i
.
.
.p
(v l)
Ta c:
r
1
p
+
r
2
p
+... +
r
p1
p
=
1 + 2 +... +p 1
p
=
p 1
2
S =
(p 1)(q 1)
2
(6.7)
b. T (6.7) suy ra S l s nguyn t cn c p, q > 1 v t nht
mt trong hai s p, q l.
Trng hp 1: p, q cng l p, q 3, p ,= q (do (p,q)=1),
kt hp vi (6.7) S l s chn ln hn 2 S khng phi
l s nguyn t.
Trng hp 2: p l s chn, q l s l
S P
_

_
_

_
(p, q) = 1
p 1 = 1
q 1
2
P
_

_
(p, q) = 1
p 1 P
q 1
2
= 1

_
_
p = 2
q = 2h + 1 (h P)
_
q = 3
p = t + 1 (t P, t 2 (mod 3))
(6.8)
Din n Ton hc Chuyn S hc
6.3. nh l thng d Trung Hoa 117
Trng hp 3: q l s chn, p l s l. Tng t trng hp
2, ta c:
_

_
_
p = 2m+ 1(m P)
q = 2
_
p = 3
q = n + 1(n P, n 2 (mod 3))
(6.9)
T (6.8) v (6.9) ta c cc cp s p, q cn tm.
V d 6.11. Cho a, b, c l cc s nguyn dng tha mn a b c
v (a, b, c) = 1. Chng minh rng nu n > ac + b th phng trnh
n = ax +by +cz c nghim nguyn dng.
Li gii. Gi (a, c) = d (b, d) = 1 A = bi
d
i=1
l H mod d
y 1, 2, ..., d sao cho by n (modd) (n by)
.
.
.d.
Do (a, c) = d a = a
1
d; c = c
1
d (a
1
, c
1
Z
+
; (a
1
, c
2
) = 1) B =
a
1
j
c
1
j=1
l H mod c
1
.
x 1, 2, ..., c
1
sao cho a
1
x
n by
d
(mod c
1
) z Z sao cho
n by
d
= a
1
x +c
1
z.
Mt khc, ta c:
n by
d
>
ac +b by
d
= (d 1)
ca
1
b
d
+a
1
c
1
a
1
c
1
a
1
x z Z
+
T y suy ra n by = ax +cz n = ax +by +cz.
Vy nu n > ac+b th phng trnh n = ax+by+cz c nghim nguyn
dng.
6.3 nh l thng d Trung Hoa
6.3.1 Kin thc c bn
nh l 6.1 Cho k s nguyn dng n
1
, n
2
, ..., n
k
i mt ngun t
cng nhau v k s nguyn bt k a
1
, a
2
, ..., a
k
. Khi tn ti s nguyn
a tha mn a a
i
(mod n
i
), i = 1, k.
Chuyn S hc Din n Ton hc
118 6.3. nh l thng d Trung Hoa
S nguyn b tha mn b a
i
(mod n
i
), i = 1, k khi v ch khi b a
(mod n) vi n = n
1
n
2
...n
k
.
Li gii. t n = n
1
n
2
...n
k
v t N
i
=
n
n
i
.
Do (n
i
, n
j
) = 1, i ,= j nn suy ra (N
i
, n
i
) = 1 i = 1; k.
Do (N
i
, n
i
) = 1, i = 1; k nn vi mi i(1 i k) tn ti b
i
sao
cho
N
i
b
i
1 (mod n
i
) (6.10)
Nh vy ta c b b
1
, b
2
, ..., b
k
. Do N
j
0 (mod n
i
) khi i ,= j, t
d nhin suy ra
N
j
b
j
0 (mod n
i
) (6.11)
t a =
k

j=1
N
j
b
j
a
j
.
Vi mi i (1 i k) ta c
a = N
i
b
i
a
i
+
k

j=1;j=i
N
j
b
j
a
j
(6.12)
T (6.10),(6.11),(6.12) suy ra a a
i
(mod n
i
), i = 1, k.
D thy, v n
1
, n
2
, ..., n
k
i mt nguyn t cng nhau nn ta c
kt lun sau: S nguyn b tha mn b a
i
(mod n
i
), i = 1, k
khi v ch khi b a (mod n) vi n = n
1
n
2
...n
k
.
Nhn xt. 1. Ngoi cch chng minh trn, ta cn c th s dng
php quy np chng minh nh l thng d Trung Hoa.
2. nh l Thng d Trung Hoa khng nh v s tn ti duy nht
ca mt lp thng d cc s nguyn tha mn ng thi nhiu
ng d tuyn tnh. Do c th dng nh l gii quyt nhng
bi ton v s tn ti v m cc s nguyn tha mn mt h cc
Din n Ton hc Chuyn S hc
6.3. nh l thng d Trung Hoa 119
iu kin quan h, chia ht,..., hay m s nghim ca phng
trnh ng d. Vic s dng hp l cc b v (trong nh l) cho
ta rt nhiu kt qu th v v t c th a ra nhiu bi ton
hay v kh.
V d 6.12. Cho m
1
, m
2
, ..., m
n
l cc s nguyn dng, r
1
, r
2
, ..., r
n
l cc s nguyn bt k. Chng minh rng iu kin cn v h
phng trnh ng d
x r
1
(mod m
1
)
x r
2
(mod m
2
)
...
x r
n
(mod m
n
)
c nghim l r
i
r
j
(mod GCD (m
i
, m
j
)); 1 i < j n.
Nu x
0
v x
1
l hai nghim tha mn h phng trnh trn th x
0
x
1
(mod m) vi m = LCM(m
1
, m
2
, ..., m
n
). Tc l h phng trnh
cho c nghim duy nht theo module m.
Li gii. Trc ht ta gi s h phng trnh cho c nghim x
0
. t
GCD (m
i
, m
j
) = d, ta c:
x
o
r
i
0 (mod m
i
)
x
o
r
j
0 (mod m
j
)
Suy ra r
i
r
j
mod (GCD (m
i
, m
j
)). Do i, j ty chn nn r
i
r
j
(mod GCD(m
i
, m
j
)), 1 i < j n. y l iu kin cn h
phng trnh c nghim.
Ngc li, ta s chng minh bng quy np theo n rng nu iu kin
trn c tha mn th h phng trnh lun c nghim duy nht theo
module m vi m = LCM (m
1
, m
2
, ..., m
n
).
Vi trng hp n = 2, t GCD (m
1
, m
2
) = d m
1
= dd
1
; m
2
= dd
2
vi GCD (d
1
, d
2
) = 1.
Suy ra r
i
r
j
r (mod d). t r
1
= r +k
1
d; r
2
= r +k
2
d.
Chuyn S hc Din n Ton hc
120 6.3. nh l thng d Trung Hoa
Ta c:
_
x r
1
(mod m
1
)
x r
2
(mod m
2
)

_
_
_
(x r) k
1
d
.
.
.dd
1
(x r) k
2
d
.
.
.dd
2

_
_
_
x r
d
k
2
(mod d
1
)
x r
d
k
2
(mod d
2
)
(6.13)
Do (d
1
, d
2
) = 1 nn theo nh l Thng d Trung Hoa, tn ti mt s
dng x sao cho x k
1
(mod d
1
); x k
2
(mod d
2
). V x v
x r
d
l hai nghim ca phng trnh
_
x k
1
(mod d
1
)
x k
2
(mod d
2
)
nn
x r
d
x
(mod d
1
d
2
) hay x xd +r (mod dd
1
d
2
).
Do m = LCM (m
1
, m
2
) = dd
1
d
2
nn theo nh l Thng d Trung Hoa,
h c nghim duy nht module m.
Gi s nh l ng n n 1. Ta s chng minh nh l ng n n.
t m

1
= LCM (m
1
, m
2
, ..., m
n1
) ; m

2
= m
n
; r

2
= r
n
. V r
i

r
j
(modGCD (m
i
, m
j
)) vi mi 1 i < j n nn theo gi thit quy
np, h phng trnh
_
x r
i
(mod m
i
)
i = 1, n 1
c duy nht nghim x r

1
(mod m

1
).
Mt khc t r
i
r
j
( mod GCD(m
i
, m
j
)) vi mi 1 i < j n suy ra
r

1
r

2
(mod GCD(m

1
, m

2
)).
Theo chng minh trn cho trng hp n = 2 ta c h phng trnh
_
x r

1
(mod m

1
)
x r

2
(mod m

2
)
c nghim duy nht theo module
m = LCM
_
m

1
, m

2
_
= LCM (m
1
, m
2
, ..., m
n
)
. Theo nguyn l quy np ta c iu phi chng minh.
Nhn xt. y chnh l nh l Thng d Trung Hoa dng m rng,
n hon ton chng minh da trn c s nh l Thng d Trung Hoa.
Trong bi vit ny, ta s khng i su vo tm hiu nh l dng m
rng m ch i su vo cc ng dng ca nh l Thng d Trung Hoa
(dng thng).
Din n Ton hc Chuyn S hc
6.3. nh l thng d Trung Hoa 121
6.3.2 ng dng
Trong L thuyt s
V d 6.13. Chng minh rng vi mi s t nhin n, tn ti n s t
nhin lin tip m mi s trong n s u l hp s.
Li gii. tng: ta s to ra mt h phng trnh ng d gm n
phng trnh ng d. Da vo nh l thng d Trung Hoa, ta kt
lun c s tn ti nghim ca h .
Gi s p
1
, p
2
, ..., p
n
l n s nguyn t khc nhau tng i mt.
Xt h phng trnh ng d x k (mod p
2
k
)(k = 1, 2, ..., n).
Theo nh l thng d Trung Hoa, tn ti x
0
N

sao cho x
0
k
(mod p
2
k
), k = 1, 2, ..., n.
Khi cc s x
0
+ 1; x
0
+ 2, ...; x
0
+n u l hp s.(pcm)
V d 6.14. Chng minh rng vi mi s t nhin n, tn ti n s t
nhin lin tip sao cho bt k s no trong cc s cng u khng
phi ly tha (vi s m nguyn dng) ca mt s nguyn t.
Nhn xt. Bi ny cng gn tng t vi tng ca bi ton v d
cng c. Tuy nhin vic tm ra h phng trnh ng d kh hn mt
cht.
Li gii. Vi mi s t nhin n, xt n s nguyn t khc nhau tng i
mt p
1
, p
2
, ..., p
n
.
Theo nh l Thng d Trung Hoa, tn ti a N

sao cho a p
k
k
(mod p
2
k
) (k = 1, 2, ..., n).
Khi d thy rng cc s a + 1, a + 2, ..., a + n u khng phi ly
tha vi s m nguyn dng ca mt s nguyn t (pcm).
V d 6.15. Cho trc cc s nguyn dng n, s. Chng minh rng tn
ti n s nguyn dng lin tip m mi s u c c l ly tha bc
s ca mt s nguyn dng ln hn 1.
Li gii. Xt dy F
n
= 2
2
n
+ 1, (n = 0, 1, 2, ...). D chng minh b
sau:
B 6.4 Nu n ,= m th (F
n
, F
m
) = 1.
Chuyn S hc Din n Ton hc
122 6.3. nh l thng d Trung Hoa
p dng nh l Thng d Trung Hoa cho n s nguyn t cng nhau
F
s
1
, F
s
2
, ..., F
s
n
v n s r
i
= i(i = 1, 2, .., n) ta c tn ti s nguyn c
sao cho c +i
.
.
.F
s
i
.
Vy dy c +i
n
i=1
l n s nguyn dng lin tip, s hng th i chia
ht cho F
s
i
.
V d 6.16. Chng minh rng tn ti mt a thc P(x) Z[x], khng
c nghim nguyn sao cho vi mi s nguyn dng n, tn ti s nguyn
x sao cho P(x) chia ht cho n.
Li gii. Ta c th xt a thc P(x) = (3x + 1)(2x + 1).
Vi mi s nguyn dng n, ta biu din n di dng n = 2
k
(2m+ 1).
V GCD(2
k
, 3) = 1 nn tn ti a sao cho 3a 1 (mod 2
k
). T
3x 1 (mod 2
k
) x a (mod 2
k
)
Tng t GCD (2, 2m+1) = 1 nn tn ti b sao cho 2b 1 (mod (2m+
1)). T
2x 1 (mod (2m+ 1)) x b (mod (2m+ 1))
Cui cng, do GCD (2
k
, 2m+1) = 1 nn theo nh l Thng d Trung
Hoa, tn ti s nguyn x l nghim ca h:
_
x a (mod 2
k
)
x b (mod (2m+ 1))
V theo l lun trn, P(x) = (3x + 1)(2x + 1)
.
.
.n.
V d 6.17. Trong li im nguyn ca mt phng ta Oxy, mt
im A vi ta (x
0
, y
0
) Z
2
c gi l nhn thy t O nu on
thng OA khng cha im nguyn no khc ngoi A, O. Chng minh
rng vi mi n nguyn dng ln ty , tn ti hnh vung nn c cc
nh nguyn, hn na tt c cc im nguyn nm bn trong v trn
bin ca hnh vung u khng nhn thy c t O.
Din n Ton hc Chuyn S hc
6.3. nh l thng d Trung Hoa 123
Li gii. D thy iu kin cn v im A(x
0
, y
0
) nhn thy c
t O l gcd(x
0
, y
0
) = 1.
gii quyt bi ton, ta s xy dng mt hnh vung n n vi n
nguyn dng ln ty sao cho vi mi im nguyn (x, y) nm trong
hoc trn hnh vung u khng th nhn thy c t O.
Tht vy, chn p
i
j
l cc s nguyn t i mt khc nhau vi 0 i, j
n. Xt hai h ng d sau:
_

_
x 0 (mod p
0
1
p
0
2
...p
0
n
)
x + 1 0 (mod p
1
1
p
1
2
...p
1
n
)
x + 2 0 (mod p
2
1
p
2
2
...p
2
n
)
...
x +n 0 (mod p
n
1
p
n
2
...p
n
n
)
v
_

_
y 0 (mod p
0
1
p
0
2
...p
0
n
)
y + 1 0 (mod p
1
1
p
1
2
...p
1
n
)
y + 2 0 (mod p
2
1
p
2
2
...p
2
n
)
...
y +n 0 (mod p
n
1
p
n
2
...p
n
n
)
Theo nh l Thng d Trung Hoa th tn ti (x
0
, y
0
) tha mn hai h
ng d trn.
Khi , r rng gcd(x
0
+i, y
0
+i) > 1, i, j = 0, 1, 2, ..., n.
iu c ngha l mi im nm bn trong hoc trn bin hnh vung
n n xc nh bi im pha di bn tri l (x
0
, y
0
) u khng th
nhn thy c t O. Bi ton c chng minh.
Trong tm s lng nghim nguyn ca mt phng trnh
nghim nguyn
V d 6.18. Cho s nguyn dng n = p

1
1
p

2
2
...p

k
k
, trong p
1
, p
2
, ..., p
k
l cc s nguyn t i mt khc nhau. Tm s nghim ca phng
trnh:
x
2
+x 0 (mod n)
Chuyn S hc Din n Ton hc
124 6.3. nh l thng d Trung Hoa
Li gii. Ta c:
x
2
+x 0 (mod n)
_
x(x + 1) 0 (mod p

i
i
)
i = 1, k

_
_
x 0 (mod p

i
i
)
x 1 (mod p

i
i
)
i = 1, k
(6.14)
Theo nh l Thng d Trung Hoa, mi h phng trnh x
2
+ x 0
(mod n)
_
_
_
x a
i
(mod p

i
i
)
a
i
1; 0
i = 1, k
c duy nht mt nghim v ta c 2
k
h (bng s b (a
1
, a
2
, ..., a
k
), a
i
1; 0), nghim ca cc h khc
nhau. Suy ra phng trnh cho c ng 2
k
nghim.
V d 6.19. Cho m = 2007
2008
. Hi c tt c bao nhiu s t nhin
n<m sao cho m[n(2n + 1)(5n + 2) .
Li gii. D thy GCD (m; 10) = 1. Do :
n(2n + 1)(5n + 2) 0 (mod m)
10n(10n + 5)(10n + 4) 0 (mod m)
(6.15)
Ta c: m = 3
4016
.223
2008
. cho thun tin, t 10n = x; 3
4016
=
q
1
; 223
2008
= q
2
.
Khi GCD (q
1
, q
2
) = 1 nn (6.15) tng ng vi:
x(x + 5)(x + 4) 0 (mod q
1
) (6.16)
x(x + 5)(x + 4) 0 (mod q
2
) (6.17)
D thy:
(6.16) xy ra khi v ch khi x 0 (mod q
1
) hoc x 5 (mod q
1
)
hoc x 4 (mod q
1
).
(6.17) xy ra khi v ch khi x 0 (mod q
2
) hoc x 5 (mod q
2
)
hoc x 4 (mod q
2
).
Din n Ton hc Chuyn S hc
6.4. Bi tp ngh & gi p s 125
Do t (6.16) v (6.17), vi lu rng x 0 (mod 10), suy ra n l
s t nhin tha mn cc iu kin bi khi v ch khi n =
x
10
, vi x
l s nguyn tha mn h iu kin sau:
_

_
x 0 (mod 10)
x 1 (mod q
1
)
x r
2
(mod q
2
)
0 x < 10q
1
q
2
r
1
, r
2
0; 4; 5
(6.18)
V 10; q
1
; q
2
i mt nguyn t cng nhau nn theo nh l Thng d
Trung Hoa, h (6.18) c nghim duy nht.
D thy s c 9 s x l nghim ca 9 h (6.18) tng ng. V mi s x
cho ta mt s n v hai s x cho hai s n khc nhau nn c 9 s n tha
mn cc iu kin bi.
Nhn xt. V d 6.19 chnh l trng hp c bit ca bi ton tng
qut sau:
V d 6.20. Cho s nguyn dng n c phn tch tiu chun n =
p

1
1
p

2
2
...p

k
k
. Xt a thc P(x) c h s nguyn. Nghim x
0
ca phng
trnh ng d P(x) 0 (mod n) l lp ng d x
0

_
0, 1, 2, ..., n 1
_
tha mn P(x
0
) 0 (mod n). Khi , iu kin cn v phng
trnh P(x) 0 (mod n) c nghim l vi mi i = 1, 2, ..., s, phng
trnh P(x) 0 (mod p

i
i
) c nghim. Hn na, nu vi mi i =
1, 2, ..., s, phng trnh P(x) 0 (mod p

i
i
) c r
i
nghim module p

i
i
th phng trnh c r = r
1
r
2
...r
s
nghim module n.
6.4 Bi tp ngh & gi p s
Bi tp ngh
Bi 1. a. Chng minh rng: Nu (a, m) = 1 v x chy qua mt
h thng d y modulo m th ax + b, vi b l mt
s nguyn ty , cng chy qua mt h thng d y
module m.
Chuyn S hc Din n Ton hc
126 6.4. Bi tp ngh & gi p s
b. Chng minh rng: Nu (a, m) = 1 v x chy qua mt h
thng d thu gn modulo m th ax cng chy qua mt h
thng d thu gn module m.
Bi 2. Mi s nguyn dng T c gi l s tam gic nu n c dng
T =
k(k + 1)
2
, trong k l mt s nguyn dng. Chng minh
rng tn ti mt H module n gm n s tam gic.
Bi 3. a. Cho m
1
, m
2
l hai s nguyn dng nguyn t cng nhau.
Chng minh rng:
(m
1
m
2
) = (m
1
).(m
2
)
b. Gi s s nguyn dng m c phn tch chnh tc thnh
tch cc tha s nguyn t m = p

1
1
p

2
2
...p

k
k
. Chng minh
rng:
(m) = p

1
1
1
p

2
1
2
...p

k
1
k
(p
1
1)(p
2
2)...(p
k
1)
Bi 4. Tnh tng sau:
S =
2012

k=6
_
17
k
11
_
Bi 5. Cho s nguyn dng n v s nguyn t p ln hn n+1. Chng
minh rng a thc P(x) = 1 +
x
n + 1
+
x
2
2n + 1
+ ... +
x
p
pn + 1
khng c nghim nguyn.
Bi 6. Cho p l s nguyn t c dng 3k + 2 (k nguyn dng). Tm
s d khi chia S =
p

k=1
(k
2
+k + 1) cho p.
Bi 7. Cho cc s nguyn dng a, b tha mn (a, b) = 1. Chng minh
rng phng trnh ax + by = 1 c v s nghim nguyn (x, y)
v (x, a) = (y, b) = 1.
Din n Ton hc Chuyn S hc
6.4. Bi tp ngh & gi p s 127
Bi 8. Tm s nguyn dng nh nht c tnh cht: chia 7 d 5, chia
11 d 7, chia 13 d 3.
Bi 9. Chng minh rng tn ti mt dy tng a
n

n=1
cc s t nhin
sao cho vi mi s t nhin k, dy k +a
n
ch cha hu hn
cc s nguyn t.
Bi 10. S nguyn dng n c gi l c tnh cht P nu nh vi cc
s nguyn dng a, b m a
3
b + 1
.
.
.n th a
3
+ b
.
.
.n. Chng minh
rng s cc s nguyn dng c tnh cht P khng vt qu 24.
Bi 11. Tm tt c cc s t nhin n tha mn 2
n
1 chia ht cho 3 v
c mt s nguyn m m
2
n
1
3
[4m
2
+ 1.
Bi 12. Chng minh rng tn ti s t nhin k sao cho tt c cc s
k.2
n
+ 1 (n = 1, 2, ...) u l hp s.
Gi p s
Bi 1. Chng minh trc tip da vo nh ngha.
Bi 2. Ta chng minh n phi c dng n = 2
k
. Phn chng, gi s
n = 2
k
.m vi m l v m > 1. S dng tnh cht h thng d
y .
Bi 3. Ta c th chng minh da vo kin thc v h thng d y
, cng c th chng minh da vo nh l Thng d trung
Hoa.
Bi 4. S dng HTG.
Bi 5. Biu din P(x) di dng P(x) = a
p
x
p
+a
p1
x
p1
+... +a
2
x
2
+
a
1
x + a
0
. Phn chng, gi s P(x) c nghim nguyn x = u.
Suy ra mu thun.
Bi 6. Tin hnh tng t V d 6.7.
Bi 7. S dng kin thc H.
Chuyn S hc Din n Ton hc
128 6.4. Bi tp ngh & gi p s
Bi 8. p s: 887.
Bi 9. Gi p
k
l s nguyn t th k, k > 0. Theo nh l Thng d
Trung Hoa, tn ti dy s a
n

n=1
tha mn a
1
= 2; a
n
=
k(modp
k+1
), k n.
Bi 10. nh l Thng d Trung Hoa.
Bi 11. Chng minh n c dng 2
k
. S dng tnh cht ca s Fecma
(xem li V d 6.15).
Bi 12. V d 6.15 v Bi 3.
Din n Ton hc Chuyn S hc
Chng
7
Mt s bi ton s hc
hay trn VMF
7.1 m
3
+ 17
.
.
.3
n
129
7.2 c(ac + 1)
2
= (5c + 2)(2c +b) 136
Phn ny gm mt s bi ton hay c tho lun nhiu trn Din
n Ton hc. Bn c c th vo trc tip topic ca bi ton
trn Din n Ton hc, bng cch click vo tiu ca bi ton
.
7.1 m
3
+ 17
.
.
.3
n
Bi ton 7.1. Chng minh rng vi mi s nguyn dng n, tn ti
mt s t nhin m sao cho
_
m
3
+ 17
_
.
.
.3
n
.
u tin, chng ta n vi chng minh xut cho bi ton u bi.
Chng minh. Ta s chng minh bi ton bng quy np.
Vi n = 1, ta chn m = 4.
Vi n = 2, ta chn m = 1.
Gi s bi ton ng n n = k, hay m N : m
3
+ 17
.
.
.3
k
Ta chng minh rng i vi trng hp n = k +1 cng ng tc l tn
ti mt s m

sao cho m
3
+ 17
.
.
.3
k+1
.
t m
3
+ 17 = 3
k
.n n ,
.
.
.3.
129
130 7.1. m
3
+ 17
.
.
.3
n

_
n 2
n 1
(mod3)
_
m
3
+ 17 2.3
k
m
3
+ 17 3
k
_
mod3
k+1
_
Trng hp 1: m
3
+ 17 2.3
k
(mod 3
k+1
)
Xt:
(m+3
k1
)
3
= m
3
+m
2
3
k
+m3
2k1
+3
3k3
m
3
+m
2
3
k
(mod 3
k+1
)
(Do k 2 3
2k1
.
.
.3
k+1
v 3
3k3
.
.
.3
k+1
).
Suy ra:
_
m+ 3
k1
_
3
+17 m
3
+m
2
.3
k
+17 2.3
k
+m
2
.3
k
0 (mod 3
k+1
)
(v m ,
.
.
.3 m
2
1 (mod 3) 2 +m
2
.
.
.3 (2 +m
2
).3
k
.
.
.3
k+1
).
Nh vy, trng hp 1, ta c:
_
m+ 3
k1
_
3
+ 17
.
.
.3
k+1
.
Trng hp 2: m
3
+ 17 3
k
(mod 3
k+1
).
Xt:
_
m3
k1
_
3
= m
3
m
2
3
k
+m3
2k1
3
3k3
m
3
m
2
3
k
(mod 3
k+1
)
(Do k 2 3
2k1
.
.
.3
k+1
v 3
3k3
.
.
.3
k+1
).
Suy ra:
_
m3
k1
_
3
+ 17 m
3
m
2
3
k
+ 17 3
k
m
2
3
k
0 (mod 3
k+1
)
(v m ,
.
.
.3 m
2
1 (mod 3) 1 m
2
.
.
.3
_
1 m
2
_
.3
k
.
.
.3
k+1
).
Nh vy, trng hp 2 ta c:
_
m3
k1
_
3
+ 17
.
.
.3
k+1
.
Tm li, ta u tm c s nguyn t ,
.
.
.3 m t
3
+ 17
.
.
.3
k+1
.
Ta chng minh c vn ng trong trng hp n = k + 1.
Theo nguyn l quy np, ta c pcm.
Mu cht bi ton ny l b sau:
Din n Ton hc Chuyn S hc
7.1. m
3
+ 17
.
.
.3
n
131
B 7.1 Cho a, b, q l cc s nguyn tha (a; q) = 1 v q > 0.
Khi y, lun tn ti k Z sao cho ak b
.
.
.q.
Chng minh. Ta chng minh i din cho trng hp ak+b
.
.
.q. Trng
hp cn li tng t.
Xt A = 1; 2; 3; ...; q l 1 h y H mod q.
Theo tnh cht ca H thng d, ta c tp B = a; 2a; 3a; ...; qa cng
l H mod q.
C = a +b; 2a +b; 3a +b; ...; qa +b cng l H mod q.
Do , tn ti k [1; q] sao cho ak +b
.
.
.q.
Nhn xt. Bi ton cho thc cht l yu cu tm 1 s x nguyn sao
cho x + 17
.
.
.3
n
v x l lp phng 1 s nguyn. B trn cho thy
s tn ti ca x nguyn x + 17
.
.
.3
n
. Cn vic tm x l x l lp
phng 1 s nguyn th ta s dng phng php quy np nh trn.
i vi 1 ngi yu ton, ta phi khng ngng sng to. Ta hy th
tng qut bi ton cho:
thay v m
3
, ta th thay m
k
vi k l s nguyn dng c nh.
thay v 3
n
, ta th thay p
n
vi p l 1 s nguyn t.
thay s 17 bi y N vi y c nh.
Kt hp cc thay i trn, ta c 1 bi ton "tng qut" hn
D on 7.1 Cho p l s nguyn t. y, k N v y, k c nh.
Khng nh hoc ph nh mnh sau
n N, x N : x
k
+y
.
.
.p
n
(7.1)
Ta th thay mt vi gi tr p, k, y vo th xem (7.1) c ng khng.
Khi thay k = 2, y = 1, p = 3 th mnh (7.1) tr thnh
n N, x N : x
2
+ 1
.
.
.3
n
(7.2)
Chuyn S hc Din n Ton hc
132 7.1. m
3
+ 17
.
.
.3
n
Rt tic, khi ny, (7.2) li sai!!!. Ta s chng minh (7.2) sai khi n 1.
Tht vy, chng minh d on 7.1 sai, ta cn c b sau
B 7.2 Cho p l s nguyn t dng 4k + 3 v a, b Z. Khi
a
2
+b
2
.
.
.p (a
.
.
.p) (b
.
.
.p)
T (7.2), suy ra x
2
+ 1
.
.
.3. p dng b 7.2 vi p = 3, ta suy ra 1
.
.
.3:
v l.
Vy khi n 1 th ,x Z : x
2
+ 1
.
.
.3
n
.
Khng nn lng, ta th thm mt vi iu kin (7.1) tr nn cht
hn v ng. Nu bn c c kin no hay, xin hy gi vo topic ny
tho lun. Sau khi thm mt s iu kin, ta c 1 bi ton hp hn
nhng lun ng.
nh l 7.1 Cho p nguyn t l. y, k N v y, k c nh.
Bit rng gcd(k, p) = gcd(k, p 1) = gcd(y, p) = 1.
Chng minh rng:
n N, x N : x
k
+y
.
.
.p
n
(7.3)
Chng minh. Trc ht, chng minh (7.3), ta cn c b sau
B 7.3 Cho p l s nguyn t l. k nguyn dng tha
(k; p) = (k 1; p) = 1
Khi , 1
k
; 2
k
; ...; (p 1)
k
l HTG modp.
Chng minh. Gi g l cn nguyn thy ca p tc l ord
p
(g) = p 1.
Khi y th g
1
, g
2
, ..., g
p1
lp thnh 1 HTG modp v r rng
g
a
1
, g
a
2
, ..., g
a
p1
l HTG modp a
1
, a
2
, .., a
p1
l H ca p 1.
Vi 1 i p 1 th tn ti a
i
m i g
a
i
(mod p) v r rng
a
i
lp thnh 1 HTG modp nn h 1
k
, 2
k
, .., (p 1)
k
c th vit li l
g
k
, g
2k
, ..., g
(p1)k
, n l HTG modp khi v ch khi k, 2k, ..., (p 1)k l
h thng d y ca p 1, tc l k nguyn t cng nhau vi p 1.
B c chng minh.
Din n Ton hc Chuyn S hc
7.1. m
3
+ 17
.
.
.3
n
133
Quay li bi ton. Ta chng minh (7.3) bng phng php quy np.
Vi n = 1, theo b 7.3 th
x
0
1; 2; ...; p 1 : x
k
0
y (mod p) x
k
0
+y
.
.
.p
Gi s bi ton ng nn hay tn ti x
k
+y
.
.
.p
n
Ta s chng minh n + 1 cng ng hay tn ti x
k
0
+y
.
.
.p
n+1
Tht vy, t gi thit quy np suy ra x
k
+y = p
n
.q
Trng hp 1: q
.
.
.p pcm
Trng hp 2:
gcd(q, p) = 1 (7.4)
Khi ta chn x
0
= v.p
n
+x
Do
x
k
0
+y = (v.p
n
+x)
k
+y
= v
k
.p
nk
+
_
1
k
_
.v
k1
.p
n(k1)
.x +... +
_
k 1
k
_
.v.p
n
.x
k1
+ (x
k
+y)
(7.5)
D dng chng minh
p
n+1
[ v
k
.p
nk
+
_
1
k
_
.v
k1
.p
n(k1)
.x +...
_
k 2
k
_
.v
2
.p
2n
.x
k2
Do vy ta xt
_
k 1
k
_
.v.p
n
.x
k1
+ (x
k
+y) = k.v.p
n
.x
k1
+p
n
.q = p
n
(k.v.x
k1
+q)
Nhn thy gi s k.x
k1
t (mod p) m gcd(k, p) = 1 v x
k
+y
.
.
.p
gcd(x, p) = 1 (do gcd(y, p) = 1) suy ra gcd(t, p) = 1
Do (k.v.x
k1
+q) tv+q (mod p) m t (7.4) ta c gcd(q, p) = 1
Cho nn lun tn ti v tha mn tv +q
.
.
.p. Do bi ton c khng
inh vi n + 1.
Theo nguyn l quy np, bi ton c chng minh.
Chuyn S hc Din n Ton hc
134 7.1. m
3
+ 17
.
.
.3
n
Cha dng li y, nu trong (7.3), ta thay k bi x, ta s c 1 bi
ton khc:
nh l 7.2 Cho p nguyn t l. y N v y c nh. Bit rng
gcd(y, p) = 1. Khi :
n N, x N : x
x
+y
.
.
.p
n
(7.6)
Chng minh. Ta chng minh bi ton ny bng phng php quy
np. Ta coi nh l 7.1 nh 1 b . D thy nu x tha (7.6) th
gcd(x; p) = 1.
Khi , vi n = 1, ta xt h ng d (I)
_
x k (mod (p 1))
x x
0
(mod p)
trong , x
0
; k N tha x
k
0
+y
.
.
.p.
Do gcd(p 1; p) = 1 nn theo nh l Thng d Trung Hoa th h (I)
lun c nghim x

.
Chn x = x

, ta chng minh x tha (7.6) khi n = 1. Tht vy


gcd(x; p) = 1 x
p1
1 (mod p) x
k
x
x
(mod p)
x
x
+y x
k
+y x
k
0
+y 0 (mod p)
Vy x N : x
x
+y
.
.
.p.
Gi s (7.6) ng n n 1, tc l tn ti x
0
x
0
x
0
+y
.
.
.p
n1
.
Theo cch chng minh quy np (7.6), ta chn c x
n
= ap
n
+ x
0
tha x
x
0
n
+y
.
.
.p
n
.
Khi , d nhn thy x
n
x
0
(mod p
n1
). Ta xt h ng d (II)
_
X x
0
(mod (p
n1
(p 1)))
X x
n
(mod p
n
)
Do gcd(p
n1
(p 1); p
n
) = 1 nn theo nh l Th ng d Trung hoa, h
(II) c nghim X. Ta chng minh x = X tha (7.6). Tht vy
Do (p 1)p
n1
= (p
n
) X
X
X
x
0
(mod p
n
) (nh l Euler).
Din n Ton hc Chuyn S hc
7.1. m
3
+ 17
.
.
.3
n
135
Mt khc X
x
0
x
n
x
0
(mod p
n
) (do cch chn trong h (II)).
X
X
+y x
n
x
0
+y 0 (mod p
n
)
Theo nguyn l quy np, bi ton c chng minh.
M rng ca bi ton u vn cn nhiu, nh tng thm iu kin
chn nh (m
3
+17
.
.
.3
n
) (m
3
+17 ,
.
.
.3
n+1
), v.v. Rt mong nhn c
kin ng gp cho vic m rng.
Li cm n
Rt cm n Nguyen Lam Thinh, Karl Heinrich Marx,nguyenta98, The
Gunner ng gp kin v m rng cho bi vit ny.
Chuyn S hc Din n Ton hc
136 7.2. c(ac + 1)
2
= (5c + 2)(2c +b)
7.2 c(ac + 1)
2
= (5c + 2)(2c + b)
Bi ton 7.2. Cho 3 s nguyn dng a; b; c tho mn ng thc:
c(ac + 1)
2
= (5c + 2b)(2c +b) (7.7)
Chng minh rng : c l s chnh phng l.
Nhn xt. Thot nhn vo bi ton, tht kh tm 1 phng php
cho loi ny. Nhn xt trong gi thit VP (7.7), th b xut hin vi
bc l 2. Th l ta c 1 hng ngh l dng tam thc bc 2 cho bi ton
ny. Ta khng nn chn c v bc ca c l 3, khng chn a v phng
trnh mi theo a hin nhin tr li (7.7)
Chng minh (Chng minh 1).
c (ac + 1)
2
= (5c + 2b) (2c +b)
2b
2
+ 9bc + 10c
2
c (ac + 1)
2
= 0

b
= 81c
2
4.2.
_
10c
2
c (ac + 1)
2
_
= c
2
+ 8c (ac + 1)
2

b
= c
_
c + 8 (ac + 1)
2
_
= x
2
, (x N

)
t
d = GCD(c; c + 8(ac + 1)
2
) d[8 (ac + 1)
2
d[c
_
(ac + 1)
2
; d
_
= 1
_
d[8
Trng hp 1: d=8
_
c
8
;
c
8
+ (ac + 1)
2
_
= 1
c
_
c + 8 (ac + 1)
2
_
= x
2
(x N)
c
8
.
_
c
8
+ (ac + 1)
2
_
=
_
x
8
_
2
8[x x = 8x
2
(x
2
N

)
c
8
.
_
c
8
+ (ac + 1)
2
_
= x
2
2

_
_
_
c
8
= t
2
c
8
+ (ac + 1)
2
= p
2
_
t; p N

(t; p) = 1
_

_
c = 8t
2
t
2
+
_
8t
2
a + 1
_
2
= p
2
M d chng minh
_
8t
2
a + 1
_
2
< t
2
+
_
8t
2
a + 1
_
2
<
_
8t
2
a + 2
_
2

_
8t
2
a + 1
_
2
< p
2
<
_
8t
2
a + 2
_
2
: mu thun
Din n Ton hc Chuyn S hc
7.2. c(ac + 1)
2
= (5c + 2)(2c +b) 137
Do , d = 8 b loi.
Trng hp 2: d=4
_
c
4
;
c
4
+ 2 (ac + 1)
2
_
= 1

c
4
;
c
4
+ 2 (ac + 1)
2
l nhng s chnh phng (*)
Nu
c
4
l s chn
c
4
+ 2 (ac + 1)
2
.
.
.2

_
c
4
;
c
4
+ 2 (ac + 1)
2
_
= 2: mu thun.
Do ,
c
4
l s l. M
c
4
l s chnh phng
c
4
1 (mod 4)
Mt khc, do c chn nn ac + 1 l s l (ac + 1)
2
1 (mod 4)

c
4
+ 2 (ac + 1)
2
1 + 2.1 3 (mod 4): v l do (*).
Do , d = 4 b loi.
Trng hp 3: d=2 .
Tng t t Trng hp 2, ta c
c
2
l
c
2
1 (mod 8)
c chn nn ac + 1 l (ac + 1)
2
1 (mod 8)

c
2
+ 4(ac + 1)
2
1 + 4.1 5 (mod 8) : v l
Do , d = 2 b loi.
Trng hp 4: d=1
Tng t trng hp 2, ta c ngay c l v do (c; c + 8(ac + 1)
2
) = 1
nn c l s chnh phng.
Vy ta c pcm.
Nhn xt. Ta thy trong bi ny, b v c c 1 mi lin quan kh cht
ch vi nhau nn ta th gii theo b, c s dng k thut GCD tc l t
d = GCD(b; c) ta c cch chng minh th 2.
Chng minh (Chng minh 2). t d = (b; c)
_
c = dm
b = dn
_
m; n N

(m; n) = 1
_
Chuyn S hc Din n Ton hc
138 7.2. c(ac + 1)
2
= (5c + 2)(2c +b)
Khi
(7.7) m(dam+ 1)
2
= d (5m+ 2n) (2m+n)
d[m(dam+ 1)
2
(d; dam+ 1) = 1
_
d[m m = dp (p; n) = (d; n) = 1
(7.7) p
_
d
2
ap + 1
_
2
= (5dp + 2n) (2dp +n)
p[ (5dp + 2n) (2dp +n)
(p; 2dp +n) = 1
_
p[5dp + 2n p[2n
(p; n) = 1 p[2 p 1; 2
Trng hp 1: p=2 , khi 2
_
2ad
2
+ 1
_
2
= (10d + 2n) (4d +n),
suy ra
_
2ad
2
+ 1
_
2
= (5d +n) (4d +n). Nhng v (5d +n; 4d +n) =
(d; 4d +n) = (d; n) = 1 Cho nn ta phi c
_
5d +n = x
2
4d +n = y
2
(x; y N

, (x; y) = 1)
Suy ra d = x
2
y
2
. Mt khc
2ad
2
+ 1 = xy a =
xy 1
2d
2
=
xy 1
2 (x
2
y
2
)
2
Ta chng minh 2
_
x
2
y
2
_
2
> (x +y)
2
> xy 1
Tht vy
(x +y)
2
4xy > xy 1
2
_
x
2
y
2
_
2
(x +y)
2
= (x +y)
2
_
2 (x y)
2
1
_
> 0
2
_
x
2
y
2
_
2
> xy 1 a < 1 : Tri gt
Vy p = 2 b loi.
Trng hp 2: p=1
d = m
_
c = d
2
, (i)
b = dn
(7.7) d
2
_
ad
2
+ 1
_
2
=
_
5d
2
+ 2dn
_ _
2d
2
+dn
_

_
ad
2
+ 1
_
2
= (5d + 2n) (2d +n) (7.8)
Din n Ton hc Chuyn S hc
7.2. c(ac + 1)
2
= (5c + 2)(2c +b) 139
(5d + 2n; 2d +n) = (d; 2d +n) = (d; n) = 1

_
5d + 2n = x
2
2d +n = y
2
_
x; y N

(x; y) = 1
_

_
d = x
2
2y
2
n = 5y
2
2x
2
Nu x = 2z vi z N


_
d = 4z
2
2y
2
n = 5y
2
8z
2
(7.8)
_
ad
2
+ 1
_
2
= 4z
2
y
2
a
_
4z
2
2y
2
_
2
+ 1 = 2zy
Phng trnh cui cng v nghim nguyn do 2 v khc tnh chn l.
Suy ra, x l d l c l. (ii)
Kt lun: (i), (ii) c l s chnh phng l.
Khng ngng tm kim, ta s tm mt li gii khc sc tch hn. Nu
ta bit n cng c v
p
(n) th s thy n s rt hiu qu cho bi ton
ny, ta c cch chng minh th v sau.
Chng minh (Chng minh 3). Gi s c chn khi ta c:
v
2
(c) = v
2
(5c + 2b) +v
2
(2c +b)
Nu b l th ta c v
2
(c) = v
2
(5c + 2b) = v
2
(5c) v
2
(5c) < v
2
(2b) = 1.
iu ny v l!
Do c l. Xt p[c l mt c nguyn t ca c.
Ta c v
p
(c) = v
p
(5c + 2b) +v
p
(2c +b).
Ta thy rng v
p
(c) > v
p
(5c + 2b), v
p
(2c +b) > 0.
Do v
p
(5c + 2b) = min[v
p
(c); v
p
(4c + 2b)]
v
p
(5c + 2b) = v
p
(4c + 2b) = v
p
(2c +b)
v
p
(c) = 2v
p
(5c + 2b): s chn nn suy ra c l s chnh phng.
V hi vng cn nhng li gii khc hay hn, sng to hn t cc bn.
Mong bn c tho lun thm v ng gp kin cho bi ton.
Li cm n
Rt cm n Karl Heinrich Marx,nguyenta98, Vng Nguyn Thy
Dng v perfectstrong ng gp kin cho bi vit ny.
Chuyn S hc Din n Ton hc
Ti liu tham kho
[1] V Hu Bnh,Phng trnh nghim nguyn v kinh nghim gii
[2] Phan Huy Khi, Cc chuyn bi dng hc sinh gii ton trung
hc. Chuyn 5: Phng trnh nghim nguyn
[3] Phm Minh Phng v nhm tc gi chuyn ton i hc S phm
H Ni, Cc chuyn S hc bi dng hc sinh gii Trung hc
c s
[4] Titu Andreescu, Dorin Andrica, Number Theory: Structures, Ex-
amples and Problems
[5] Tp ch Ton Tui Th, Ton hc v Tui tr, Mathematical Re-
flections, v.v
[6] Cc thi hc sinh gii, tuyn sinh vo THPT, TST, IMO,v.v
[7] Ti nguyn Internet, c bit:
http://diendantoanhoc.net/forum/,
http://www.artofproblemsolving.com/,
http://boxmath.vn
[8] Gv THPT chuyn HKHTN H Ni,Bi ging S hc
[9] ng Hng Thng, ng d v phng trnh ng d
[10] Phan Huy Khi, Cc bi ton c bn ca S hc
[11] H Huy Khoi, Chuyn bi dng HSG THPT S Hc
[12] K yu ca cc hi tho Ton hc, Tp ch Ton hc v Tui tr,
tp ch Crux,v.v
141
142 Ti liu tham kho
[13] Nguyn Trng Nam, L thuyt ng d v ng dng trong m sa
sai
Din n Ton hc Chuyn S hc

You might also like